You are on page 1of 228

JULITO SAGALES, Petitioner, - versus - RUSTANS COMMERCIAL

CORPORATION, Respondent.
THIRD DIVISION , G.R. No. 166554, November 27, 2008
REYES, R.T., J.:

Facts:
Petitioner Julito Sagales was employed by respondent Rustans Commercial
Corporation from October 1970 until July 26, 2001, when he was terminated.
At the time of his dismissal, he was occupying the position of Chief Cook at
the Yum Yum Tree Coffee Shop. He was paid a basic monthly salary of
P9,880.00. He was also receiving service charge of not less than P3,000.00 a
month and other benefits under the law and the existing collective
bargaining agreement between respondent and his labor union.
In the course of his employment, petitioner was a consistent recipient of
numerous citations for his performance. After receiving his latest award on
March 27, 2001, petitioner conveyed to respondent his intention of retiring
on October 31, 2001, after reaching thirty-one (31) years in service.
Petitioner, however, was not allowed to retire with his honor intacts.
On June 18, 2001, Security Guard Waldo Magtangob, upon instructions from
Senior Guard Bonifacio Aranas, apprehended petitioner in the act of taking
out from Rustans Supermarket a plastic bag. Upon examination, it was
discovered that the plastic bag contained 1.335 kilos of squid heads worth
P50.00. Petitioner was not able to show any receipt when confronted. Thus,
he was brought to the Security Office of respondent corporation for proper
endorsement to the Makati Headquarters of the Philippine National Police.
Subsequently, petitioner was brought to the Makati Police Criminal
Investigation Division where he was detained. Petitioner was later ordered
released pending further investigation.
On June 19, 2001, petitioner underwent inquest proceedings for qualified
theft before Assistant Prosecutor Amado Y. Pineda. Although petitioner
admitted that he was in possession of the plastic bag containing the squid
heads, he denied stealing them because he actually paid for them. As proof,
petitioner presented a receipt. The only fault he committed was his failure to
immediately show the purchase receipt when he was accosted because he
misplaced it when he changed his clothes. He also alleged that the squid
heads were already scraps as these were not intended for cooking. Neither
were the squid heads served to customers. He bought the squid heads so
that they could be eaten instead of being thrown away. If he intended to

steal from respondent, he could have stolen other valuable items instead of
scrap.
Assistant Prosecutor Pineda believed the version of petitioner and
recommended the dismissal of the case for lack of evidence. The
recommendation was approved upon review by the City Prosecutor.
Notwithstanding the dismissal of the complaint, respondent, on June 25,
2001, required petitioner to explain in writing within forty-eight (48) hours
why he should not be terminated in view of the June 18, 2001 incident.
Respondent also placed petitioner under preventive suspension.
Respondent did not find merit in the explanation of petitioner. Thus,
petitioner was dismissed from service on July 26, 2001. At that time,
petitioner had been under preventive suspension for one (1) month.
Aggrieved, petitioner filed a complaint for illegal dismissal against
respondent. He also prayed for unpaid salaries/wages, overtime pay, as well
as moral and exemplary damages, attorneys fees, and service charges.
Issues:
(1) Is the evidence on record sufficient to conclude that petitioner committed
the crime charged? and
(2) Assuming that the answer is in the affirmative, is the penalty of dismissal
proper?
Held:
1. YES. The evidence on record is sufficient to conclude that petitioner
committed the crime charged.
Security of tenure is a paramount right of every employee that is held sacred
by the Constitution. The reason for this is that labor is deemed to be
property within the meaning of constitutional guarantees. Indeed, as it is
the policy of the State to guarantee the right of every worker to security of
tenure as an act of social justice, such right should not be denied on mere
speculation of any similar or unclear nebulous basis. Indeed, the right of
every employee to security of tenure is all the more secured by the Labor
Code by providing that the employer shall not terminate the services of an
employee except for a just cause or when authorized by law. Otherwise, an
employee who is illegally dismissed shall be entitled to reinstatement
without loss of seniority rights and other privileges and to his full backwages,

inclusive of allowances, and to his other benefits or their monetary


equivalent computed from the time his compensation was withheld from him
up to the time of his actual reinstatement.
Necessarily then, the employer bears the burden of proof to show the basis
of the termination of the employee.
In the case at bar, respondent has discharged its onus of proving that
petitioner committed the crime charged.
We stress that the quantum of proof required for the application of the loss of
trust and confidence rule is not proof beyond reasonable doubt. It is
sufficient that there must only be some basis for the loss of trust and
confidence or that there is reasonable ground to believe, if not to entertain
the moral conviction, that the employee concerned is responsible for the
misconduct and that his participation in the misconduct rendered him
absolutely unworthy of trust and confidence.
It is also of no moment that the criminal complaint for qualified theft against
petitioner was dismissed. It is well settled that the conviction of an employee
in a criminal case is not indispensable to the exercise of the employers
disciplinary authority.
2. NO. The penalty of dismissal is too harsh under the circumstances. The
free will of management to conduct its own business affairs to achieve its
purpose cannot be denied. The only condition is that the exercise of
management prerogatives should not be done in bad faith or with abuse of
discretion. Truly, while the employer has the inherent right to discipline,
including that of dismissing its employees, this prerogative is subject to the
regulation by the State in the exercise of its police power.
In this regard, it is a hornbook doctrine that infractions committed by an
employee should merit only the corresponding penalty demanded by the
circumstance. The penalty must be commensurate with the act, conduct or
omission imputed to the employee and must be imposed in connection with
the disciplinary authority of the employer.
We do not condone dishonesty. After all, honesty is the best policy. However,
punishment should be commensurate with the offense committed. The
supreme penalty of dismissal is the death penalty to the working man. Thus,
care should be exercised by employers in imposing dismissal to erring
employees. The penalty of dismissal should be availed of as a last resort.

SOLVIC INDUSTRIAL CORP. and ANTONIO C. TAM, petitioners, vs.


NATIONAL LABOR RELATIONS COMMISSION and DIOSDADO LAUZ,
respondents.
[G.R. No. 125548. September 25, 1998] FIRST DIVISION
PANGANIBAN, J.:
Facts:
Complainant started his employment with respondent sometime in 1977. He
occupied the position as extruder operator. In the course of his employment,
he performed his utmost best, and in fact has never been suspended or
reprimanded. On 17 January 1994, sans cause or due process, he was
arbitrarily terminated from service. Additionally, complainant alleged that he
was not paid his service leave pay.
Respondent on the other hand, averred that the complainant who was hired
in 1977 was actually terminated for cause. That the termination of
complainant arose from the incident that transpired on 17 January 1994 at
about 7:00 p.m. On said occasion, complainant upon seeing Foreman Carlos
Aberin confronted him and thereafter struck him in the shoulder beside the
neck with a bladed weapon in the process, inflicting bodily injury on him.
That several days after said incident, complainant did not report for work,
hence,
was
issued
a
memorandum
of
preventive
suspension.
Correspondingly, Mr. Aberin executed an affidavit and submitted a medical
certificate.
Complainant on the other hand, submitted his letter of explanation denying
complicity in the acts imputed to him. Thereafter, a series of administrative
investigation was conducted where complainant refused to give any further
statement or explanation. Subsequently, he was served his letter of
termination dated 21 February 1994, which however, he refused to receive.
Relatedly, in a meeting/conference held with the union officers by Carlos
Aberin and Diosdado Lauz, complainant admitted to attempting to take the
life of Mr. Aberin and apologized for the same.

In reply, complainant countered that he never struck Mr. Aberin with a bladed
weapon, and that the incident was not job related, hence cannot serve as
basis for termination.
Respondents, on the other hand in reply, argued that complainant was given
his day in court as an investigation was conducted. Moreover, complainant in
the course of his meeting with Mr. Aberin and with the union officers,
admitted that he assaulted the latter and even apologized in exchange for
the withdrawal of the criminal case filed against him.
The NLRC found that the wrong imputed to the private respondent did not
merit the penalty of dismissal. Thus, ordering his reinstatement, but omitting
the award of back wages. It held that the imposition of the supreme penalty
of dismissal is not commensurate with the gravity of the offense he
committed.
Issue:
Whether or not the reinstatement of private respondent is proper.
Held:
YES. Petitioners arguments are not persuasive. Fighting within work
premises may be deemed a valid ground for the dismissal of an employee.
Such act adversely affects the employers interests for it distracts
employees, disrupts operations and creates a hostile work atmosphere. The
facts of this case, however, do not justify the dismissal of private respondent.
As found by Respondent NLRC, the infraction was committed outside the
work premises and did not lead to any disruption of work or any hostile
environment in the work premises.
We agree with the NLRC that the acts of private respondent are not so
serious as to warrant the extreme penalty of dismissal. Private respondent
was accused of hitting the victim once with the blunt side of a bolo. Private
respondent could have attacked him with the blade of the weapon, and he
could have struck him several times. But he did not, thus negating any intent
on his part to inflict fatal injuries. In fact, the victim merely sustained a minor
abrasion and has since forgiven and reconciled with the private respondent.
If the party most aggrieved -- namely, the foreman -- has already forgiven
the private respondent, then petitioner cannot be more harsh and
condemning than the victim. Besides, no criminal or civil action has been

instituted against private respondent. Furthermore, in his twenty years of


service in the company, he has not been charged with any similar
misconduct.
Verily, we do not condone the action of the private respondent. We believe,
however, that the NLRC did not commit grave abuse of discretion in ruling
that the penalty of dismissal was too harsh and not commensurate with the
said offense. Where a penalty less punitive would suffice, whatever
missteps may be committed by labor ought not to be visited with a
consequence so severe.
In so ruling, we reiterate that an employers power to discipline its workers
must be exercised with caution, lest it erode the constitutional guarantee of
security of tenure. This is especially true when the penalty being imposed is
dismissal, which leads to severance of employment ties and the economic
dislocation of the employee. Because of the serious implications of this
penalty, our Labor Code decrees that an employee cannot be dismissed,
except for the most serious causes. The overly concern of our laws for the
welfare of employees is in accord with the social justice philosophy.

ELMER M. MENDOZA, petitioner, vs. RURAL BANK OF LUCBAN,


respondent.
G.R. No. 155421 , July 7, 2004, FIRST DIVISION
PANGANIBAN, J.:
Facts:
On April 25, 1999, the Board of Directors of the Rural Bank of Lucban, Inc.,
issued several Board Resolutions which states that all officers and employees
are subject to reshuffle of assignments. Moreover, this resolution does not
preclude the transfer of assignment of bank officers and employees from the
branch office to the head office and vice-versa.
Pursuant to such Resolution, petitioner was one of the employees who were
reshuffled to a new assignment without changes in their compensation and
other benefits. He was assigned as an appraiser and later assigned as a
Clerk-Meralco Collection.
The new assignments were to be effective on May 1, 1999 without changes
in salary, allowances, and other benefits received by the aforementioned
employees. However, in a letter, petitioner wrote to the management about
such assignment. In his letter, he averred that his assignment is tantamount
to a demotion without any legal basis.
In its reply, the respondent informed petitioner that it was never the
intention of management to downgrade his position in the bank considering
that his due compensation as Bank Appraiser is maintained and no future
reduction was intended. The respondent also said that the conduct of
reshuffle is also a prerogative of bank management.
On June 7, 1999, petitioner applied for a leave of absence from work for ten
(10) days. On June 21, 1999, petitioner again submitted a letter asking for
another leave of absence for twenty days effective on the same date.

On June 24, 1999, while on his second leave of absence, petitioner filed a
Complaint before the NLRC for illegal dismissal, underpayment, separation
pay and damages against the Rural Bank of Lucban and/or its president,
Alejo B. Daya.
The labor arbiter held that respondent is guilty of illegal dismissal. It also
ordered the reinstatement of the complainant to his former position without
loss of seniority rights with full backwages.
On appeal, the NLRC reversed the labor arbiter and held that there was no
bad faith or malice to the respondent bank for its implementation of its Board
Resolution directing the reshuffle of employees at its Tayabas branch to
positions other than those they were occupying.
After the NLRC denied his Motion for Reconsideration, petitioner brought
before the CA. The CA held that there was no grave abuse of discretion
committed by the NLRC in issuing its decision. It ruled thus that when
Mendoza was reshuffled to the position of clerk at the bank, he was not
demoted as there was no diminution of his salary benefits and rank. The
reshuffling of its employees was done in good faith and cannot be made the
basis of a finding of constructive dismissal. The fact that Mendoza was no
longer included in the bank's payroll for July 1 to 15, 1999 does not signify
that the bank has dismissed the former from its employ. Mendoza separated
himself from the bank's employ when, on June 24, 1999, while on leave, he
filed the illegal dismissal case against his employer for no apparent reason at
all. Hence, this Petition.
Issue:
Whether or not the reshuffling of private respondent's employees was done
in good faith and cannot be made as the basis of a finding of constructive
dismissal, even as the petitioner's demotion in rank is admitted by both
parties.
Held:
NO. Jurisprudence recognizes the exercise of management prerogatives. For
this reason, courts often decline to interfere in legitimate business decisions
of employers. Indeed, labor laws discourage interference in employers'

judgments concerning the conduct of their business. The law must protect
not only the welfare of employees, but also the right of employers.
In the pursuit of its legitimate business interest, management has the
prerogative to transfer or assign employees from one office or area of
operation to another provided there is no demotion in rank or diminution of
salary, benefits, and other privileges; and the action is not motivated by
discrimination, made in bad faith, or effected as a form of punishment or
demotion without sufficient cause.
This privilege is inherent in the right of employers to control and manage
their enterprise effectively. The right of employees to security of tenure does
not give them vested rights to their positions to the extent of depriving
management of its prerogative to change their assignments or to transfer
them.
Managerial prerogatives, however, are subject to limitations provided by law,
collective bargaining agreements, and general principles of fair play and
justice. The test for determining the validity of the transfer of employees was
explained in Blue Dairy Corporation v. NLRC as follows:
"Like other rights, there are limits thereto. The managerial prerogative to
transfer personnel must be exercised without grave abuse of discretion,
bearing in mind the basic elements of justice and fair play. Having the right
should not be confused with the manner in which that right is exercised.
Thus, it cannot be used as a subterfuge by the employer to rid himself of an
undesirable worker. In particular, the employer must be able to show that the
transfer is not unreasonable, inconvenient or prejudicial to the employee;
nor does it involve a demotion in rank or a diminution of his salaries,
privileges and other benefits. Should the employer fail to overcome this
burden of proof, the employee's transfer shall be tantamount to constructive
dismissal, which has been defined as a quitting because continued
employment is rendered impossible, unreasonable or unlikely; as an offer
involving a demotion in rank and diminution in pay. Likewise, constructive
dismissal exists when an act of clear discrimination, insensibility or disdain
by an employer has become so unbearable to the employee leaving him with
no option but to forego with his continued employment."

PHILIPPINE TELEGRAPH AND TELEPHONE CORPORATION, petitioner,


vs. ALICIA LAPLANA, Hon. RICARDO ENCARNACION, and NATIONAL
LABOR RELATIONS COMMISSION, respondents.
G.R. No. 76645 , July 23, 1991, FIRST DIVISION
NARVASA, J.
Facts:
Alicia Laplana was the cashier of the Baguio City Branch Office of the
Philippine Telegraph and Telephone Corporation (hereafter, simply PT & T).
Sometime in March 1984, PT & T's treasurer, Mrs. Alicia A. Arogo, directed
Laplana to transfer to the company's branch office at Laoag City. Laplana
refused the reassignment and proposed instead that qualified clerks in the
Baguio Branch be trained for the purpose. She set out her reasons therefor in
her letter that she have already established Baguio City as her permanent
residence and that working in Laoag will involve additional expenses for her
part like her board and lodgingly, fare, and other miscellaneous expenses. As
a result, her salary alone will not be enough for her family.
On April 12, 1984, Mrs. Arogo reiterated her directive for Laplana's transfer
to the Laoag Branch, this time in the form of a written Memorandum,
informing Laplana that effective April 16, 1984, she will be reassigned to
Laoag branch assuming the same position of branch cashier and ordering her
to turn over your accountabilities such as PCF, undeposited collections, used
and unused official receipts, other accountable forms and files to Rose
Caysido who will be in charge of cashiering in Baguio.

Apparently Laplana was not allowed to resume her work as Cashier of the
Baguio Branch when April 16, 1984 came. She thereupon wrote again to Mrs.
Arogo advising that the directed transfer was unacceptable, reiterating the
reasons already given by her in her first letter. Subsequently, Laplana
received a telegram from Mrs. Arogo ordering her to report to Manila for a
ner job assignment but the same was refused by petitioner and asked that
she be retrenched instead.
Termination of Laplana's employment on account of retrenchment thereupon
followed. On July 4, 1984, Laplana signed the quitclaim and received the
check representing her 13th month and separation pay.
On October 9, 1984, Laplana filed with the Labor Arbiters' Office at Baguio
City, thru the CLAO, a complaint against PT & T its Baguio Northwestern
Luzon Branch, Baguio City, and Paraluman Bautista, Area Manager. In her
complaint she alleged, as right of action, that when she insisted on her right
of refusing to be transferred, the Defendants made good its warning by
terminating her services on May 16, 1984 on alleged ground of
"retrenchment," although the truth is, she was forced to be terminated and
that there was no ground at all for the retrenchment; that the company's act
of transferring is not only without any valid ground but also arbitrary and
without any purpose but to harass and force her to eventually resign.
In answer, the defendants alleged that Laplana was being transferred to
Laoag City because of increase in sales due to the additional installations of
vodex line; they also alleged that the company was exercising management
prerogatives in transferring complainant and there is no showing that this
exercise was arbitrarily and whimsically done; Lastly, Laplana's services were
terminated on her explicit declaration that "she was willing to be retrenched
rather than be assigned to Laoag City or Manila;"
Issue:
Whether or not Laplanas termination is legal.
Held:
YES. In Philippine Japan Active Carbon Corp. v. NLRC, promulgated on March
8, 1989 the Court held that it is the employer's prerogative, based on its
assessment and perception of its employees' qualifications, aptitudes, and

competence, to move them around in the various areas of its business


operations in order to ascertain where they will function with maximum
benefit to the company. An employee's right to security of tenure does not
give him such a vested right in his position as would deprive the company of
its prerogative to change his assignment or transfer him where he will be
most useful. When his transfer is not unreasonable, nor inconvenient, nor
prejudicial to him, and it does not involve a demotion in rank or diminution of
his salaries, benefits, and other privileges, the employee may not complain
that it amounts to a constructive dismissal.
In this case, the employee (Laplana) had to all intents and purposes resigned
from her position. She had unequivocally asked that she be considered
dismissed, herself suggesting the reason therefor retrenchment. When so
dismissed, she accepted separation pay. On the other hand, the employer
has not been shown to be acting otherwise than in good faith, and in the
legitimate pursuit of what it considered its best interests, in deciding to
transfer her to another office. There is no showing whatever that the
employer was transferring Laplana to another work place, not because she
would be more useful there, but merely as a subterfuge to rid itself of an
undesirable worker, or to penalize an employee for union activities. The
employer was moreover not unmindful of Laplana's initial plea for
reconsideration of the directive for her transfer to Laoag; in fact, in response
to that plea not to be moved to the Laoag Office, the employer opted instead
to transfer her to Manila, the main office, offering at the same time the
normal benefits attendant upon transfers from an office to another.
The situation here presented is of an employer transferring an employee to
another office in the exercise of what it took to be sound business judgment
and in accordance with pre-determined and established office policy and
practice, and of the latter having what was believed to be legitimate reasons
for declining that transfer, rooted in considerations of personal convenience
and difficulties for the family. Under these circumstances, the solution
proposed by the employee herself, of her voluntary termination of her
employment and the delivery to her of corresponding separation pay, would
appear to be the most equitable. Certainly, the Court cannot accept the
proposition that when an employee opposes his employer's decision to
transfer him to another work place, there being no bad faith or underhanded
motives on the part of either party, it is the employee's wishes that should
be made to prevail.

The termination of services of private respondent is declared legal and


proper.

ALLIED BANKING CORPORATION, petitioner, vs. COURT OF APPEALS


and POTENCIANO L. GALANIDA,respondents.
[G.R. No. 144412. November 18, 2003] FIRST DIVISION
CARPIO, J.:

Facts:
Private respondent Potenciano Galanida was hired by petitioner Allied
Banking Corporation on 11 January 1978 and rose from accountant-book
keeper to assistant manager in 1991. His appointment was covered by a
notice of personnel action which provided as one of the conditions of
employment the provision on petitioners right to transfer employees as the
need arises and in the interest of maintaining smooth and uninterrupted
service to the public.
Private respondent was promoted several times and was transferred to
several branches.
Effecting a rotation/movement of officers assigned in the Cebu homebase,
petitioner listed respondent as second in the order of priority of assistant
managers to be assigned outside of Cebu City having been stationed in Cebu

for seven years already. Private respondent manifested his refusal to be


transferred to Bacolod City in a letter citing as reason parental obligations,
expenses, and the anguish that would result if he is away from his family. He
then filed a complaint before the Labor Arbiter for constructive dismissal.
Subsequently, petitioner bank informed private respondent that he was to
report to the Tagbilaran City Branch effective 23 May 1994. Private
respondent refused. Subsequently, petitioner warned and required of private
respondent that tthere is no discrimination in her transfer. Petitioner also
invoked its right to transfer employees as the need arises and in the interest
of maintaining smooth and uninterrupted service to the public.
Petitioner also warned respondent that her refusal to follow instruction
concerning his transfer and reassignment to Bacolod City and to Tagbilaran
City is penalized under Article XII of the Banks Employee Discipline Policy
and Procedure. In case of refusal, the penalty may range from suspension to
dismissal as determined by management.
Respondent was also given an opportunity to explain why no disciplinary
action should be meted against him for refusing to follow instructions
concerning the transfer and reassignment.
On 16 June 1994, Galanida replied that whether the banks penalty for his
refusal be Suspension or Dismissal it will all the more establish and fortify his
complaint now pending before the NLRC. In the same letter, he charged
Allied Bank with discrimination and favoritism in ordering his transfer.
On 5 October 1994, Galanida received an inter-office communication stating
that Allied Bank had terminated his services effective 1 September 1994 for
his continued refusal to be transferred from the Jakosalem, Cebu City branch
After several hearings, the Labor Arbiter held that Allied Bank had abused its
management prerogative in ordering the transfer of Galanida to its Bacolod
and Tagbilaran branches. In ruling that Galanidas refusal to transfer did not
amount to insubordination. The Labor Arbiter reasoned that Galanidas
transfer was inconvenient and prejudicial because Galanida would have to
incur additional expenses for board, lodging and travel. On the other hand,
the Labor Arbiter held that Allied Bank failed to show any business urgency
that would justify the transfer.
On appeal, the NLRC likewise ruled that Allied Bank terminated Galanida
without just cause. The NLRC agreed that the transfer order was
unreasonable and unjustified, considering the family considerations

mentioned by Galanida. The NLRC characterized the transfer as a demotion


since the Bacolod and Tagbilaran branches were smaller than the Jakosalem
branch, a regional office, and because the bank wanted Galanida, an
assistant manager, to replace an assistant accountant in the Tagbilaran
branch. The NLRC found unlawful discrimination since Allied Bank did not
transfer several junior accountants in Cebu.
Allied Bank filed a motion for reconsideration which the NLRC denied.
Dissatisfied, Allied Bank filed a petition for review questioning the Decision
and Resolution of the NLRC before the Court of Appeals. The Court of Appeals
held that Galanidas refusal to comply with the transfer orders did not
warrant his dismissal. The appellate court ruled that the transfer from a
regional office to the smaller Bacolod or Tagbilaran branches was effectively
a demotion. The appellate court agreed that Allied Bank did not afford
Galanida procedural due process because there was no hearing and no
notice of termination. The Memo merely stated that the bank would issue a
notice of termination but there was no such notice.
Allied Bank filed a motion for reconsideration which the appellate court
denied. Hence this petition.
Issue:
Whether under the facts presented there is legal basis in petitioners
exercise of its management prerogative.
Held:
YES. The rule is that the transfer of an employee ordinarily lies within the
ambit of the employers prerogatives. The employer exercises the
prerogative to transfer an employee for valid reasons and according to the
requirement of its business, provided the transfer does not result in demotion
in rank or diminution of the employees salary, benefits and other privileges.
In illegal dismissal cases, the employer has the burden of showing that the
transfer is not unnecessary, inconvenient and prejudicial to the displaced
employee.
The constant transfer of bank officers and personnel with accounting
responsibilities from one branch to another is a standard practice of Allied
Bank, which has more than a hundred branches throughout the country.
Allied Bank does this primarily for internal control. It also enables bank
employees to gain the necessary experience for eventual

promotion. The Bangko Sentral ng Pilipinas, in its Manual of Regulations for


Banks and Other Financial Intermediaries, requires the rotation of these
personnel. The Manual directs that the duties of personnel handling cash,
securities and
bookkeeping records should be rotated and that such rotation should be
irregular, unannounced and long enough to permit disclosure of any
irregularities or manipulations.
Galanida was well aware of Allied Banks policy of periodically transferring
personnel to different branches. As the Court of Appeals found, assignment
to the different branches of Allied Bank was a condition of Galanidas
employment. Galanida consented to this condition when he signed the
Notice of Personnel Action.
The evidence on record contradicts the charge that Allied Bank discriminated
against Galanida and was in bad faith when it ordered his transfer. Allied
Banks letter of 13 June 1994 showed that at least 14 accounting officers and
personnel from various branches, including Galanida, were transferred to
other branches. Allied Bank did not single out Galanida. The same letter
explained that Galanida was second in line for assignment outside Cebu
because he had been in Cebu for seven years already. The person first in
line, Assistant Manager Roberto Isla, who had been in Cebu for more than ten
years, had already transferred to a branch in Cagayan de Oro City. We note
that none of the other transferees joined Galanida in his complaint or
corroborated his allegations of widespread discrimination and favoritism.
The employer has the prerogative, based on its assessment of the
employees qualifications and competence, to rotate them in the various
areas of its business operations to ascertain where they will function with
maximum benefit to the company.
Neither was Galanidas transfer in the nature of a demotion. Galanida did not
present evidence showing that the transfer would diminish his salary,
benefits or other privileges. Instead, Allied Bank assured Galanida that he
would not suffer any reduction in rank or grade, and that the transfer would
involve the same rank, duties and obligations.

GENUINE ICE COMPANY INC., Petitioner, - versus ALFONSO S.


MAGPANTAY, Respondent.
G.R. No. 147790, June 27, 2006, FIRST DIVISION
AUSTRIA-MARTINEZ, J.:
Facts:
Alfonso Magpantay (respondent) was employed as a machine operator with
Genuino Ice Company, Inc. (petitioner) from March 1988 to December 1995.
On November 18, 1996, respondent filed against petitioner a complaint for
illegal dismissal alleging that he was dismissed from service effective
immediately by virtue of a memorandum, after which he was not allowed
anymore to enter the company premises. Respondent bewailed that his
termination from employment was done without due process.
Petitioner countered that he was not illegally dismissed, since the dismissal
was based on a valid ground, i.e., he led an illegal strike at petitioners sister
company, Genuino Agro Industrial Development Corporation, which lasted
from November 18 to 22, 1995, resulting in big operation losses on the
latters part. Petitioner also maintained that respondents dismissal was
made after he was accorded due process.

Respondent replied, however, that assuming that he led such illegal strike,
he could not be liable therefore because it was done in petitioners sister
company which is a separate and distinct entity from petitioner.
Petitioner initially claimed that respondents acts were tantamount to serious
misconduct or willful disobedience, gross and habitual neglect of duties, and
breach of trust. Subsequently, petitioner amended its position paper to
include insubordination among the grounds for his dismissal, since it came
out during respondents cross-examination, and the matter was reported
only after the new personnel manager assumed his position in August 1996.
Issue:
Whether or not the respondent was illegally dismissed from employment.
Held:
No.
Both the Labor Arbiter and the NLRC were one in concluding that petitioner
had just cause for dismissing respondent, as his act of leading a strike at
petitioners company for four days, his absence from work during such time,
and his failure to perform his duties during such absence, make up a cause
for habitual neglect of duties, while his failure to comply with petitioners
order for him to transfer to the GMA, Cavite Plant constituted insubordination
or willful disobedience. The CA, however, differed with said conclusion and
found that respondents attitude has not been proved to be visited with any
wrongdoing, and that his four-day absence does not appear to be both gross
and habitual.
The Court sustains the CAs finding that respondents four-day absence does
not amount to a habitual neglect of duty; however, the Court finds that
respondent was validly dismissed on ground of willful disobedience or
insubordination.
Under Article 282 of the Labor Code, as amended, an employer may
terminate an employment for any of the following causes: (a) serious
misconduct or willful disobedience by the employee of the lawful orders of
his employer or representative in connection with his work; (b) gross and
habitual neglect by the employee of his duties; (c) fraud or willful breach by
the employee of the trust reposed in him by his employer or duly authorized
representative; (d) commission of a crime or offense by the employee
against the person of his employer or any immediate member of his family or

his duly authorized representative; and, (e) other causes analogous to the
foregoing. The employer has the burden of proving that the dismissal was for
a just cause; failure to show this would necessarily mean that the dismissal
was unjustified and, therefore, illegal.
Neglect of duty, to be a ground for dismissal, must be both gross and
habitual. Gross negligence connotes want of care in the performance of ones
duties. Habitual neglect implies repeated failure to perform ones duties for a
period of time, depending upon the circumstances. On the other hand, fraud
and willful neglect of duties imply bad faith on the part of the employee in
failing to perform his job to the detriment of the employer and the latters
business. Thus, the single or isolated act of negligence does not constitute a
just cause for the dismissal of the employee.

PHILIPPINE DUPLICATORS, INC., petitioner, vs. NATIONAL LABOR


RELATIONS COMMISSION and PHILIPPINE DUPLICATORS EMPLOYEES
UNION-TUPAS,respondents.
G.R. No. 110068 February 15, 1995, EN BANC
FELICIANO, J.:

Facts:
This Case differentiates Productivity Bonuses and Commissions Productivity
bonuses are generally tied to the productivity or profit generation of the
employer corporation. Productivity bonuses are not directly dependent on
the extent an individual employee exerts himself. A productivity bonus is
something extra for which no specific additional services are rendered by any
particular employee and hence not legally demandable, absent a contractual
undertaking to pay it. Sales commissions are intimately related to or directly
proportional to the extent or energy of an employee's endeavours.
Commissions are paid upon the specific results achieved by a salesman-

employee. It is a percentage of the sales closed by a salesman and operates


as an integralpart of such salesman's basic pay.
Issue:
1.WON The commissions received by the salesmen were part of thewages
to be considered for their 13thmonth pay. Yes
2.WON Productivity bonus shall be considered as part of wages in 13th
month pay No
Held:
1.The commissions were an integral part of the pay of the workers,
considering that the fixed wage was only 30% of what they were normally
receiving.
2.Productivity bonuses are generally tied to the productivity, or capacity for
revenue production, of a corporation; such bonuses closely resemble profitsharing payments and have no clear director necessary relation to the
amount of work actually done by each individual employee. More generally,
a bonus is an amount granted and paid ex gratia to the employee; its
payment constitutes an act of enlightened generosity and self-interest on the
part of the employer, rather than as a demandable or enforceable obligation.
Since productivity bonus is not demandable, then it cannot be considered
part of basic salary when time comes to compute 13th month pay. Additional
payments made to employees, to the extent they partake of the nature of
profit-sharing payments, are properly excluded from the ambit of the term
"basic salary" for purposes of computing the 13th month pay due to
employees. Such additional payments are not "commissions" within the
meaning of the second paragraph of Section 5 (a) of the Revised Guidelines
Implementing13th Month Pay. The Supplementary Rules and Regulations
Implementing P.D. No.851 subsequently issued by former Labor Minister Ople
sought to clarify the scope of items excluded in the computation of the
13thmonth pay; viz .:Sec. 4. Overtime pay, earnings and other
remunerations which are not part of the basic salary shall not be included in
the computation of the 13th month pay.

EASTERN TELECOMMUNICATIONS PHILIPPINES, INC., Petitioner,versus - EASTERN TELECOMS EMPLOYEES UNION, Respondent.
February 8, 2012, G.R. No. 185665, THIRD DIVISION
MENDOZA, J.:
Facts:
Eastern Telecommunications Phils., Inc. (ETPI) is a corporation engaged in the
business of providing telecommunications facilities, particularly leasing
international date lines or circuits, regular landlines, internet and data
services, employing approximately 400 employees.
Eastern Telecoms Employees Union (ETEU) is the certified exclusive
bargaining agent of the companys rank and file employees with a strong
following of 147 regular members. It has an existing collective bargaining

agreement with the company to expire in the year 2004 with a Side
Agreement signed on September 3, 2001.
The Eastern Telecoms Employees Union (ETEU) claimed that Eastern
Telecommunications Philippines, Inc. (ETPI) had consistently and voluntarily
been giving out 14th month bonus during the month of April, and 15th and
16th month bonuses every December of each year (subject bonuses) to its
employees from 1975 to 2002, even when it did not realize any net profits.
ETEU posited that by reason of its long and regular concession, the payment
of these monetary benefits had ripened into a company practice which could
no longer be unilaterally withdrawn by ETPI. ETEU added that this longstanding company practice had been expressly confirmed in the Side
Agreements of the 1998-2001 and 2001-2004 Collective Bargaining
Agreements(CBA) which provided for the continuous grant of these bonuses
in no uncertain terms. ETEU theorized that the grant of the subject bonuses
is not only a company practice but also a contractual obligation of ETPI to the
union members.
ETEU contended that the unjustified and malicious refusal of the company to
pay the subject bonuses was a clear violation of the economic provision of
the CBA and constitutes unfair labor practice (ULP). According to ETEU, such
refusal was nothing but a ploy to spite the union for bringing the matter of
delay in the payment of the subject bonuses to the National Conciliation and
Mediation Board (NCMB). It prayed for the award of moral and exemplary
damages as well as attorneys fees for the unfair labor practice allegedly
committed by the company.
The NLRC dismissed ETEUs complaint and held that ETPI could not be forced
to pay the union members the 14th, 15th and 16th month bonuses for the
year 2003 and the 14th month bonus for the year 2004 inasmuch as the
payment of these additional benefits was basically a management
prerogative, being an act of generosity and munificence on the part of the
company and contingent upon the realization of profits. The NLRC
pronounced that ETPI may not be obliged to pay these extra compensations
in view of the substantial decline in its financial condition. Likewise, the NLRC
found that ETPI was not guilty of the ULP charge elaborating that no
sufficient and substantial evidence was adduced to attribute malice to the
company for its refusal to pay the subject bonuses.
Respondent ETEU moved for reconsideration but the motion was denied by
the NLRC.

Aggrieved, ETEU filed a petition for certiorari before the CA ascribing grave
abuse of discretion on the NLRC for disregarding its evidence which allegedly
would prove that the subject bonuses were part of the union members
wages, salaries or compensations.
The CA declared that the Side Agreements of the 1998 and 2001 CBA
created a contractual obligation on ETPI to confer the subject bonuses to its
employees without qualification or condition. It also found that the grant of
said bonuses has already ripened into a company practice and their denial
would amount to diminution of the employees benefits.
Issue:
1. Whether the members of ETEU are entitled to the payment of 14th, 15th
and 16th month bonuses for the year 2003 and 14th month bonus for year
2004.
2. Whether these bonuses are demandable or not. Stated differently, can
these bonuses be considered part of the wage, salary or compensation
making them enforceable obligations?
Held:
1. YES. From a legal point of view, a bonus is a gratuity or act of liberality of
the giver which the recipient has no right to demand as a matter of right. The
grant of a bonus is basically a management prerogative which cannot be
forced upon the employer who may not be obliged to assume the onerous
burden of granting bonuses or other benefits aside from the employees
basic salaries or wages.
A bonus, however, becomes a demandable or enforceable obligation when it
is made part of the wage or salary or compensation of the employee.
Particularly instructive is the ruling of the Court in Metro Transit Organization,
Inc. v. National Labor Relations Commission, where it was written:
Whether or not a bonus forms part of wages depends upon the
circumstances and conditions for its payment. If it is additional
compensation which the employer promised and agreed to give without any
conditions imposed for its payment, such as success of business or greater
production or output, then it is part of the wage. But if it is paid only if profits
are realized or if a certain level of productivity is achieved, it cannot be
considered part of the wage. Where it is not payable to all but only to some

employees and only when their labor becomes more efficient or more
productive, it is only an inducement for efficiency, a prize therefore, not a
part of the wage.
2. YES. A reading of the CBA provision reveals that the same provides for the
giving of 14th, 15th and 16th month bonuses without qualification. The
wording of the provision does not allow any other interpretation. There were
no conditions specified in the CBA Side Agreements for the grant of the
benefits contrary to the claim of ETPI that the same is justified only when
there are profits earned by the company. Terse and clear, the said provision
does not state that the subject bonuses shall be made to depend on the
ETPIs financial standing or that their payment was contingent upon the
realization of profits. Neither does it state that if the company derives no
profits, no bonuses are to be given to the employees. In fine, the payment of
these bonuses was not related to the profitability of business operations.
The records are also bereft of any showing that the ETPI made it clear before
or during the execution of the Side Agreements that the bonuses shall be
subject to any condition. Indeed, if ETPI and ETEU intended that the subject
bonuses would be dependent on the company earnings, such intention
should have been expressly declared in the Side Agreements or the bonus
provision should have been deleted altogether. In the absence of any proof
that ETPIs consent was vitiated by fraud, mistake or duress, it is presumed
that it entered into the Side Agreements voluntarily, that it had full
knowledge of the contents thereof and that it was aware of its commitment
under the contract.
Verily, by virtue of its incorporation in the CBA Side Agreements, the grant of
14th, 15th and 16th month bonuses has become more than just an act of
generosity on the part of ETPI but a contractual obligation it has undertaken.
Moreover, the continuous conferment of bonuses by ETPI to the union
members from 1998 to 2002 by virtue of the Side Agreements evidently
negates its argument that the giving of the subject bonuses is a
management prerogative.
The rule is settled that any benefit and supplement being enjoyed by the
employees cannot be reduced, diminished, discontinued or eliminated by the
employer. The principle of non-diminution of benefits is founded on the
constitutional mandate to protect the rights of workers and to promote their
welfare and to afford labor full protection.

MANILA JOCKEY CLUB EMPLOYEES LABOR UNION- PTGWO, Petitioner,


-versusMANILA JOCKEY CLUB, INC., Respondent.
G.R. No. 167760, March 7, 2007, FIRST DIVISION
GARCIA, J.:
Facts:
Petitioner Manila Jockey Club Employees Labor Union-PTGWO and respondent
Manila Jockey Club, Inc., a corporation with a legislative franchise to conduct,
operate and maintain horse races, entered into a Collective Bargaining

Agreement (CBA) effective January 1, 1996 to December 31, 2000. The CBA
governed the economic rights and obligations of respondents regular
monthly paid rank-and-file employees. In the CBA, the parties agreed to a 7hour work schedule from 9:00 a.m. to 12:00 noon and from1:00 p.m. to 5:00
p.m. on a work week of Monday to Saturday.
Accordingly, overtime on an ordinary working day shall be remunerated in an
amount equivalent to the worker's regular basic wage plus twenty five
percent (25%) thereof. Where the employee is permitted or suffered to work
on legally mandated holidays or on his designated rest day which is not a
legally mandated holiday, thirty percent (30%) shall be added to his basic
wage for a seven hour work; while work rendered in excess of seven hours
on legally mandated holidays and rest days not falling within the aforestated
categories day shall be additionally compensated for the overtime work
equivalent to his rate for the first seven hours on a legally mandated holiday
or rest day plus thirty percent (30%) thereof.
The CBA likewise reserved in respondent certain management prerogatives,
including the determination of the work schedule.
On April 3, 1999, respondent issued an inter-office memorandum declaring
that, effective April 20, 1999, the hours of work of regular monthly-paid
employees shall be from 1:00 p.m. to 8:00 p.m. when horse races are held,
that is, every Tuesday and Thursday. The memorandum, however,
maintained the 9:00 a.m. to 5:00 p.m. schedule for non-race days.
On October 12, 1999, petitioner and respondent entered into an Amended
and Supplemental CBA and clarified that any conflict arising therefrom shall
be referred to a voluntary arbitrator for resolution.
Subsequently, before a panel of voluntary arbitrators of the National
Conciliation and Mediation Board (NCMB), petitioner questioned the
memorandum as violative of the prohibition against non-diminution of wages
and benefits guaranteed under Section 1, Article IV, of the CBA which
specified the work schedule of respondent's employees to be from 9:00 a.m.
to 5:00 p.m. Petitioner claimed that as a result of the memorandum, the
employees are precluded from rendering their usual overtime work from 5:00
p.m. to 9:00 p.m.

The NCMBs panel of voluntary arbitrators upheld respondent's prerogative


to change the work schedule of regular monthly-paid employees. Petitioner
moved for reconsideration but the panel denied the motion.
Dissatisfied, petitioner then appealed the panels decision to the CA . The CA
upheld that of the panel and denied petitioners subsequent motion for
reconsideration.
Hence, petitioners present recourse,
Issues:
Whether or not the act of the company is changing the work schedule of the
employees is part of management prerogative.
Held:
YES. Respondent, as employer, cites the change in the program of horse
races as reason for the adjustment of the employees work schedule. It
rationalizes that when the CBA was signed, the horse races started at 10:00
a.m. When the races were moved to 2:00 p.m., there was no other choice for
management but to change the employees' work schedule as there was no
work to be done in the morning. Evidently, the adjustment in the work
schedule of the employees is justified.
We are not unmindful that every business enterprise endeavors to increase
profits. As it is, the Court will not interfere with the business judgment of an
employer in the exercise of its prerogative to devise means to improve its
operation, provided that it does not violate the law, CBAs, and the general
principles of justice and fair play. We have thus held that management is free
to regulate, according to its own discretion and judgment, all aspects of
employment, including hiring, work assignments, working methods, time,
place and manner of work, processes to be followed, supervision of workers,
working regulations, transfer of employees, work supervision, layoff of
workers and discipline, dismissal, and recall of workers.

DUNCAN ASSOCIATION OF DETAILMAN-PTGWO and PEDRO A.


TECSON, petitioners, vs. GLAXO WELLCOME PHILIPPINES, INC.,
Respondent.
G.R. No. 162994, September 17, 2004, SECOND DIVISION
TINGA, J.:
Facts:
Petitioner Pedro A. Tecson was hired by respondent Glaxo Wellcome
Philippines, Inc. as medical representative after Tecson had undergone
training and orientation.

Thereafter, Tecson signed a contract of employment which stipulates, among


others, that he agrees to study and abide by existing company rules; to
disclose to management any existing or future relationship by consanguinity
or affinity with co-employees or employees of competing drug companies
and should management find that such relationship poses a possible conflict
of interest, to resign from the company.
The Employee Code of Conduct of Glaxo similarly provides that an employee
is expected to inform management of any existing or future relationship by
consanguinity or affinity with co-employees or employees of competing drug
companies. If management perceives a conflict of interest or a potential
conflict between such relationship and the employees employment with the
company, the management and the employee will explore the possibility of a
"transfer to another department in a non-counterchecking position" or
preparation for employment outside the company after six months.
Tecson was initially assigned to market Glaxos products in the Camarines
Sur-Camarines Norte sales area.
Subsequently, Tecson entered into a romantic relationship with Bettsy, an
employee of Astra Pharmaceuticals (Astra), a competitor of Glaxo. Bettsy
was Astras Branch Coordinator in Albay. She supervised the district
managers and medical representatives of her company and prepared
marketing strategies for Astra in that area.
Even before they got married, Tecson received several reminders from his
District Manager regarding the conflict of interest which his relationship with
Bettsy might engender. Still, love prevailed, and Tecson married Bettsy in
September 1998.
In January 1999, Tecsons superiors informed him that his marriage to Bettsy
gave rise to a conflict of interest. Tecsons superiors reminded him that he
and Bettsy should decide which one of them would resign from their jobs,
although they told him that they wanted to retain him as much as possible
because he was performing his job well.
Tecson requested for time to comply with the company policy against
entering into a relationship with an employee of a competitor company. He
explained that Astra, Bettsys employer, was planning to merge with Zeneca,
another drug company; and Bettsy was planning to avail of the redundancy
package to be offered by Astra. With Bettsys separation from her company,

the potential conflict of interest would be eliminated. At the same time, they
would be able to avail of the attractive redundancy package from Astra.
In August 1999, Tecson again requested for more time resolve the problem.
In September 1999, Tecson applied for a transfer in Glaxos milk division,
thinking that since Astra did not have a milk division, the potential conflict of
interest would be eliminated. His application was denied in view of Glaxos
"least-movement-possible" policy.
In November 1999, Glaxo transferred Tecson to the Butuan City-Surigao CityAgusan del Sur sales area. Tecson asked Glaxo to reconsider its decision, but
his request was denied.
Tecson sought Glaxos reconsideration regarding his transfer and brought the
matter to Glaxos Grievance Committee. Glaxo, however, remained firm in its
decision and gave Tescon until February 7, 2000 to comply with the transfer
order. Tecson defied the transfer order and continued acting as medical
representative in the Camarines Sur-Camarines Norte sales area.
During the pendency of the grievance proceedings, Tecson was paid his
salary, but was not issued samples of products which were competing with
similar products manufactured by Astra. He was also not included in product
conferences regarding such products.
Because the parties failed to resolve the issue at the grievance machinery
level, they submitted the matter for voluntary arbitration. Glaxo offered
Tecson a separation pay of one-half () month pay for every year of service,
or a total of P50,000.00 but he declined the offer. On November 15, 2000,
the National Conciliation and Mediation Board (NCMB) rendered its Decision
declaring as valid Glaxos policy on relationships between its employees and
persons employed with competitor companies, and affirming Glaxos right to
transfer Tecson to another sales territory.
Aggrieved, Tecson filed a Petition for Review with the Court of Appeals
assailing the NCMB Decision.
The Court of Appeals denied the Petition for Review on the ground that the
NCMB did not err in rendering its Decision. The appellate court held that
Glaxos policy prohibiting its employees from having personal relationships

with employees of competitor companies is a valid exercise of its


management prerogatives.
Tecson filed a Motion for Reconsideration of the appellate courts Decision,
but the motion was denied by the appellate court.
Hence this petition.
Issue:
Whether the Glaxos policy against its employees marrying employees from
competitor companies is valid;
Held:
YES. No reversible error can be ascribed to the Court of Appeals when it ruled
that Glaxos policy prohibiting an employee from having a relationship with
an employee of a competitor company is a valid exercise of management
prerogative.
Glaxo has a right to guard its trade secrets, manufacturing formulas,
marketing strategies and other confidential programs and information from
competitors, especially so that it and Astra are rival companies in the highly
competitive pharmaceutical industry.
The prohibition against personal or marital relationships with employees of
competitor companies upon Glaxos employees is reasonable under the
circumstances because relationships of that nature might compromise the
interests of the company. In laying down the assailed company policy, Glaxo
only aims to protect its interests against the possibility that a competitor
company will gain access to its secrets and procedures.
That Glaxo possesses the right to protect its economic interests cannot be
denied. No less than the Constitution recognizes the right of enterprises to
adopt and enforce such a policy to protect its right to reasonable returns on
investments and to expansion and growth.20 Indeed, while our laws
endeavor to give life to the constitutional policy on social justice and the
protection of labor, it does not mean that every labor dispute will be decided
in favor of the workers. The law also recognizes that management has rights
which are also entitled to respect and enforcement in the interest of fair play.

PHILIPPINE TELEGRAPH AND TELEPHONE COMPANY, petitioner, vs.


NATIONAL LABOR RELATIONS COMMISSION and GRACE DE GUZMAN,
respondents.
G.R. No. 118978, May 23, 1997, SECOND DIVISION
REGALADO, J.
Facts:
Grace de Guzman was initially hired by petitioner as a reliever, specifically as
a "Supernumerary Project Worker," for a fixed period from November 21,
1990 until April 20, 1991 vice one C.F. Tenorio who went on maternity leave.
Under the Reliever Agreement which she signed with petitioner company,
her employment was to be immediately terminated upon expiration of the

agreed period. Thereafter, from June 10, 1991 to July 1, 1991, and from July
19, 1991 to August 8, 1991, private respondent's services as reliever were
again engaged by petitioner, this time in replacement of one Erlinda F. Dizon
who went on leave during both periods. After August 8, 1991, and pursuant
to their Reliever Agreement, her services were terminated.
On September 2, 1991, private respondent was once more asked to join
petitioner company as a probationary employee, the probationary period to
cover 150 days. In the job application form that was furnished her to be filled
up for the purpose, she indicated in the portion for civil status therein that
she was single although she had contracted marriage a few months earlier,
that is, on May 26, 1991.
It now appears that private respondent had made the same representation in
the two successive reliever agreements which she signed on June 10, 1991
and July 8, 1991. When petitioner supposedly learned about the same later,
its branch supervisor in Baguio City, Delia M. Oficial, sent to private
respondent a memorandum requiring her to explain the discrepancy. In that
memorandum, she was reminded about the company's policy of not
accepting married women for employment.
In her reply letter, private respondent stated that she was not aware of
PT&T's policy regarding married women at the time, and that all along she
had not deliberately hidden her true civil status. Petitioner nonetheless
remained unconvinced by her explanations. Private respondent was
dismissed from the company which she readily contested by initiating a
complaint for illegal dismissal, coupled with a claim for non-payment of cost
of living allowances (COLA), before the Regional Arbitration Branch of the
National Labor Relations Commission in Baguio City.
The Labor Arbiter declared that private respondent, who had already gained
the status of a regular employee, was illegally dismissed by petitioner. Her
reinstatement, plus payment of the corresponding back wages and COLA,
was correspondingly ordered.
On appeal to the National Labor Relations Commission (NLRC), said public
respondent upheld the labor arbiter and it ruled that private respondent had
indeed been the subject of an unjust and unlawful discrimination by her
employer, PT & T.
Issue:

Whether the act of the petitioner constituted a marital discrimination.


Held:
YES. It is recognized that regulation of manpower by the company falls within
the so-called management prerogatives, which prescriptions encompass the
matter of hiring, supervision of workers, work assignments, working methods
and assignments, as well as regulations on the transfer of employees, lay-off
of workers, and the discipline, dismissal, and recall of employees. As put in a
case, an employer is free to regulate, according to his discretion and best
business judgment, all aspects of employment, "from hiring to firing," except
in cases of unlawful discrimination or those which may be provided by law.
In the case at bar, petitioner's policy of not accepting or considering as
disqualified from work any woman worker who contracts marriage runs afoul
of the test of, and the right against, discrimination, afforded all women
workers by our labor laws and by no less than the Constitution. Contrary to
petitioner's assertion that it dismissed private respondent from employment
on account of her dishonesty, the record discloses clearly that her ties with
the company were dissolved principally because of the company's policy that
married women are not qualified for employment in PT & T, and not merely
because of her supposed acts of dishonesty.
In the present controversy, petitioner's expostulations that it dismissed
private respondent, not because the latter got married but because she
concealed that fact, does have a hollow ring. Her concealment, so it is
claimed, bespeaks dishonesty hence the consequent loss of confidence in
her which justified her dismissal.
STAR PAPER CORPORATION, JOSEPHINE ONGSITCO & SEBASTIAN
CHUA, Petitioners, -versus- RONALDO D. SIMBOL, WILFREDA N.
COMIA & LORNA E. ESTRELLA, Respondents.
G.R. No. 164774 , April 12, 2006, SECOND DIVISION
PUNO, J.:
Facts:
Petitioner Star Paper Corporation is a corporation engaged in trading
principally of paper products. Josephine Ongsitco is its Manager of the

Personnel and Administration Department while Sebastian Chua is its


Managing Director.
Respondents Ronaldo D. Simbol , Wilfreda N. Comia and Lorna E. Estrella
were all regular employees of the company.
Simbol was employed by the company on October 27, 1993. He met Alma
Dayrit, also an employee of the company, whom he married on June 27,
1998. Prior to the marriage, Ongsitco advised the couple that should they
decide to get married, one of them should resign pursuant to a company
policy.
Simbol resigned on June 20, 1998 pursuant to the company policy.
Comia was hired by the company on February 5, 1997. She met Howard
Comia, a co-employee, whom she married on June 1, 2000. Ongsitco likewise
reminded them that pursuant to company policy, one must resign should
they decide to get married. Comia resigned on June 30, 2000.
Estrella was hired on July 29, 1994. She met Luisito Zuiga , also a co-worker.
Petitioners stated that Zuiga, a married man, got Estrella pregnant. The
company allegedly could have terminated her services due to immorality but
she opted to resign on December 21, 1999.
The respondents each signed a Release and Confirmation Agreement. They
stated therein that they have no money and property accountabilities in the
company and that they release the latter of any claim or demand of
whatever nature.
Respondents offer a different version of their dismissal. Simbol and Comia
allege that they did not resign voluntarily; they were compelled to resign in
view of an illegal company policy. As to respondent Estrella, she alleges that
she had a relationship with co-worker Zuiga who misrepresented himself as
a married but separated man. After he got her pregnant, she discovered that
he was not separated. Thus, she severed her relationship with him to avoid
dismissal due to the company policy. On November 30, 1999, she met an
accident and was advised by the doctor at the Orthopedic Hospital to
recuperate for twenty-one (21) days. She returned to work on December 21,
1999 but she found out that her name was on-hold at the gate. She was
denied entry. She was directed to proceed to the personnel office where one

of the staff handed her a memorandum. The memorandum stated that she
was being dismissed for immoral conduct. She refused to sign the
memorandum because she was on leave for twenty-one (21) days and has
not been given a chance to explain. The management asked her to write an
explanation. However, after submission of the explanation, she was
nonetheless dismissed by the company. Due to her urgent need for money,
she later submitted a letter of resignation in exchange for her thirteenth
month pay.
Respondents later filed a complaint for unfair labor practice, constructive
dismissal, separation pay and attorneys fees. They averred that the
aforementioned company policy is illegal and contravenes Article 136 of the
Labor Code.
The Labor Arbiter dismissed the complaint for lack of merit. On appeal to the
NLRC, the Commission affirmed the decision of the Labor Arbiter.
Respondents filed a Motion for Reconsideration but was denied by the NLRC.
They appealed to the Court of Appeals which reversed the NLRC decision
declaring illegal, the petitioners dismissal from employment and ordering
private respondents to reinstate petitioners to their former positions without
loss of seniority rights with full backwages from the time of their dismissal
until actual reinstatement;
Issue:
Whether the policy of the employer banning spouses from working in the
same company is a valid exercise of management prerogative.
Held:
NO. The Labor Code is the most comprehensive piece of legislation
protecting labor. The case at bar involves Article 136 of the Labor Code
which provides:
Art. 136. It shall be unlawful for an employer to require as a condition of
employment or continuation of employment that a woman employee shall
not get married, or to stipulate expressly or tacitly that upon getting married
a woman employee shall be deemed resigned or separated, or to actually
dismiss, discharge, discriminate or otherwise prejudice a woman employee
merely by reason of her marriage.

It is true that the policy of petitioners prohibiting close relatives from working
in the same company takes the nature of an anti-nepotism employment
policy. Companies adopt these policies to prevent the hiring of unqualified
persons based on their status as a relative, rather than upon their ability.
These policies focus upon the potential employment problems arising from
the perception of favoritism exhibited towards relatives.
With more women entering the workforce, employers are also enacting
employment policies specifically prohibiting spouses from working for the
same company. We note that two types of employment policies involve
spouses: policies banning only spouses from working in the same company
(no-spouse employment policies), and those banning all immediate family
members, including spouses, from working in the same company (antinepotism employment policies).
A requirement that a woman employee must remain unmarried could be
justified as a bona fide occupational qualification, or BFOQ, where the
particular requirements of the job would justify the same, but not on the
ground of a general principle, such as the desirability of spreading work in
the workplace. A requirement of that nature would be valid provided it
reflects an inherent quality reasonably necessary for satisfactory job
performance.
We do not find a reasonable business necessity in the case at bar. It is
significant to note that in the case at bar, respondents were hired after they
were found fit for the job, but were asked to resign when they married a coemployee. Petitioners failed to show how the marriage of Simbol, then a
Sheeting Machine Operator, to Alma Dayrit, then an employee of the
Repacking Section, could be detrimental to its business operations. Neither
did petitioners explain how this detriment will happen in the case of Wilfreda
Comia, then a Production Helper in the Selecting Department, who married
Howard Comia, then a helper in the cutter-machine. The policy is premised
on the mere fear that employees married to each other will be less efficient.
If we uphold the questioned rule without valid justification, the employer can
create policies based on an unproven presumption of a perceived danger at
the expense of an employees right to security of tenure.

Lastly, the absence of a statute expressly prohibiting marital discrimination


in our jurisdiction cannot benefit the petitioners. The protection given to
labor in our jurisdiction is vast and extensive that we cannot prudently draw
inferences from the legislatures silence that married persons are not
protected under our Constitution and declare valid a policy based on a
prejudice or stereotype. Thus, for failure of petitioners to present undisputed
proof of a reasonable business necessity, we rule that the questioned policy
is an invalid exercise of management prerogative.

ANSELMO FERRAZZINI, plaintiff-appellee, vs. CARLOS GSELL,


defendant-appellant.
G.R. No. L-10712, August 10, 1916, EN BANC
TRENT, J.:
Facts:

The plaintiff had been employed by the defendant for an indefinite time to
work in the latter's industrial enterprises involving the manufacture of
umbrellas in the city of Manila. The defendant admitted that he discharged
the plaintiff without giving him the "written advice of six months in advance"
as provided in the contract, but alleged that the discharge was lawful on
account of absence, unfaithfulness, and disobedience of orders. The
defendant sought affirmative relief for a further alleged breach of the
contract by the plaintiff after his discharge.
That portion of the contract upon which the defendant's counterclaimed is
based reads as follows:
That during the term of this contract, and for the period of five years after
the termination of the employment of the said party of the second part,
whether this contract continue in force for the period of one, two, three or
more years, or be sooner terminated, the said party of the second party shall
not engage or interest himself in any business enterprises similar to or in
competition with those conducted, maintained or operated by the said party
of the first day in the Philippines, and shall not assist, aid or encourage any
such enterprise by the furnishing of information, advice or suggestions of any
kind, and shall not enter into the employ of any enterprises in the Philippine
Islands, whatever, save and except after obtaining special written permission
therefor from the said party of the first part. It is further stipulated and
agreed that the said party of the second part is hereby obligated and bound
to pay unto the party of the first part the sum of ten thousand pesos,
Philippine currency (P10,000) as liquidated damages for each and every
breach of the present clause of this contract, whether such breach occurred
during the employment of the said party of the second part or at any time
during the period of five years from and after the termination of said
employment, and without regard to the cause of the termination of said
employment.
The plaintiff admits that he entered the employment of Mr. Whalen in the
Philippine Islands as a foreman on some construction work for a cement
factory within a few days after his discharge and without the consent, either
written or verbal, of the defendant. This work was entirely different and
disassociated from that engaged in by the defendant Gsell, yet this act of the
plaintiff was a technical violation of the above-quoted provisions of the
contract wherein he expressly agreed and obligated himself "not to enter
into the employment of any enterprise in the Philippine Islands, whatever,

save and except after obtaining special written permission therefor" from the
defendant.
Issue:
Whether the provisions of the contract are valid and binding upon the
plaintiff.
Held:
No.
The contract under consideration, tested by the law, rules and principlest
forth is clearly one in undue or unreasonable restraint of trade and therefore
against public policy. It is limited as to time and space but not as to trade. It
is not necessary for the protection of the defendant, as this is provided for in
another part of the clause. It would force the plaintiff to leave the Philippine
Islands in order to obtain a livelihood in case the defendant declined to give
him the written permission to work elsewhere in this country.

ALFONSO DEL CASTILLO, plaintiff-appellant, vs. SHANNON


RICHMOND, defendant-appellee.
G.R. No. L-21127, February 9, 1924, EN BANC
JOHNSON, J.:

Facts:
Shannon Richmond hired Alfonso del Castillo as pharmacist of the formers
drugstore known as the Botica Americana situated in the district of Legaspi
of the municipality and Province of Albay. In their agreement, the following
stipulation appeared:
3. That in consideration of the fact that the said Alfonso del Castillo has just
graduated as a pharmacist and up to the present time has not been
employed in the capacity of a pharmacist and in consideration of this
employment and the monthly salary mentioned in this contract, the said
Alfonso del Castillo also agrees not to open, nor own nor have any interest
directly or indirectly in any other drugstore either in his own name or in the
name of another; nor have any connection with or be employed by any other
drugstore situated within a radius of our miles from the district of Legaspi,
municipality and Province of Albay, while the said Shannon Richmond or his
heirs may own or have open a drugstore, or have an interest in any other
one within the limits of the districts of Legaspi, Albay, and Daraga of the
municipality of Albay, Province of Albay.
The said contract was acknowledge before a notary on the same day of its
execution.
The plaintiff alleges that the provisions and conditions contained in the third
paragraph of said contract constitute an illegal and unreasonable restriction
upon his liberty to contract, are contrary to public policy, and are
unnecessary in order to constitute a just and reasonable protection to the
defendant; and asked that the same be declared null and void and of no
effect. The defendant interposed a general and special defense. In his special
defense he alleges "that during the time the plaintiff was in the defendant's
employ he obtained knowledge of his trade and professional secrets and
came to know and became acquainted and established friendly relations with
his customers so that to now annul the contract and permit plaintiff to
establish a competing drugstore in the town of Legaspi, as plaintiff has
announced his intention to do, would be extremely prejudicial to defendant's
interest."
Issue:
Whether or not the qestioned stipulation is valid.
Held:

YES.
A contract by which an employee agrees to refrain for a given lenght of time,
after the expiration of the term of his employment, from engaging in a
business, competitive with that of his employer, is not void as being in
restraint of trade if the restraint imposed is not greater than that which is
necessary to afford a reasonable protection. In all cases like the present, the
question is whether, under the particular circumstances of the case and the
nature of the particular contract involved in it, the contract is, or is not,
unreasonable. Of course in establishing whether the contract is a reasonable
or unreasonable one, the nature of the business must also be considered.
What would be a reasonable restriction as to time and place upon the
manufacture of railway locomotive engines might be a very unreasonable
restriction when imposed upon the employment of a day laborer.
Considering the nature of the business in which the defendant is engaged, in
relation with the limitation placed upon the plaintiff both as to time and
place, we are of the opinion, and so decide, that such limitation is legal and
reasonable and not contrary to public policy.

ROLANDO C. RIVERA, Petitioner, vs. SOLIDBANK CORPORATION,


Respondent.
G.R. No. 163269 , April 19, 2006, FIRST DIVISION

CALLEJO, SR., J.:


Facts:
Petitioner had been working for Solidbank Corporation since July 1, 1977. He
was initially employed as an Audit Clerk, then as Credit Investigator, Senior
Clerk, Assistant Accountant, and Assistant Manager. Prior to his retirement,
he became the Manager of the Credit Investigation and Appraisal Division of
the Consumers Banking Group.
In December 1994, Solidbank offered two retirement programs to its
employees. Since Rivera was only 45 years old, he was not qualified for
retirement under the ORP. Under the SRP, he was entitled to receive
P1,045,258.95 by way of benefits.
Deciding to devote his time and attention to his poultry business in Cavite,
Rivera applied for retirement under the SRP. Solidbank approved the
application and Rivera was entitled to receive the net amount of
P963,619.28. Rivera received the amount and confirmed his separation from
Solidbank on February 25, 1995.
Subsequently, Solidbank required Rivera to sign an undated Release, Waiver
and Quitclaim, which contains the following provisions:
I will not, at any time, in any manner whatsoever, directly or indirectly
engage in any unlawful activity prejudicial to the interest of the BANK, its
parent, affiliate or subsidiary companies, their stockholders, officers,
directors, agents or employees, and their successors-in-interest and will not
disclose any information concerning the business of the BANK, its manner or
operation, its plans, processes or data of any kind.
The second undertaking is incorporated in the Undertaking following
petitioners execution of the Release, Waiver and Quitclaim which reads:
That as a supplement to the Release and Quitclaim, I executed in favor of
Solidbank on FEBRUARY 28, 1995, I hereby expressly undertake that I will not
seek employment with any competitor bank or financial institution within one
(1) year from February 28, 1995.
On May 1, 1995, the Equitable Banking Corporation employed Rivera as
Manager of its Credit Investigation and Appraisal Division of its Consumers
Banking Group. Upon discovering this, Solidbank wrote a letter informing
Rivera that he had violated the Undertaking. She likewise demanded the

return of all the monetary benefits he received in consideration of the SRP


within five (5) days from receipt; otherwise, appropriate legal action would
be taken against him.
When Rivera refused to return the amount demanded within the given
period, Solidbank filed a complaint for Sum of Money with Prayer for Writ of
Preliminary Attachment before the Regional Trial Court.
Issue:
Whether the employment ban incorporated in the Undertaking which
petitioner executed upon his retirement is unreasonable, oppressive, hence,
contrary to public policy;
Held:
Yes.
On the face of the Undertaking, the post-retirement competitive employment
ban is unreasonable because it has no geographical limits; respondent is
barred from accepting any kind of employment in any competitive bank
within the proscribed period. Although the period of one year may appear
reasonable, the matter of whether the restriction is reasonable or
unreasonable cannot be ascertained with finality solely from the terms and
conditions of the Undertaking, or even in tandem with the Release, Waiver
and Quitclaim.

RENATO S. GATBONTON, Petitioner, - versus - NATIONAL LABOR


RELATIONS COMMISSION, MAPUA INSTITUTE OF TECHNOLOGY and
JOSE CALDERON, Respondents.
G.R. NO. 146779, January 23, 2006, FIRST DIVISION
AUSTRIA-MARTINEZ, J.:
Facts:
Petitioner Renato S. Gatbonton is an associate professor of respondent
Mapua Institute of Technology (MIT), Faculty of Civil Engineering. Some time
in November 1998, a civil engineering student of respondent MIT filed a
letter-complaint against petitioner for unfair/unjust grading system, sexual
harassment and conduct unbecoming of an academician. Pending
investigation of the complaint, respondent MIT placed petitioner under a 30day preventive suspension effective January 11, 1999. The committee
believed that petitioners continued stay during the investigation affects his
performance as a faculty member, as well as the students learning; and that
the suspension will allow petitioner to prepare himself for the investigation
and will prevent his influences to other members of the community.
Thus, petitioner filed with the NLRC a complaint for illegal suspension.
Petitioner questioned the validity of the administrative proceedings with the
Regional Trial Court in a petition for certiorari but the case was terminated on
May 21, 1999 when the parties entered into a compromise agreement
wherein respondent MIT agreed to publish in the school organ the rules and
regulations implementing Republic Act No. 7877 (R.A. No. 7877) or the AntiSexualHarassment Act; disregard the previous administrative proceedings
and conduct anew an investigation on the charges againstpetitioner.
Petitioner agreed to recognize the validity of the published rules and
regulations, as well as the authority of respondent to investigate, hear and
decide the administrative case against him.
Issue:
Whether or not the preventive suspension is valid.
Held:
NO. Preventive suspension is a disciplinary measure for the protection of the
companys property pending investigation of any alleged malfeasance or
misfeasance committed by the employee. The employer may place the

worker concerned under preventive suspension if his continued employment


poses a serious and imminent threat to the life or property of the employer
or of his co-workers. However, when it is determined that there is no
sufficient basis to justify an employees preventive suspension, the latter is
entitled to the payment of salaries during the time of preventive suspension.

ALFREDO ROCO and CANDELARIA ROCO, respondents.


[G.R. No. 150660. July 30, 2002] FIRST DIVISION
KAPUNAN, J.:

Facts:
CALS Poultry Supply Corporation is engaged in the business of selling
dressed chicken and other related products and managed by Danilo Yap.
On May 16, 1995, it hired Candelaria Roco as helper, at its chicken dressing
plant on a probationary basis.
With respect to Candelaria Roco, there is no dispute that she was employed
on probationary basis. She was hired on May 16, 1995 and her services were
terminated on November 15, 1995 due to poor work performance. She did
not measure up to the work standards on the dressing of chicken. The Labor
Arbiter sustained CALS in terminating her employment. The NLRC affirmed
the Labor Arbiters ruling.
The Court of Appeals did not disagree with the NLRCs finding that Candelaria
was dismissed because she did not qualify as a regular employee in
accordance with the reasonable standards made known by the company to
her at the time of her employment.
However, the Court of Appeals set aside the NLRC ruling on the ground that
at the time Candelarias services were terminated, she had attained the
status of a regular employee as the termination on November 15, 1995 was
effected four (4) days after the 6-month probationary period had expired,
hence, she is entitled to security of tenure in accordance with Article 281 of
the Labor Code.
Issue:
What is the proper computation of the probationary employment period.

Held:

In Cebu Royal v. Deputy Minister of Labor, our computation of the 6-month


probationary period is reckoned from the date of appointment up to the
same calendar date of the 6th month.

[G.R. No. 149859. June 9, 2004]


RADIN C. ALCIRA, petitioner, vs. NATIONAL LABOR RELATIONS
COMMISSION, MIDDLEBY PHILIPPINES CORPORATION/FRANK

THOMAS, XAVIER G. PEA and TRIFONA F. MAMARADLO,


respondents.
CORONA, J.:
Facts:
The petitioner, Radin Alcira, was hired by the respondent Middleby
Philippines Corporation as engineering support services supervisor under
probationary status for 6 months. Afterwards, the service of the petitioner
was terminated by the respondent on the ground that the latter was not
satisfied on the performance of the former. As a result, the petitioner filed a
complaint foe illegal dismissal in the National Labor Relations Commission
(NLRC) against the respondent.
Petitioner contended that his termination in the service tantamount to illegal
dismissal since he attained the status of a regular employee as of the time of
dismissal. He presented the appointment paper showing that he was hired on
May 20, 1996, consequently, his dismissal on November 20, 1996 was illegal
because at that time, he was already a regular employee since the 6-month
probationary period ended on November 16, 1996.
The respondent, on the other hand, asserted that during the petitioners
probationary period, he showed poor performance on his assigned tasks, was
late couple of times and violated the companys rule. Thus, the petitioner
was terminated and his application to become a regular employment was
disapproved. The respondent also insisted that the removal of the petitioner
from office was within the probationary period.
The Labor Arbiter dismissed the complaint on the ground that the dismissal
of the petitioner was done before his regularization because the 6- month
probationary period, counting from May 20, 1996 shall end on November 20,
1996. The NLRC affirmed the decision of the Labor Arbiter. The Court of
Appeals affirmed the decision of NLRC. Hence, the present recourse.
Issue:
Whether the petitioner was already a regular employee in respondents
company at the time of his dismissal from the service
Held:

The Supreme Court ruled in the negative. The status of the petitioner at the
time of his termination was still probationary. His dismissal on November 20,
1996 was within the 6- month probationary period. Article 13 of the Civil
Code provides that when the law speaks of years, months, and days and
nights, it shall be understood that years are of 365 days, months of 30 days,
days of 24 hours and nights are from sunset to sunrise. Since, one month is
composed of 30 days, then, 6 months shall be understood to be composed of
180 days. And the computation of the 6- month period is reckoned from the
date of appointment up to the same calendar date of the 6th month
following. Since, the number of days of a particular month is irrelevant,
petitioner was still a probationary employee at the time of his dismissal.
Wherefore, the petition is dismissed.

G.R. No. 148738


MITSUBISHI MOTORS PHILIPPINES CORPORATION, Petitioner,
- versus CHRYSLER PHILIPPINES LABOR UNION and NELSON PARAS,
Respondents.

PUNO, J.,
FACTS
Private respondent Nelson Paras first worked with Mitsubishi Philippines as a
shuttle bus driver on March 19, 1976. He resigned on June 16, 1982 because
he went to Saudi Arabia and worked there as a diesel mechanic and heavy
machine operator from 1982 to 1993. Upon his return, Mitsubishi Philippines
re-hired him as a welder-fabricator at a tooling shop from November 1, 1994
to March 3, 1995.
On May 1996, Paras was re-hired again, this time as a probationary
manufacturing trainee at the Plant Engineering Maintenance Department. He
had an orientation on May 15, 1996 and afterwhich, with respect to the
companys rules and guidelines, started reporting for work on May 27, 1996.
Paras was evaluated by his immediate supervisors after six months of
working. The supervisors rating Paras performance were Lito R. Lacambacal
and Wilfredo J. Lopez, as part of the MMPCs company policies. Upon this
evaluation, Paras garnered an average rating.
Later, respondent Paras was informed by his supervisor, Lacambacal, that he
received an average performance rating but it is a rate which would still
qualify him to be regularized. But as part of the company protocols, the
Division Managers namely A.C. Velando, H.T. Victoria and Dante Ong
reviewed the performance evaluation made on Paras. Despite the
recommendations of the supervisors, they unanimously agreed that the
performance was unsatisfactory. As a consequence, Paras was not
considered for regularization.
Paras received a Notice of Termination on November 26, 1996 which was
dated November 25, 1996. This letters intent is to formally relieve him off of
his services and position effective the date since he failed to meet the
companys standards.

ISSUE:
Whether or not respondent Paras termination was legal or not.

HELD:
The Court holds that a company employer may indeed hire an employee on a
probationary basis in order to determine his fitness to perform work. The
Court stresses the existence of the statements under Article 281 of the Labor
Code which specifies that the employer must inform the employee of the
standards they were to meet in order to be granted regularization and that
such probationary period shall not exceed six (6) months from the date the
employee started working, unless specified in the apprenticeship agreement.
Respondent Paras was employed on a probationary basis and was apprised
of the standards upon which his regularization would be based during the
orientation. His first day to report for work was on May 27, 1996. As per the
company's policy, the probationary period was from three (3) months to a
maximum of six (6) months. Applying Article 13 of the Civil Code, the
probationary period of six (6) months consists of one hundred eighty (180)
days. The Court conforms with paragraph one, Article 13 of the Civil Code
providing that the months which are not designated by their names shall be
understood as consisting of thirty (30) days each. This case, the Labor Code
pertains to 180 days. Also, as clearly provided for in the last paragraph of
Article 13, it is said that in computing a period, the first day shall be
excluded and the last day included. Thus, the one hundred eighty (180) days
commenced on May 27, 1996, and ended on November 23, 1996. The
termination letter dated November 25, 1996 was served on respondent Paras
only at 3:00 a.m. of November 26, 1996. The Court held that by that time, he
was actually already a regular employee of the petitioner under Article 281
of the Labor Code. His position as a regularized employee is thus secured
until further notice.

[G.R. No. 149371. April 13, 2005]


ABERDEEN COURT, INC., and RICHARD NG, petitioners, vs. MATEO C.
AGUSTIN JR., respondent.
AZCUNA, J.:

FACTS:
On September 16, 1996, Aberdeen Court, Inc. (Aberdeen), one of the
petitioners, employed Mateo C. Agustin (Agustin), herein respondent, for the
purpose of trouble shooting the electrical problems in said petitioners
establishment. Agustin was engaged on a six-month probationary basis. The
employment contract provided, inter alia, that:
Should my performance be considered unsatisfactory at any time by
management during my probationary period, I understand and agree that
the management can terminate my services at any time, even before the
termination of the agreed six-month period.
On January 12 and 13, 1997 the personnel of Centigrade Industries, Inc.
performed a reading of the exhaust air balancing at the fifth and sixth floors
of Aberdeens premises. Petitioners claim that Agustin was placed in charge
of the undertaking. On the other hand, Agustin asserts that Engr. Abad
merely requested him to accompany the aforesaid personnel to show the
location of the exhaust air outlet at the fifth and sixth floors of the premises.
He avers that:
The request of Engr. Abad is actually the responsibility of the companys
mechanical engineers. Despite the fact that the request of Engr. Abad is not
a part of his job since he is not a mechanical engineer and there were three
(3) other mechanical engineers on duty in the company premises, petitioner
[herein respondent], being a subordinate of Engr. Abad, obliged and
accompanied the aforementioned personnel to the location. There were no
other specific instructions from Engr. Abad to petitioner with respect to the
conduct or actual reading to be made by the Centigrade personnel.
It must be noted that the reading of exhaust air balancing is under the
category of heating, ventilating and air conditioning (HVAC) which are within
the realm of field of work of mechanical engineers. Being an electrical
engineer, petitioner obviously has no knowledge of the procedure and the
equipment used by mechanical engineers in the conduct of the reading of
the exhaust air balancing.
After the Centigrade personnel finished their job, they submitted their report
to Agustin. Petitioners allege that Agustin accepted and signed the report,

without verifying its correctness. Engineer Abad later checked the work of
the Centigrade employees only to find out that four rooms in the fifth floor
and five rooms in the sixth floor were incorrectly done. In contrast, Agustin
states that after the report was handed to him, he took the same to Engr.
Abad, who he claims was responsible for evaluating and confirming the said
report.
Allegedly, instead of signing it himself, Engr. Abad directed
respondent to sign it, giving the reason that Agustin was present when the
reading was conducted. Respondent Agustin complied, but he now points out
that his signature was not accompanied by any qualification that he
accepted the report on behalf of Aberdeen. He claims that he signed merely
to evidence that he received a copy of the report.
The parties also differ on the occurrences two days after the signing of the
report or on January 15, 1997.
According to petitioners, Aberdeen
management confronted Agustin with his failure to check the job and asked
him to explain his side. Agustin allegedly ignored management and left the
company, which made it impossible for Aberdeen to transmit any further
notice to him.
However, Agustin claims that:
On January 15, 1997 or two days after the report was submitted by
Centigrade Industries, petitioner [herein respondent] was summarily
dismissed. In the afternoon of that day, he received a telephone call from
the personnel office of respondent company ordering him to report to that
office after his tour of duty. At about seven p.m. at the personnel office, Ms.
Lani Carlos of the Personnel Department, informed him that Aberdeen Court
is terminating his services as electrical engineer.
Petitioner was
flabbergasted. Ms. Carlos then informed him that he could get his two (2)
weeks salary in the amount of P4,000, more or less, on the condition that he
will sign some documents which provides that the company has no more
liability and that he is voluntarily resigning from Aberdeen Court. Aware of
his rights, petitioner did not sign the offered documents. He was then
hurriedly led to the door by Ms. Carlos.
The following day or on January 16, 1997, petitioner requested assistance
from the Department of Labor and Employment (DOLE). A DOLE personnel
told him to report for work since private respondents did not serve him a
notice of termination. As instructed, petitioner reported for work on the
same day. Upon arriving at the company premises, petitioner asked Ms.

Carlos if he could still report for work but private respondents personnel
officer told him that he cannot do so.
ISSUE:
Whether or not Agustin qualified as a regular employee
Ruling:
It can be gleaned from Article 281 of the Labor Code that there are two
grounds to legally terminate a probationary employee. It may be done either:
a) for a just cause or b) when employee fails to qualify as a regular employee
in accordance with reasonable standards made known by the employer to
the employee at the start of the employment.
Petitioners say that Agustin was terminated because he failed to qualify as a
regular employee.
Petitioners, however, allegedly did not show that
respondent was apprised of these reasonable standards at the start of the
employment.
In Servidad v. NLRC et al., where effectively the probationary period was for
one year, the Court stated:
If the nature of the job did actually necessitate at least one year for the
employee to acquire the requisite training and experience, still, the same
could not be a valid probationary employment as it falls short of the
requirement of Article 281 of the Labor Code. It was not brought to light that
the petitioner was duly informed at the start of his employment, of the
reasonable standards under which he could qualify as a regular employee.
The rudiments of due process demand that an employee should be apprised
beforehand of the conditions of his employment and the basis for his
advancement.
Similarly, in Secon Philippines Ltd. v. NLRC, the dismissal of the employee
was declared illegal by the Court because the employer did not prove that
the employee was properly apprised of the standards of the job at the time
of his engagement and, naturally, the employer could not show that the
employee failed to meet such standards.
The Implementing Rules of the Labor Code in Book VI, Rule I, Section 6, also
provides:

Probationary employment. -- There is probationary employment where the


employee, upon his engagement, is made to undergo a trial period during
which the employer determines his fitness to qualify for regular employment,
based on reasonable standards made known to him at the time of
engagement.
Probationary employment shall be governed by the following rules:
(c)
The services of an employee who has been engaged on probationary
basis may be terminated only for a just cause, when he fails to qualify as a
regular employee in accordance with the reasonable standards prescribed by
the employer.
(d)
In all cases of probationary employment, the employer shall make
known to the employee the standards under which he will qualify as a
regular employee at the time of his engagement. Where no standards are
made known to the employee at that time, he shall be deemed a regular
employee.
The above rule, however, should not be used to exculpate a probationary
employee who acts in a manner contrary to basic knowledge and common
sense, in regard to which there is no need to spell out a policy or standard to
be met. This is what the NLRC found to be the fact in this case. Said the
NLRC:
It bears stressing that even if technically the reading of air exhaust balancing
is not within the realm of expertise of the complainant, still it ought not to be
missed that prudence and due diligence imposed upon him not to readily
accept the report handed to him by the workers of Centigrade Industries.
Required of the complainant was that he himself proceed to the work area,
inquire from the workers as to any difficulties encountered, problems fixed
and otherwise observe for himself the progress and/or condition/quality of
the work performed.
As it is, We find it hard to believe that complainant would just have been
made to sign the report to signify his presence. By saying so, complainant is
inadvertently degrading himself from an electrical engineer to a mere
watchdog. It is in this regard that We concur with the respondents that by
his omission, lack of concern and grasp of basic knowledge and common
sense, complainant has shown himself to be undeserving of continued
employment from probationary employee to regular employee.

[G.R. No. 145417. December 11, 2003]


FLORENCIO M. DE LA CRUZ, JR., petitioner, vs. NATIONAL LABOR
RELATIONS COMMISSION (4th Division) SHEMBERG MARKETING
CORPORATION and ERNESTO U. DACAY, JR., respondents.
CORONA, J.:
FACTS:
On May 27, 1996, petitioner Florencio M. de la Cruz, Jr. was hired by private
respondent Shemberg Marketing Corporation (Shemberg) as senior sales
manager with a monthly salary of P40,500. Shemberg was engaged in the
business of manufacturing, trading, distributing and importing various
consumer products. The position of senior sales manager was then newly
created in line with Shembergs objective of product positioning in the
consumer market. Its duties included, among others, the supervision and
control of the sales force of the company. The senior sales manager was also
vested with some discretion to decide on matters within the scope of his
functions, including the appointment of district sales representatives and the
reshuffling of salesmen to achieve sales targets.
However, on September 14, 1996, Shembergs human resource department
manager, Ms. Lilybeth Y. Llanto, summoned petitioner and informed him of
the managements decision to terminate his services. Petitioner asked
Llanto for the reason but the latter merely informed him that it had
something to do with the drop in the companys sales. Petitioner then
requested a meeting with Shembergs vice president, Ernesto U. Dacay, Jr.,
but was told that the decision of the management was final. His request to
be furnished a 30-day written notice was also denied by the management.
Hence, petitioner filed a complaint for illegal dismissal, non-payment of
salary, backwages, 13th month pay and damages against Shemberg, Ernesto
Dacay, Jr. and Lilybeth Llanto.
Respondents answered that petitioners dismissal was premised on the
following: (1) his poor performance as evidenced by the steady and
substantial drop in company sales since his assumption as senior sales
manager; (2) the dissatisfaction of his subordinates over his management
style and dealings with the companys distributors which resulted in the low
morale of Shembergs sales force, as evidenced by the joint affidavit of two
of his subordinates, Ruel O. Salgado and Joel D. Sol; (3) his unauthorized use

of company cellular phone for overseas personal calls and (4) the
unauthorized reimbursement of the plane tickets of his wife and child. In
short, petitioner was terminated for his failure to meet the required company
standards and for loss of trust and confidence.
ISSUES:
Whether or not De la Cruz is a probationary employee
RULING:
Petitioner was holding a managerial position in which he was tasked to
perform key functions in accordance with an exacting work ethic. His
position required the full trust and confidence of his employer. While
petitioner could exercise some discretion, this obviously did not cover acts
for his own personal benefit. As found by the court a quo, he committed a
transgression that betrayed
the trust and confidence of his employer
reimbursing his familys personal travel expenses out of company funds.
Petitioner failed to present any persuasive evidence or argument to prove
otherwise. His act amounted to fraud or deceit which led to the loss of trust
and confidence of his employer.
We reiterate the well-established rule that findings of fact of the Court of
Appeals are conclusive on the parties and are not generally reviewable by
this Court when supported by substantial evidence.
Petitioner was hired by respondent Shemberg Marketing Corporation on May
27, 1996 and was terminated on September 14, 1996. Article 281 of the
Labor Code provides:
Probationary employment Probationary employment shall not exceed six
(6) months from the date the employee started working, unless it is covered
by an apprenticeship agreement stipulating a longer period. The services of
an employee who has been engaged on a probationary basis may be
terminated for a just cause or when he fails to qualify as a regular employee
in accordance with reasonable standards, made known by the employer to
the employee at the time of his engagement. An employee who is allowed to
work after a probationary period shall be considered a regular employee.
Petitioner vigorously contends that he was not a probationary employee
since Shemberg failed to disclose to him the reasonable standards for
qualifying as a regular employee.

This Court notes, however, the evidence on record clearly showing that
petitioner was well informed of the standards to be met before he could
qualify as a regular employee.
G.R. No. 148931
September 12, 2006
CATHAY PACIFIC AIRWAYS, LIMITED, petitioner, vs.
PHILIP LUIS F. MARIN and THE HON. COURT OF APPEALS (Former
First Division), respondents.
CALLEJO, SR., J.:
FACTS:
Marin used to work for Saudia Airlines as a ticketing agent. When he applied
for employment as a Reservation Officer in Cathay Pacific Airways, Ltd.
(Cathay), he was interviewed by the following: Senior Supervisor Nenita
Montallana, Reservations Manager Elizabeth Leviste, Staff and Administrative
Supervisor M.A. Canizares, and Country Manager (Philippines) Peter W.
Foster.
In a letter dated March 30, 1992, Foster confirmed Marin's appointment as
Reservations Officer effective April 6, 1992 for a probationary period of six
months. He was to receive a monthly salary of P5,334.00, including holidays
and rest days, with a promise of a salary review upon satisfactory completion
of the probationary period. The letter also stated that Cathay reserved the
right to "terminate [Marin's] services during the probationary period if [his]
performance proves to be unsatisfactory, in which case, [he] will receive the
salary due [him] at the time of the termination of [his] services." It was also
understood that Marin "had accepted the [recognized] terms of
employment," and that he would be "reconfirmed as a member of [the]
regular staff upon completion of the probationary period."
On October 15, 1992, Marin filed a complaint6 for illegal dismissal against
Cathay and Foster before the NLRC. The complaint was later amended to
include claims for 13th month pay, moral and exemplary damages, and
attorney's fees.

ISSUE:

Whether or not Marin is a probationary employee

RULING
It is settled that a probationary employee enjoys only a temporary
employment status, not a permanent status. In general terms, he is
terminable anytime as long as such termination is made before the
expiration of the six-month probationary period. The employment of a
probationary employee may only be terminated either (1) for a just cause; or
(2) when the employee fails to qualify as a regular employee in accordance
with the reasonable standards made known to him by the employer at the
start of his employment. The power of the employer to terminate an
employee on probation is thus subject to the following conditions: (1) it must
be exercised in accordance with the specific requirements of the contract; (2)
the dissatisfaction on the part of the employer must be real and in good
faith, not prejudicial so as to violate the contract or the law; and (3) there
must be no unlawful discrimination in the dismissal. The burden of proving
just or valid cause for dismissing an employee rests on the employer.

In Secon Philippines, Ltd. v. NLRC, this Court held that the probationary
employment of an employee may be terminated when he fails to qualify as
regular employee in accordance with reasonable standards made known to
him by his employer at the time of employment and after due process; in
Manlimos v. National Labor Relations Commission, it was held that the
constitutional protection on the probationary employee ends upon the
expiration of the period provided for in the probationary contract of
employment. Thus, a probationary employee remains secure in his or her
employment during the time that the employment contract remains in effect,
but the moment the probationary employment period expires, the employee
can no longer invoke the constitutional protection. Thereafter, the parties are
free to renew the contract or not; or for the employer to extend to such
employee a regular or permanent employment. If the employee is not given
a permanent or regular employment contract on account of his
unsatisfactory work performance, it cannot be said that he was illegally
dismissed. In such case, the contract merely expired.

We agree with the rulings of the Labor Arbiter and NLRC that respondent's
employment was not terminated during the period of his probationary
employment, and that he was not extended a regular employment by
petitioner Cathay on account of his unsatisfactory work performance during
the probationary period.
G.R. No. 164582

March 28, 2007

PILAR ESPINA, ELEANOR G. AQUINO, LORENE C. BARNUEVO,


MARICRIS S. J. BANDINO, JULIO M. PETALIO, JR., NOEL T. DE BORJA,
REMEGIO P. BASCO, MATEO D. DEOCAREZA, EMILIANO A. EBREO,
BENJAMIN PAZ, LEONORA PAZ, CLAUDIO DE LOS REYES, LEANDRO R.
CELIS, PATERNO FERNANDEZ, ANICETO M. RODRIGUEZ, DONATO M.
PUNZALAN, LOURDES ALFONSO Q., ALLAN PANLILIO, DAISY V.
ARCEO, ALEJANDRO D. PASCUAL, MA. CORAZON T. BAJO, ARNOLD M.
BLANCO, CRISTITO S. ABELA, DIOSCORO FAJANILAG, and AGUSTIN
WONG, Petitioners,
vs.
HON. COURT OF APPEALS, MONDE M.Y. SAN BISCUIT CORP., M.Y. SAN
BISCUIT INC., MRS. MHEW WHA LIM and MR. KENG SUN MAR,
Respondents.

CHICO-NAZARIO, J.:
FACTS:
Respondent M.Y. San Biscuits, Inc. (M.Y. San) was previously engaged in the
business of manufacturing biscuits and other related products.
On 27 December 2000, in a conciliation proceeding before the Department of
Labor and Employment (DOLE) NCMB-NCR Director Leopoldo de Jesus, the
duly authorized representative of M.Y. San Workers Union-PTGWO and M.Y.
San Sales Force Union-PTGWO was informed of the closure or cessation of
business operations of respondent M.Y. San as a result of the intended sale of
the business and all the assets of respondent M.Y. San to respondent Monde
M.Y. San Corporation (Monde) and was notified of their termination, effective
31 January 2001.

On 28 December 2000, the written notice of the sale and purchase of the
assets of respondent M.Y San to respondent Monde and of the termination of
all the employees of respondent M.Y. San were filed before the DOLE
Regional Office No. IV.5
On 22 January 2001, respondent M.Y. San and the Union signed a
Memorandum of Agreement (MOA) embodying the agreements set forth in
the Minutes/Agreement, dated 27 December 2000. Embodied in the MOA is
an agreement that the existing Collective Bargaining Agreement shall cease
to be effective on 31 January 2001 and shall in no way be binding upon the
buyer, respondent Monde, and that respondent M.Y. San shall provide
respondent Monde a list of all its present employees who shall be given
preference in employment by the latter. Pertinent provisions of the
Agreement:
The Company agrees to submit the list of all its present employees to the
new corporation for purposes of rehiring if said employee applies and
qualifies, subject to such criteria as the new corporation may impose. In the
rehiring, the covered employees shall be given hiring preference, if qualified.
The corresponding Notice as to whom of the covered employees have been
hired by the new corporation shall be issued immediately after January 31,
2001. During the entire rehiring process and until the election and
qualification of the new officers, the PTGWO, through its National President,
or his authorized representative, shall act as the TRUSTEE of the UNION.
All employees hired by MONDE M.Y. SAN CORPORATION and/or the new
owner of the COMPANY, shall upon hiring, subject to the terms and conditions
of their probationary employment, become members of the UNION. The
continued existence of the UNION in the company and/or MONDE M.Y. SAN
CORPORATION shall not be interrupted by the payment of the Companys
employees of their separation package or the temporary closure of the
Companys operations.6
On 31 January 2001, all the employees of respondent M.Y. San received their
separation pay and the cash equivalent of their vacation and sick leaves.
Thereafter, they signed their respective Quitclaims.
On 1 February 2001, an Asset Purchase Agreement was executed between
respondents M.Y. San and Monde.

On 2 February 2001, respondent Monde commenced its operations. All the


former employees of respondent M.Y. San who were terminated upon its
closure and who applied and qualified for probationary employment,
including petitioners herein, started working for respondent Monde on a
contractual basis for a period of six months.
Subsequently, petitioners were terminated on various dates.
ISSUE:
Whether or not petitioners were illegally terminated
HELD:
Work is a necessity that has economic significance deserving legal
protection. The provisions on social justice and protection to labor in the
Constitution dictate so.
However, employers are also accorded rights and privileges to assure their
self-determination and independence and reasonable return of capital. This
mass of privileges comprises the so-called management prerogatives.
Although they may be broad and unlimited in scope, the State has the right
to determine whether an employers privilege is exercised in a manner that
complies with the legal requirements and does not offend the protected
rights of labor. One of the rights accorded an employer is the right to close
an establishment or undertaking. Just as no law forces anyone to go into
business, no law can compel anybody to continue the same.
The right to close the operations of an establishment or undertaking is
explicitly recognized under the Labor Code as one of the authorized causes
in terminating employment of workers, the only limitation being that the
closure must not be for the purpose of circumventing the provisions on
terminations of employment embodied in the Labor Code. Article 283 of the
Labor Code reads:
ART. 283. Closure of establishment and reduction of personnel.The
employer may also terminate the employment of any employee due to the
installation of labor saving devices, redundancy, retrenchment to prevent
losses or the closing or cessation of operation of the establishment or
undertaking unless the closing is for the purpose of circumventing the
provisions of this Title, by serving a written notice on the worker and the

Ministry of Labor and Employment at least one (1) month before the
intended date thereof. x x x. In case of retrenchment to prevent losses and in
cases of closures or cessation of operations of establishment or undertaking
not due to serious business losses or financial reverses, the separation pay
shall be equivalent to one (1) month pay or at least one-half (1/2) month pay
for every year of service, whichever is higher. A fraction of at least six (6)
months shall be considered as one (1) whole year. (Emphasis supplied.)
The phrase "closure or cessation of operations of establishment or
undertaking" includes a partial or total closure or cessation.
The closure, therefore, of the business operation of respondent M.Y. San was
not tainted with bad faith or other circumstance that would give rise to
suspicions of malicious intent. Other than their mere allegations, petitioners
failed to present independent evidence that would otherwise show that the
closure of M.Y. San was without factual basis and done in utter bad faith.
Mere allegation is not evidence. It is a basic rule in evidence that each party
must prove his affirmative allegation.

G.R. No. 128682

March 18, 1999

JOAQUIN T. SERVIDAD, petitioner,


vs.
NATIONAL LABOR RELATIONS COMMISSION, INNODATA PHILIPPINES,
INC./ INNODATA CORPORATION, TODD SOLOMON, respondents.
PURISIMA, J.:
FACTS:
Petitioner Joaquin T. Servidad was employed on May 9, 1994 by respondent
INNODATA as a "Data Control Clerk", under a contract of employment Section
2 of which, reads:
Sec. 2.
This Contract shall be effective for a period of 1 years
commencing on May 10, 1994, until May 10, 1995 unless sooner terminated
pursuant to the provisions hereof.
From May 10, 1994 to November 10, 1994, or for a period of six (6) months,
the EMPLOYEE shall be contractual during which the EMPLOYER can
terminate the EMPLOYEE's services by serving written notice to that effect.
Such termination shall be immediate, or at whatever date within the sixmonth period, as the EMPLOYER may determine. Should the EMPLOYEE
continue his employment beyond November 10, 1994, he shall become a
regular employee upon demonstration of sufficient skill in the terms of his
ability to meet the standards set by the EMPLOYER. If the EMPLOYEE fails to
demonstrate the ability to master his task during the first six months he can
be placed on probation for another six (6) months after which he will be
evaluated for promotion as a regular employee.

On November 9, 1995, or after working for six (6) months, he was made to
sign a three-month probationary employment and later, an extended threemonth probationary employment good until May 9, 1995.
On July 7, 1994, the petitioner was given an overall rating of 100% and 98%
in the work evaluations conducted by the company. In another evaluation,
petitioner received a rating of 98.5% given by the private respondent.
On May 9, 1995, petitioner was dismissed from the service on the ground of
alleged termination of contract of employment.
ISSUE:
Whether or not Servidad is a probationary employee?
RULING:
The private respondent sought to alternatively avail of probationary
employment and employment for a fixed term so as to preclude the
regularization of the status of petitioner. The utter disregard of public policy
by the contract in question negates the ruling of NLRC that said contract is
the law between the parties. The private agreement of the parties cannot
prevail over Article 1700 of the Civil Code, which provides:
Art. 1700. The relation between capital and labor are not merely
contractual. They are so impressed with public interest that labor contracts
must yield to the common good. Therefore, such contracts are subject to
special laws on labor unions, collective bargaining, strikes and lockouts,
closed shops, wages, working conditions, hours of labor and similar subjects.
Similarly telling is the case of Pakistan Airlines Corporation vs. Pole, et al. 20
There, it was said:
. . . provisions of applicable law, especially provisions relating to matters
affected with public policy, are deemed written into the contract. Put a little
differently, the governing principle is that the parties may not contract away
applicable provisions of law especially peremptory provisions dealing with
matters heavily impressed with public interest. The law relating to labor and
employment is clearly such an area and parties are not at liberty to insulate

themselves and their relationships from the impact of labor laws and
regulations by simply contracting with each other. . . .
On the averment that NLRC gravely abused its discretion in finding that
petitioner failed to meet the standards of the company, we find for petitioner.
The decision at NLRC on the matter simply stated that the petitioner fell
short of the expectations of the company without specifying factual basis
therefor. The public respondent overlooked the undisputed satisfactory
ratings of the performance of petitioner in the two job evaluations conducted
by the respondent company. Even granting, therefore, that the contract
litigated upon is valid; still, the petitioner, who was permitted to work beyond
six months could not be dismissed on the ground of failure to meet the
standards of Innodata. By the provisions of the very contract itself, petitioner
has become a regular employee of private respondent. Therein, it is
stipulated that: ". . . Should the EMPLOYEE continue employment beyond
November 10, 1994, he shall become a regular employee upon
demonstration of sufficient skill in the terms of his ability to meet the
standards set by the EMPLOYER. . . ."

[G.R. No. 119253. April 10, 1997]


AMOR CONTI and LEOPOLDO CRUZ, petitioners, vs. NATIONAL LABOR
RELATIONS COMMISSION (Third Division), CORFARM HOLDINGS
CORPORATION, CARLITO J. RABANG and CIPRIANO Q. BARAYANG,
respondents.
PADILLA, J.:
FACTS:
Petitioner Amor Conti was employed by respondent Corfarm as cashier on 2
February 1991. Petitioner Leopoldo Cruz was employed by the same
respondent corporation as a warehouseman on 16 May 1991. Both Amor
Conti and Leopoldo Cruz were subsequently promoted to the positions of
Head of Commissary and Store Supervisor, respectively. In their respective
employment contracts with Corfarm, it was stipulated that their employment
shall be coterminous with the effectivity of the contract executed by and
between Corfarm and MERALCO for the management of the latter's
commissary (hereinafter referred to as the "management contract").
On 31 December 1992, said management contract between Corfarm and
MERALCO expired. However, Corfarm continued to operate the MERALCO
commissary despite the non-renewal of said contract.
On 13 January 1993, petitioners received a memorandum, dated 12 January
1993, from private respondents terminating their services effective on said
date, allegedly for two reasons: 1) the expiration of their employment
contracts, these being coterminous with the management contract between
Corfarm and MERALCO, and; 2) the on-going evaluation of their past
performances, and investigation of the internal auditor of Corfarm of certain
anomalous transactions involving them (petitioners).

ISSUE:
Whether or not petitioners were illegally dismissed?
RULING:
At the outset, it will be noted that the Office of the Solicitor General (OSG), in
its "Manifestation and Motion in lieu of Comment", dated 19 June 1995,
agreed with the findings of the labor arbiter that the petitioners were illegally
dismissed, and prayed of this Court that the questioned NLRC decision dated
24 November 1994 and resolution dated 26 January 1995, be set aside.
Petitioners contend that they were denied due process when they were
dismissed without a written notice (specifying the particular charges
constituting the grounds for their dismissal), and a hearing, as required by
law. They further contend that the memorandum dated 11 January 1993,
supposedly issued by Corfarm to petitioners directing them "to explain why
they should not be dismissed for alleged acts of negligence and
carelessness" was never received by them. Besides, said memorandum did
not specify the particular acts or omissions of petitioners. It merely stated
that based on the results of the investigation conducted by Corfarm's
internal audit staff, petitioners were found to have been negligent in the
performance of their duties.
Petitioners' contentions are meritorious.
This Court has consistently held that the twin requirements of notice and
hearing constitute essential elements of due process in the dismissal of
employees.As to the requirement of notice, it has been held that the
employer must furnish the worker with two written notices before
termination of employment can be legally effected: (a) notice which apprises
the employee of the particular acts or omissions for which his dismissal is
sought, and; (b) subsequent notice which informs the employee of the
employer's decision to dismiss him.
With regard to the requirement of a hearing, this Court has held that the
essence of due process is simply an opportunity to be heard, and not that an
actual hearing should always and indispensably be held.

FABELA vs SAN MIGUEL CORPORATION Case Digest


[G.R. No. 150658 February 9, 2007] NOELITO FABELA, MARCELO
DELA CRUZ III, ROGELIO LASAT, HENRY MALIWANAG, MANUEL DELOS
SANTOS, and ROMMEL QUINES, Petitioners, vs. SAN MIGUEL
CORPORATION and ARMAN HICARTE, Respondents.
FACTS:
Petitioners were hired by respondent San Miguel
Corporation (SMC) as Relief Salesmen for the Greater Manila Area (GMA)
under separate but almost similarly worded Contracts of Employment with
Fixed Period. After having entered into successive contracts of the same
nature with SMC, the services of petitioners were terminated after SMC no
longer agreed to forge another contract with them. Respondent SMC claimed
that the hiring of petitioners was not intended to be permanent, as the same
was merely occasioned by the need to fill in a vacuum arising from SMCs
gradual transition to a new system of selling and delivering its products.
Respondents explained that SMC previously operated under the Route
System, but began implementing in 1993 the Pre-Selling System in which
the salesmen under the earlier system would be replaced by Accounts
Specialists which called for upgraded qualifications. While some of the
qualified regular salesmen were readily upgraded to the position of Accounts
Specialist, respondents claimed that SMC still had to sell its beer products
using the conventional routing system during the transition stage, thus
giving rise to the need for temporary employees; and the members of the
regular Route Crew then existing were required to undergo a training
program to determine whether they possessed or could be trained for the
necessary attitude and aptitude required of an Accounts Specialist, hence,
the hiring of petitioners and others for a fixed period, co-terminus with the
completion of the transition period and Training Program for all prospective
Accounts Specialists. Claiming that they were illegally dismissed, petitioners
filed complaints for illegal dismissal against respondents.
ISSUE:

Whether or not petitioners were validly hired for a fixed period.


HELD:
The SC held that under article 280 of the Labor Code, there are two kinds of
regular employees, namely: (1) those who are engaged to perform activities
which are necessary or desirable in the usual business or trade of the
employer, and (2) those casual employees who have rendered at least one
year of service, whether continuous or broken, with respect to the activity in
which they are employed. Article 280 also
recognizes project employees, those whose employment has been fixed for
a specific
project or undertaking. Project employment is distinct from casual
employment referred to in the second paragraph of Article 280 for the
proviso that any employee who has rendered at least one year of
service . . . shall be considered a regular employee does not apply to project
employees, but only to casual employees. Although Article 280 does not
expressly recognize employment for a fixed period, which is distinct from
employment which has been fixed for a specific project or undertaking, it has
been clarified that employment for a fixed period is not in itself illegal. Even
if the duties of an employee consist of activities usually necessary or
desirable in the usual business of the employer, it does not necessarily follow
that the parties are forbidden from agreeing on a period of time for the
performance of such activities through a contract of employment for a fixed
term.
Unfortunately, respondents contention that there are fixed periods stated in
the contracts of employment does not lie. Brent instructs that a contract of
employment stipulating a fixed-term, even if clear as regards the existence
of a period, is invalid if it can be shown that the same was executed with the
intention of circumventing security of tenure, and should thus be ignored.
Indeed, substantial evidence exists in the present case showing that the
subject contracts were utilized to deprive petitioners of their security of
tenure.The contract of employment of petitioner Fabela, for instance, states
that the transition period from the Route System to the Pre-Selling System
would be twelve (12) months from April 4, 1995. It bears noting, however,
that petitioner Fabela, besides being hired again for another fixed period of
four (4) months after the lapse in April 1996 of the one-year contract, had
already been working for respondent SMC on a fixed-term basis as early as

1992, or one year before respondent SMC even began its shift to the Preselling System in 1993. Thus, there is sufficient basis to believe that the shift
of SMC to the Pre-Selling System was not the real basis for the forging of
fixed-term contracts of employment with petitioners and that the periods
were fixed only as a means to preclude petitioners from acquiring security of
tenure. A fixed-term employment is valid only under certain circumstances,
such as when the employee himself insists upon the period, or where the
nature of the engagement is such that, without being seasonal or for a
specific project, a definite date of termination is a sine qua non. Petition is
granted.
G.R. No. 150478. April 15, 2005
HACIENDA BINO/HORTENCIA STARKE, INC./HORTENCIA L. STARKE,
Petitioners,
vs.
CANDIDO CUENCA, FRANCISCO ACULIT, ANGELINA ALMONIA,
DONALD ALPUERTO, NIDA BANGALISAN, ROGELIO CHAVEZ, ELMO
DULINGGIS, MERCEDES EMPERADO, TORIBIO EMPERADO, JULIANA
ENCARNADO, REYNALDO ENCARNADO, GENE FERNANDO, JOVEN
FERNANDO, HERNANI FERNANDO, TERESITA FERNANDO, BONIFACIO
GADON, JOSE GALLADA, RAMONITO KILAYKO, ROLANDO KILAYKO,
ALFREDO LASTIMOSO, ANTONIO LOMBO, ELIAS LOMBO, EMMA
LOMBO, LAURENCIA LOMBO, LUCIA LOMBO, JOEL MALACAPAY, ADELA
MOJELLO, ERNESTO MOJELLO, FRUCTOSO MOJELLO, JESSICA
MOJELLO, JOSE MOJELLO, MARITESS MOJELLO, MERLITA MOJELLO,
ROMEO MOJELLO, RONALDO MOJELLO, VALERIANA MOJELLO, JAIME
NEMENZO, RODOLFO NAPABLE, SEGUNDIA OCDEN, JARDIOLINA
PABALINAS, LAURO PABALINAS, NOLI PABALINAS, RUBEN
PABALINAS, ZALDY PABALINAS, ALFREDO PANOLINO, JOAQUIN
PEDUHAN, JOHN PEDUHAN, REYNALDO PEDUHAN, ROGELIO
PEDUHAN, JOSEPHINE PEDUHAN, ANTONIO PORRAS, JR., LORNA
PORRAS, JIMMY REYES, ALICIA ROBERTO, MARCOS ROBERTO, JR.,
MARIA SANGGA, RODRIGO SANGGA, ARGENE SERON, SAMUEL
SERON, SR., ANGELINO SENELONG, ARMANDO SENELONG, DIOLITO
SENELONG, REYNALDO SENELONG, VICENTE SENELONG, FEDERICO
STA. ANA, ROGELIO SUASIM, EDNA TADLAS, ARTURO TITONG, JR.,
JOSE TITONG, JR., NANCY VINGNO, ALMA YANSON, JIMMY YANSON,
MYRNA VILLANUEVA BELENARIO, SALVADOR MALACAPAY, and
RAMELO TIONGCO, Respondents.

CALLEJO, SR., J.:


FACTS:
The petitioner disfavored the fact that the private respondent employees
have formed a union. When the union became the collective bargaining
representative in the certification election, the petitioner refused to sit down
to negotiate a CBA. Moreover, the respondents were not given work for a
month amounting to unjustified dismissal. As a result, the complainants
staged a strike to protest but was settled through a memorandum of
agreement which contained a list of those considered as regular employees
for the payroll.
The NLRC held that there was illegal dismissal and this was affirmed by the
Court of Appeals.
ISSUE:
W/N the employees are regular workers
RULING:
Yes, they are regular and not seasonal employees. For them to be excluded
as regulars, it is not enough that they perform work that is seasonal in
nature but they also are employed for the duration of one season. The
evidence only proved the first but not the second requirement.
The ruling in Mercado v. NLRC is not applicable since in that case, the
workers were merely required to perform phases of agricultural work for a
definite period of time, after which, their services are available to other
employers. The management's sudden change of assignment reeks of bad
faith, it is likewise guilty of ULP.

G.R. No. 167045

August 29, 2008

COCOMANGAS HOTEL BEACH RESORT and/or SUSAN MUNRO,


petitioners,
vs.
FEDERICO F. VISCA, JOHNNY G. BAREDO, RONALD Q. TIBUS, RICHARD
G. VISCA and RAFFIE G. VISCA, respondents.
AUSTRIA-MARTINEZ, J.:

Facts:
The respondents alleged that they were regular employees of petitioners
tasked with the maintenance and repair of the resort facilities. They did not
report for work after they were informed that the ongoing constructions and
repairs would be temporarily suspended because they caused irritation and
annoyance to the resort's guests and due to budgetary constraints. They
later on discovered that not less than ten workers were subsequently hired
by petitioners to do repairs in two cottages of the resort and two workers
were retained after the completion without respondents being allowed to
resume work.
Issue:
Whether respondents are regular or project employees
Held:

They are regular employees, not project employees. A project employee is


one whose "employment has been fixed for a specific project or undertaking,
the completion or termination of which has been determined at the time of
the engagement of the employee or where the work or service to be
performed is seasonal in nature and the employment is for the duration of
the season." Before an employee hired on a per-project basis can be
dismissed, a report must be made to the nearest employment office, of the
termination of the services of the workers every time completes a project.
In the present case, respondents cannot be classified as project employees,
since they worked continuously for petitioners from three to twelve years
without any mention of a "project" to which they were specifically assigned.
While they had designations as "foreman," "carpenter" and "mason," they
performed work other than carpentry or masonry. They were tasked with the
maintenance and repair of the furniture, motor boats, cottages, and
windbreakers and other resort facilities.
There is likewise no evidence of the project employment contracts covering
respondents' alleged periods of employment. More importantly, there is no
evidence that petitioners reported the termination of respondents' supposed
project employment to the DOLE as project employees.
Department Order No. 19, as well as the old Policy Instructions No. 20,
requires employers to submit a report of an employees termination to the
nearest public employment office every time his employment is terminated
due to a completion of a project. Petitioners' failure to file termination reports
is an indication that the respondents were not project employees but regular
employees.
This Court has held that an employment ceases to be coterminous with
specific projects when the employee is continuously rehired due to the
demands of employers business and re-engaged for many more projects
without interruption.
The repeated and continuing need for their services is sufficient evidence of
the necessity, if not indispensability, of their services to petitioners' resort
business.
That respondents were regular employees is further bolstered by the
following evidence:

(a) the SSS Quarterly Summary of Contribution Payments listing respondents


as employees of
petitioners;
(b) the Service Record Certificates stating that respondents were employees
of petitioners for
periods ranging from three to twelve years and all have given "very
satisfactory performance";
(c) petty cash vouchers showing payment of respondents' salaries and
holiday and overtime pays.

G.R. No. 157680

October 8, 2008

EQUIPMENT TECHNICAL SERVICES or JOSEPH JAMES DEQUITO,


petitioners,
vs.
COURT OF APPEALS, ALEX ALBINO, REY ALBINO, JULIUS ABANES,
MIGUEL ALINAB, CHRISTOPHER BIOL, NELSON CATONG, RENATO
DULOT, FLORO PACUNDO, MARCELITO GAMAS, REYNALDO LIMA,
SAMMY MESAGAL, ERNESTO PADILLA, and CONRADO SULIBAGA,
respondents.
VELASCO, JR., J.:
FACTS:
Petitioner Equipment Technical Services (ETS) is primarily engaged in the
business of sub-contracting plumbing works of on-going building
construction. Among its clients was Uniwide Sales, Inc. (Uniwide). Petitioner
Joseph James Dequito was, during the period material, occupying the position
of manager of ETS,1 albeit the CA referred to him as ETS president. On
various occasions involving different projects, ETS hired the services of
private respondents as pipe fitters, plumbers, or threaders.
In December 1998, ETS experienced financial difficulties when Uniwide, its
client at the time, failed to pay for the plumbing work being done at its
Coastal Mall. As a result, ETS was only able to pay its employees 13th month
pay equivalent to two weeks salary.

Unhappy over what they thought was ETS failure to release the balance of
their 13th month pay, private respondents brought their case before the
Arbitration Branch of the NLRC, docketed as NLRC NCR Case No. 00-0100571-99 and entitled as Alex Albino, Renato Dulot, Miguel Alinab, Marcelito
Gamas, Julius Abanes, Christopher Biol, Sammy Mesagal, Conrado Sulibaga,
Floro Pacundo v. Equipment Technical Services or Joseph James Dequito.
Later, two other cases were filed against ETS for illegal dismissal and
payment of money claims when the complainants thereat were refused work
in another ETS project, i.e., Richville project, allegedly because they refused
to sign individual employment contracts with ETS. These two other cases
were Nelson Catong, Roger Lamayon, Christopher Lamayon v. Equipment
Technical Services or Joseph James Dequito, docketed as NLRC NCR Case No.
00-02-01429-99; and Rey Albino, Ernesto Padilla, Reynaldo Lima v.
Equipment Technical Services or Joseph James Dequito, docketed as NLRC
NCR Case No. 00-02-01615-99.
The three cases were consolidated before the labor arbiter. Following failed
conciliation efforts, all concerned, except Roger and Christopher Lamayon,
submitted, as the labor arbiter directed, their respective position papers.
Private respondents position2 is summed up as follows: (1) they are regular
employees of ETS; (2) ETS dismissed them without cause and without due
process after they filed cases for money claims against ETS in the arbitration
branch of the NLRC; (3) ETS has not paid them their salaries, 13th month
pay, service incentive leave pay, overtime pay, and premium pay for
holidays and rest days; and (4) they are entitled to reinstatement to their
former positions with paid backwages in addition to their money claims and
payment of attorneys fees.
ETS position may be summed up as follows: (1) private respondents were its
contractual/project employees engaged for different projects of the
company; (2) they were not illegally dismissed, having been hired on a per
project basis; (3) ETS was unable to fully release private respondents 13th
month pay because Uniwide failed to pay for its contracted plumbing project;
(4) ETS was forced to abandon the Uniwide project and undertake another
project, the Richville project, because the chances of being paid by Uniwide
were dim; (5) ETS asked private respondents to sign employment contracts
to formalize their previous agreement but said private respondents refused;
and (6) as a result, ETS was constrained to deny employment to private

respondents as it considered the execution of employment contracts part of


management prerogative before employment commences.
ISSUE:
Whether or not employees were project employees?
RULING:
The principal test for determining whether an employee is properly
characterized as "project employee," as distinguished from "regular
employee," is whether or not "the project employee" was assigned to carry
out "a specific project or undertaking," the duration and scope of which were
specified at the time the employees were engaged for that project.8 And as
Article 280 of the Labor Code, defining a regular employee vis--vis a project
employee, would have it:
Art. 280. Regular and casual employment. The provisions of written
agreement to the contrary notwithstanding and regardless of the oral
agreement of the parties, an employment shall be deemed to be regular
where the employee has been engaged to perform activities which are
usually necessary or desirable in the usual business or trade of the employer,
except where the employment has been fixed for a specific project or
undertaking the completion or termination of which has been determined at
the time of the engagement of the employee x x x.
It bears to stress at the outset that ETS admits hiring or employing private
respondents to perform plumbing works for various projects. Given this
postulate, regular employment may reasonably be presumed and it
behooves ETS to prove otherwise, that is, that the employment in question
was contractual in nature ending upon the expiration of the term fixed in the
contract or for a specific project or undertaking. But the categorical finding of
the CA, confirmatory for the most part of that of the labor arbiter, is that not
a single written contract of employment fixing the terms of employment for
the duration of the Uniwide project, or any other project, was submitted by
ETS despite the latters allegations that private respondents were merely
contractual employees. Records of payroll and other pertinent documents,
such as job contracts secured by ETS showing that private respondents were
hired for specific projects, were also not submitted by ETS.

Moreover, if private respondents were indeed employed as project


employees, petitioners should have had submitted a report of termination
every time their employment was terminated owing to the completion of
each plumbing project. As correctly held by the CA in its Amended Decision,
citing Tomas Lao Construction v. NLRC, ETS failure to report the employment
termination and file the necessary papers after every project completion
tends to support the claim of private respondents about their not being
project employees. Under Policy Instruction No. 20, Series of 1977, the report
must be made to the nearest public office employment. The decision in
Violeta v. NLRC is also apropos, particularly when it held:
[The employer] should have filed as many reports of termination as there
were construction projects actually finished if petitioners [employees] were
indeed project employees, considering that petitioners were hired and again
[hired] for various projects or phases of work therein. Its failure to submit
reports of termination cannot but sufficiently convince us further that
petitioners are truly regular employees. Just as important, the fact that
petitioners had rendered more than one year of service at the time of their
dismissal overturns private respondents allegations that petitioners were
hired for a specific or fixed undertaking for a limited period of time.

CACERES VS. UNIVERSAL ROBINA SUGAR MILLING CORPORATION


G.R. No. 159343, September 28, 2007
FACTS:
Universal Robina Sugar Milling Corporation (respondent) is a corporation
engaged in the cane sugar milling business. Pedy Caseres (petitioner
Caseres) started working for respondent in 1989, while Andito Pael
(petitioner Pael) in 1993. At the start of their respective employments, they
were made to sign a Contract of Employment for Specific Project or
Undertaking. Petitioners' contracts were renewed from time to time; until
May 1999 when they were informed that their contracts will not be renewed
anymore. Petitioners filed a complaint for illegal dismissal, regularization,
incentive leave pay, 13th month pay, damages and attorneys fees.
Issue:
Whether or not the petitioners are seasonal/project/term employees and not
regular employees of respondents
Ruling:
Article 280 of the Labor Code provides:
ART. 280. Regular and Casual Employees. The provision of written
agreement to the contrary notwithstanding and regardless of the oral
agreement of the parties, an employment shall be deemed to be regular
where the employee has been engaged to perform activities which are
usually necessary or desirable in the usual business or trade of the employer,

except where the employment has been fixed for a specific project or
undertaking the completion or termination of which has been determined at
the time of the engagement of the employee or where the work or services
to be performed is seasonal in nature and the employment is for the duration
of the season.
An employment shall be deemed to be casual if it is not covered by the
preceding paragraph: Provided, That, any employee who has rendered at
least one year of service, whether such service is continuous or broken, shall
be considered a regular employee with respect to the activity in which he is
employed and his employment shall continue while such actually exists.
The foregoing provision provides for three kinds of employees: (a) regular
employees or those who have been engaged to perform activities which
are usually necessary or desirable in the usual business or trade of the
employer; (b) project employees or those whose employment has been
fixed for a specific project or undertaking, the completion or termination of
which has been determined at the time of the engagement of the employee
or where the work or services to be performed is seasonal in nature and the
employment is for the duration of the season; and (c) casual employees or
those who are neither regular nor project employees. The principal test for
determining whether an employee is a project employee or a regular
employee is whether the employment has been fixed for a specific project or
undertaking, the completion or termination of which has been determined at
the time of the engagement of the employee. A project employee is one
whose employment has been fixed for a specific project or undertaking, the
completion or termination of which has been determined at the time of the
engagement of the employee or where the work or service to be performed
is seasonal in nature and the employment is for the duration of the season. A
true project employee should be assigned to a project which begins and ends
at determined or determinable times, and be informed thereof at the time of
hiring.
Petitioners contend that respondent's repeated hiring of their services
qualifies them to the status of regular employees.
On this score, the LA ruled:
This is further buttressed by the fact that the relationship between
complainants and the respondent URSUMCO, would clearly reveal that the
very nature of the terms and conditions of their hiring would show that
complainants were required to perform phases of special projects which are

not related to the main operation of the respondent for a definite period,
after which their services are available to any farm owner.
The NLRC, agreeing with the LA, further ruled that:
In the case at bar, We note that complainants never bothered to deny that
they voluntarily, knowingly and willfully executed the contracts of
employment. Neither was there any showing that respondents exercised
moral dominance on the complainants, x x x it is clear that the contracts of
employment are valid and binding on the complainants.
The execution of these contracts in the case at bar is necessitated by the
peculiar nature of the work in the sugar industry which has an off milling
season. The very nature of the terms and conditions of complainants' hiring
reveals that they were required to perform phases of special projects for a
definite period after, their services are available to other farm owners. This is
so because the planting of sugar does not entail a whole year operation, and
utility works are comparatively small during the off-milling season.
Finally, the CA noted:
Petitioner Pedy Caseres first applied with private respondent URSUMCO on
January 9, 1989 as a worker assisting the crane operator at the transloading
station. Upon application, Caseres was interviewed and made to understand
that his employment would be co-terminus with the phase of work to which
he would be then assigned, that is until February 5, 1989 and thereafter he
would be free to seek employment elsewhere. Caseres agreed and signed
the contract of employment for specific project or undertaking. After an
absence of more than five (5) months, Caseres re-applied with respondent as
a seasonal project worker assisting in the general underchassis
reconditioning to transport units on July 17, 1989. Like his first assignment,
Caseres was made to understand that his services would be co-terminus with
the work to which he would be then assigned that is from July 17, 1989 to
July 20, 1989 and that thereafter he is free to seek employment elsewhere to
which Caseres agreed and readily signed the contract of employment for
specific project or undertaking issued to him. Thereafter Caseres voluntarily
signed several other employment contracts for various undertakings with a
determinable period. As in the first contract, Caseres' services were
co-terminus with the work to which he was assigned, and that thereafter, he
was free to seek employment with other sugar millers or elsewhere.

The nature and terms and conditions of employment of petitioner Andito Pael
were the same as that of his co-petitioner Caseres. It must be noted that
there were intervals in petitioners' respective employment contracts, and
that their work depended on the availability of such contracts or projects.
Consequently, the employment of URSUMCO's work force was not permanent
but co-terminous with the projects to which the
employees were assigned and from whose payrolls they were paid
(Palomares vs. NLRC, 277 SCRA 439).
Petitioners' repeated and successive re-employment on the basis of a
contract of employment for more than one year cannot and does not make
them regular employees. Length of service is not the controlling determinant
of the employment tenure of a project employee (Rada vs. NLRC,)
G.R. No. 112629

July 7, 1995

PHILIPPINE NATIONAL CONSTRUCTION CORPORATION (PNCC),


petitioner,
vs.
NATIONAL LABOR RELATIONS COMMISSION, PHILIPPINE OVERSEAS
EMPLOYMENT ADMINISTRATION, BONIFACIO M. ROQUERO, and
ALFREDO I. DAVILA, respondents.
DAVIDE, JR., J.:
FACTS:
Petitioner Philippine National Construction Corporation (PNCC) asks that we
set aside the resolution 1 of public respondent National Labor Relations
Commission (NLRC) in NLRC NCR CA No. 003767-92 dismissing for lack of
merit the appeal from the decision 2 of the Philippine Overseas and
Employment Administration (POEA) in POEA Case No. 90-10-1183 entitled
"Alfredo Davila and Bonifacio Roquero vs. Philippine National Construction
Corporation (PNCC), et al.."
In the complaint filed with the POEA, private respondents Alfredo Davila and
Bonifacio Roquero sought to recover from the petitioner salary, overtime pay,
vacation and sick leave, and completion bonus differentials; Davila further
asked for payment of his salary corresponding to the unexpired portion of his
contract. They therein alleged that they had been working as security guards

of PNCC since 1980. Having passed the criteria set by PNCC for overseas
workers, they were assigned as company security guards at PNCC Iraq
Expressway Project with a salary of US$350.00 a month each. Their contracts
are evidenced by master employment contracts approved by the POEA which
explicitly state:
This is to confirm your employment with the Philippine National Construction
Corporation-Iraq Expressway Project (Employer/Principal)
. . . other relevant data are as follows:
Position
:Company Guard
Salary
:
US$350.00/month
Jobsite
:
Samawah, Iraq
Commencement of contract: Upon Departure
They departed for Iraq on 14 May 1985; however, before they left they were
made to sign printed forms in blank. The necessary papers for their overseas
assignment were not given to them not until they were already at the Manila
International Airport. They found out to their disgust that contrary to the
master employment plan, the printed forms they had earlier signed in blank
already contain an entry that their salary rate is US$260.00 a month. Thus,
private respondent Roquero received only US$260.00 as monthly salary
during his entire two-year assignment in Iraq and three-week extended
period of assignment therein. Private respondent Davila received the same
salary until he was repatriated prior to the expiration of his contract due to a
reduction of work force. For their four-hour daily overtime work, they were
paid only two-hour overtime pay at the rate of US$260.00 per month.
The PNCC resisted the complaint by claiming that the so-called Master
Employment Contracts relied upon by the private respondents were but
notices or offers for overseas employment, and mere offers without
acceptance by them do not constitute contracts of employment. The
contracts which bound them were those providing for a salary of US$260.00
per month.
ISSUE:
Whether or not the private respondents are employees of PNCC
RULING:

It is true that the only way by which a labor case may reach this Court is
through a petition for certiorari under Rule 65 of the Rules of Court. 8 It must,
however, be shown that the NLRC has acted without or in excess of
jurisdiction, or with grave abuse of discretion, and there is no appeal, nor any
plain, speedy, and adequate remedy in the ordinary course of law. 9
Section 14, Rule VII of the New Rules of Procedure of the NLRC allows an
aggrieved party to file a motion for the reconsideration of any order,
resolution, or decision of the Commission based on palpable or patent errors.
Such a motion constitutes a plain, speedy, and adequate remedy which the
aggrieved party may avail of.
It is settled that before certiorari may be availed of, the petitioner must have
filed a motion for the reconsideration of the order or act complained of to
enable the tribunal, board, or office concerned to pass upon and correct its
mistakes without the intervention of the higher court. 10 The petitioner has
not endeavored to show any justifiable reason why it did not file a motion for
reconsideration to give the NLRC an opportunity to re-examine its resolution.
At any rate, at the bottom of the petitioner's grievance is an issue of fact. It
is doctrinally entrenched that the factual findings of labor officials are
conclusive and binding on this Court when supported by substantial
evidence. 11 An examination of the decision of the POEA, which was affirmed
by the NLRC, discloses that the findings of facts therein are supported by
substantial evidence. Hence, they can no longer be disturbed by this Court.
Besides, in an earlier case brought by the petitioner and involving the same
issue but with other employees similarly situated as the private respondents,
12 this Court upheld the resolution of the NLRC affirming the POEA findings
as follows:
. . . The only dispute which remains unsolved is whether or not the monthly
salary of herein complainants is US$350.00 a month or US$260.00.
As correctly invoked by complainants paragraph (i) of Article 34 of the Labor
Code prohibits the substitution or alteration of employment contracts
approved and verified by the Department of Labor from the time (of) the
actual signing thereof by the parties up to and including the period of
expiration of the same without the approval of the Department of Labor.

With regard to the first issue in this case the approved contract of
employment of the herein complainants with the respondent is US$350.00 a
month. This can be inferred from the POEA approved contract of employment
and by the certification issued by respondent's chief recruiting officer. This
being so, herein complainants have the right to be paid as monthly salaries
the aforementioned amount.
Complainants having been granted voluntarily by the respondent a two-hour
daily overtime (Exh. "G", "G-1") during the durations of their contract, are
also entitled to be paid thereto based on the monthly salaries of US$350.00
and not US$260.00.
The petitioner's contention that the private respondents' claims are barred
by laches do not deserve even a short shrift.
G.R. No. 106090
February 28, 1994
RICARDO FERNANDEZ, petitioner,
vs.
NATIONAL LABOR RELATIONS COMMISSION and D. M. CONSUNJI,
INC., respondents.
NOCON, J.:
FACTS:
Petitioner was hired as a laborer at the D.M. Consunji, Inc., a construction
firm, on November 5, 1974. He became a skilled welder and worked for
private respondent until March 23, 1986 when his employment was
terminated on the ground that the project petitioner had been assigned to
was already completed and there was no more work for him to do.
Skeptic of private respondent's reason, petitioner brought his plight before
the Labor Arbiter who consolidated the same with three (3) other separate
complaints for illegal dismissal and various money claims against private
respondent. After filing their respective position papers and other documents
pertinent to their causes/defenses, the parties agreed to submit the case for
decision based on record.

On May 12, 1988, Labor Arbiter Fernando V. Cinco rendered a decision,


finding that complainants worked continuously in various projects ranging
from five (5) to twenty (20) years and belonged to a work pool.
ISSUE:
Whether or not petitioner belonged in a work pool?
RULING:
Noteworthy in this case is the fact that herein private respondent's lay-off
reports and the termination reports were duly submitted to the then Ministry
of Labor and Employment everytime a project was completed in accordance
with Policy Instruction No. 20, which provides:
Project employees are not entitled to termination pay if they are terminated
as a result of the completion of the project or any phase thereof in which
they are employed, regardless of the number of projects in which they have
been employed by a particular construction company. Moreover, the
company is not required to obtain a clearance from the Secretary of Labor in
connection with such termination. What is required of the company is a
report to the nearest Public Employment Office for statistical purposes.
The presence of this factor makes this case different from the cases decided
by the Court where the employees were deemed regular employees. The
cases of Ochoco v. National Labor Relations Commission, 5 Philippine
National Construction Corporation v. National Labor Relations Commission, 6
Magante v. National Labor Relations Commission, 7 and Philippine National
Construction Corporation v. National Labor Relations, et al., 8 uniformly held
that the failure of the employer to report to the nearest employment office
the termination of workers everytime a project is completed proves that the
employees are not project employees. Contrariwise, the faithful and regular
effort of private respondent in reporting every completion of its project and
submitting the lay-off list of its employees proves the nature of employment
of the workers involved therein as project employees. Given this added
circumstance behind petitioner's employment, it is clear that he does not
belong to the work pool from which the private respondent would draw
workers for assignment to other projects at its discretion.

ALEJANDRO MARAGUINOT, JR. AND PAUILINO ENERO v. NLRC, VIC DEL


ROSARIO, VIVA FILMS
GR No. 120969
Facts:
Maraguinot and Enero were separately hired by Vic Del Rosario under Viva
Films as part of the filming crew. Sometime in May 1992, sought the
assistance of their supervisor to facilitate their request that their salary be
adjusted in accordance with the minimum wage law.
On June 1992, Mrs. Cesario, their supervisor, told them that Mr. Vic Del
Rosario would agree to their request only if they sign a blank employment
contract. Petitioners refused to sign such document. After which, the Mr.
Enero was forced to go on leave on the same month and refused to take him
back when he reported for work. Mr. Maraguinot on the other hand was
dropped from the payroll but was returned days after. He was again asked to
sign a blank employment contract but when he refused, he was terminated.
Consequently, the petitioners sued for illegal dismissal before the Labor
Arbiter. The private respondents claim the following: (a) that VIVA FILMS is
the trade name of VIVA PRODUCTIONS, INC. and that it was primarily
engaged in the distribution & exhibition of movies- but not then making of
movies; (b) That they hire contractors called producers who act as
independent contractors as that of Vic Del Rosario; and (c) As such, there is
no employee-employer relation between petitioners and private respondents.

The Labor Arbiter held that the complainants are employees of the private
respondents. That the producers are not independent contractor but should
be considered as labor-only contractors and as such act as mere agent of the
real employer. Thus, the said employees are illegally dismissed.
The private respondents appealed to the NLRC which reversed the decision
of the Labor Arbiter declaring that the complainants were project employees
due to the ff. reasons: (a) Complainants were hired for specific movie
projects and their employment was co-terminus with each movie project;
(b)The work is dependent on the availability of projects. As a result, the total
working hours logged extremely varied; (c) The extremely irregular working
days and hours of complainants work explains the lump sum payment for
their service; and (d) The respondents alleged that the complainants are not
prohibited from working with other movie companies whenever they are not
working for the independent movie producers engaged by the respondents.
A motion for reconsideration was filed by the complainants but was denied
by NLRC. In effect, they filed an instant petition claiming that NLRC
committed a grave abuse of discretion in: (a) Finding that petitioners were
project employees; (b) Ruling that petitioners were not illegally dismissed;
and (c) Reversing the decision of the Labor Arbiter.
In the instant case, the petitioners allege that the NLRC acted in total
disregard of evidence material or decisive of the controversy.
Issues:
(a) W/N there exist an employee- employer relationship between the
petitioners and the private respondents.
(b) W/N the private respondents are engaged in the business of making
movies.
(c) W/N the producer is a job contractor.
Held:
There exist an employee- employer relationship between the petitioners and
the private respondents because of the ff. reasons that nowhere in the
appointment slip does it appear that it was the producer who hired the crew

members. Moreover, it was VIVAs corporate name appearing on heading of


the slip. It can likewise be said that it was VIVA who paid for the petitioners
salaries.
Respondents also admit that the petitioners were part of a work pool wherein
they attained the status of regular employees because of the ff. requisites:
(a) There is a continuous rehiring of project employees even after cessation
of a project; (b) The tasks performed by the alleged project employees are
vital, necessary and indispensable to the usual business or trade of the
employer; and (c) However, the length of time which the employees are
continually re-hired is not controlling but merely serves as a badge of regular
employment.
Since the producer and the crew members are employees of VIVA and that
these employees works deal with the making of movies. It can be said that
VIVA is engaged of making movies and not on the mere distribution of such.
The producer is not a job contractor because of the ff. reasons: (Sec. Rule VII,
Book III of the Omnibus Rules Implementing the Labor Code.)
a. A contractor carries on an independent business and undertakes the
contract work on his own account under his own responsibility according to
his own manner and method, free from the control and direction of his
employer or principal in all matters connected with the performance of the
work except as to the results thereof. The said producer has a fix time frame
and budget to make the movies.
b. The contractor should have substantial capital and materials necessary to
conduct his business. The said producer, Del Rosario, does not have his own
tools, equipment, machinery, work premises and other materials to make
motion pictures. Such materials were provided by VIVA.
It can be said that the producers are labor-only contractors. Under Article
106 of the Labor Code (reworded) where the contractor does not have the
requisites as that of the job contractors.

G.R. No. 121605

February 2, 2000

PAZ MARTIN JO and CESAR JO, petitioners,


vs.
NATIONAL LABOR RELATIONS COMMISSION and PETER MEJILA,
respondents.
QUISUMBING, J.:
FACTS:
Private respondent Peter Mejila worked as barber on a piece rate basis at
Dina's Barber Shop. In 1970, the owner, Dina Tan, sold the barbershop to
petitioners Paz Martin Jo and Cesar Jo. All the employees, including private
respondent, were absorbed by the new owners. The name of the barbershop
was changed to Windfield Barber Shop.
The owners and the barbers shared in the earnings of the barber shop. The
barbers got two-thirds (2/3) of the fee paid for every haircut or shaving job
done, while one-third (1/3) went to the owners of the shop.
In 1977, petitioners designated private respondent as caretaker of the shop
because the former caretaker became physically unfit. Private respondent's
duties as caretaker, in addition to his being a barber, were: (1) to report to
the owners of the barbershop whenever the airconditioning units
malfunctioned and/or whenever water or electric power supply was
interrupted, (2) to call the laundry woman to wash dirty linen; (3) to
recommend applicants for interview and hiring; (4) to attend to other needs
of the shop. For this additional job, he was given an honorarium equivalent to
one-third (1/3) of the net income of the shop.1wphi1.nt

When the building occupied by the shop was demolished in 1986, the
barbershop closed. But soon a place nearby was rented by petitioners and
the barbershop resumed operations as Cesar's Palace Barbershop and
Massage Clinic. In this new location, private respondent continued to be a
barber and caretaker, but with a fixed monthly honorarium as caretaker, to
wit: from February 1986 to 1990 P700; from February 1990 to March 1991
P800; and from July 1992 P1,300.
In November 1992, private respondent had an altercation with his co-barber,
Jorge Tinoy. The bickerings, characterized by constant exchange of personal
insults during working hours, became serious so that private respondent
reported the matter to Atty. Allan Macaraya of the labor department. The
labor official immediately summoned private respondent and petitioners to a
conference. Upon investigation, it was found out that the dispute was not
between private respondent and petitioners; rather, it was between the
former and his fellow barber. Accordingly, Atty. Macaraya directed
petitioners' counsel, Atty. Prudencio Abragan, to thresh out the problem.
During the mediation meeting held at Atty. Abragan's office a new twist was
added. Despite the assurance that he was not being driven out as caretakerbarber, private respondent demanded payment for several thousand pesos
as his separation pay and other monetary benefits. In order to give the
parties enough time to cool off, Atty. Abragan set another conference but
private respondent did not appear in such meeting anymore.
Meanwhile, private respondent continued reporting for work at the
barbershop. But, on January 2, 1993, he turned over the duplicate keys of the
shop to the cashier and took away all his belongings therefrom. On January
8, 1993, he began working as a regular barber at the newly opened
Goldilocks Barbershop also in Iligan City.
On January 12, 1993, private respondent filed a complaint2 for illegal
dismissal with prayer for payment of separation pay, other monetary
benefits, attorney's fees and damages. Significantly, the complaint did not
seek reinstatement as a positive relief.

ISSUE:

Whether or not Magcalas is a regular employee, not just a project employee


RULING:
Absent a clear showing that petitioners and private respondent had intended
to pursue a relationship of industrial partnership, we entertain no doubt that
private respondent was employed by petitioners as caretaker-barber. Initially,
petitioners, as new owners of the barbershop, hired private respondent as
barber by absorbing the latter in their employ. Undoubtedly, the services
performed by private respondent as barber is related to, and in the pursuit of
the principal business activity of petitioners. Later on, petitioners tapped
private respondent to serve concurrently as caretaker of the shop. Certainly,
petitioners had the power to dismiss private respondent being the ones who
engaged the services of the latter. In fact, private respondent sued
petitioners for illegal dismissal, albeit contested by the latter. As a caretaker,
private respondent was paid by petitioners wages in the form of honorarium,
originally, at the rate of one-third (1/3) of the shop's net income but
subsequently pegged at a fixed amount per month. As a barber, private
respondent earned two-thirds (2/3) of the fee paid per haircut or shaving job
done. Furthermore, the following facts indubitably reveal that petitioners
controlled private respondent's work performance, in that: (1) private
respondent had to inform petitioners of the things needed in the shop; (2) he
could only recommend the hiring of barbers and masseuses, with petitioners
having the final decision; (3) he had to be at the shop at 9:00 a.m. and could
leave only at 9:00 p.m. because he was the one who opened and closed it,
being the one entrusted with the key.7 These duties were complied with by
the private respondent upon instructions of petitioners. Moreover, such task
was far from being negligible as claimed by petitioners. On the contrary, it
was crucial to the business operation of petitioners as shown in the
preceding discussion. Hence, there was enough basis to declare private
respondent an employee of petitioners. Accordingly, there is no cogent
reason to disturb the findings of the labor arbiter and NLRC on the existence
of employer-employee relationship between herein private parties.
With regard to the second issue, jurisprudence has laid out the rules and
valid ground for termination of employment. To constitute abandonment,
there must be concurrence of the intention to abandon and some overt acts
from which it may be inferred that the employee concerned has no more
interest in working.8 In other words, there must be a clear, deliberate and

unjustified refusal to resume employment and a clear intention to sever the


employer-employee relationship on the part of the employee.

BRENT SCHOOL, INC.DIMACHE vs. RONALDO ZAMORA and DOROTEO


R. ALEGRE
G.R. No. L-48494 February 5, 1990 en banc
FACTS:
Private respondent Doroteo R. Alegre was engaged as athletic director by
petitioner Brent School, Inc. at a yearly compensation of P20,000.00. The
contract fixed a specific term for its existence, five (5) years, i.e., from July
18, 1971, the date of execution of the agreement, to July 17, 1976.
Subsequent subsidiary agreements dated March 15, 1973, August 28, 1973,
and September 14, 1974 reiterated the same terms and conditions, including
the expiry date, as those contained in the original contract of July 18, 1971.
On April 20,1976, Alegre was given a copy of the report filed by Brent School
with the Department of Labor advising of the termination of his services
effective on July 16, 1976. The stated ground for the termination was
"completion of contract, expiration of the definite period of employment."
Although protesting the announced termination stating that his services were
necessary and desirable in the usual business of his employer, and his
employment lasted for 5 years - therefore he had acquired the status of
regular employee - Alegre accepted the amount of P3,177.71, and signed a
receipt therefor containing the phrase, "in full payment of services for the
period May 16, to July 17, 1976 as full payment of contract."
The Regional Director considered Brent School's report as an application for
clearance to terminate employment (not a report of termination), and

accepting the recommendation of the Labor Conciliator, refused to give such


clearance and instead required the reinstatement of Alegre, as a "permanent
employee," to his former position without loss of seniority rights and with full
back wages.
ISSUE:
Whether or not the provisions of the Labor Code, as amended, have
anathematized "fixed period employment" or employment for a term.
RULING:
Respondent Alegre's contract of employment with Brent School having
lawfully terminated with and by reason of the expiration of the agreed term
of period thereof, he is declared not entitled to reinstatement.
The employment contract between Brent School and Alegre was executed on
July 18, 1971, at a time when the Labor Code of the Philippines (P.D. 442)
had not yet been promulgated. At that time, the validity of term employment
was impliedly recognized by the Termination Pay Law, R.A. 1052, as amended
by R.A. 1787. Prior, thereto, it was the Code of Commerce (Article 302) which
governed employment without a fixed period, and also implicitly
acknowledged the propriety of employment with a fixed period. The Civil
Code of the Philippines, which was approved on June 18, 1949 and became
effective on August 30,1950, itself deals with obligations with a period. No
prohibition against term-or fixed-period employment is contained in any of its
articles or is otherwise deducible therefrom.
It is plain then that when the employment contract was signed between
Brent School and Alegre, it was perfectly legitimate for them to include in it a
stipulation fixing the duration thereof Stipulations for a term were explicitly
recognized as valid by this Court.
The status of legitimacy continued to be enjoyed by fixed-period
employment contracts under the Labor Code (PD 442), which went into effect
on November 1, 1974. The Code contained explicit references to fixed period
employment, or employment with a fixed or definite period. Nevertheless,
obscuration of the principle of licitness of term employment began to take
place at about this time.

Article 320 originally stated that the "termination of employment of


probationary employees and those employed WITH A FIXED PERIOD shall be
subject to such regulations as the Secretary of Labor may prescribe." Article
321 prescribed the just causes for which an employer could terminate "an
employment without a definite period." And Article 319 undertook to define
"employment without a fixed period" in the following manner: where the
employee has been engaged to perform activities which are usually
necessary or desirable in the usual business or trade of the employer, except
where the employment has been fixed for a specific project or undertaking
the completion or termination of which has been determined at the time of
the engagement of the employee or where the work or service to be
performed is seasonal in nature and the employment is for the duration of
the season.
Subsequently, the foregoing articles regarding employment with "a definite
period" and "regular" employment were amended by Presidential Decree No.
850, effective December 16, 1975.
Article 320, dealing with "Probationary and fixed period employment," was
altered by eliminating the reference to persons "employed with a fixed
period," and was renumbered (becoming Article 271).
As it is evident that Article 280 of the Labor Code, under a narrow and literal
interpretation, not only fails to exhaust the gamut of employment contracts
to which the lack of a fixed period would be an anomaly, but would also
appear to restrict, without reasonable distinctions, the right of an employee
to freely stipulate with his employer the duration of his engagement, it
logically follows that such a literal interpretation should be eschewed or
avoided. The law must be given a reasonable interpretation, to preclude
absurdity in its application. Outlawing the whole concept of term
employment and subverting to boot the principle of freedom of contract to
remedy the evil of employer's using it as a means to prevent their employees
from obtaining security of tenure is like cutting off the nose to spite the face
or, more relevantly, curing a headache by lopping off the head.
Such interpretation puts the seal on Bibiso upon the effect of the expiry of an
agreed period of employment as still good rulea rule reaffirmed in the
recent case of Escudero vs. Office of the President (G.R. No. 57822, April 26,
1989) where, in the fairly analogous case of a teacher being served by her

school a notice of termination following the expiration of the last of three


successive fixed-term employment contracts, the Court held:
Reyes (the teacher's) argument is not persuasive. It loses sight of the fact
that her employment was probationary, contractual in nature, and one with a
definitive period. At the expiration of the period stipulated in the contract,
her appointment was deemed terminated and the letter informing her of the
non-renewal of her contract is not a condition sine qua non before Reyes may
be deemed to have ceased in the employ of petitioner UST. The notice is a
mere reminder that Reyes' contract of employment was due to expire and
that the contract would no longer be renewed. It is not a letter of
termination.
Paraphrasing Escudero, respondent Alegre's employment was terminated
upon the
expiration of his last contract with Brent School on July 16, 1976 without the
necessity of any notice. The advance written advice given the Department of
Labor with copy to said petitioner was a mere reminder of the impending
expiration of his contract, not a letter of termination, nor an application for
clearance to terminate which needed the approval of the Department of
Labor to make the termination of his services effective. In any case, such
clearance should properly have been given, not denied.

G.R. No. 79182 September 11, 1991


PNOC-ENERGY DEVELOPMENT CORPORATION, petitioner,
vs.
NATIONAL LABOR RELATIONS COMMISSION (Third Division) and
DANILO MERCADO, respondents.
PARAS, J.
FACTS:
In June 1985, Danilo Mercado was dismissed by PNOC-Energy Development
Corporation (PNOC-EDC) due to serious acts of dishonesty allegedly
committed by Mercado. Mercado then filed a complaint for illegal dismissal
against PNOC-EDC. PNOC-EDC filed a motion to dismiss on the ground that
the Labor arbiter and/or the National Labor Relations Commission (NLRC) has
no jurisdiction over PNOC-EDC because it is a subsidiary of the Philippine
National Oil Company (PNOC), a government owned or controlled
corporation, and as a subsidiary, it is also a GOCC and as such, the proper
forum for Mercados suit is the Civil Service Commission.
ISSUE:
Whether or not PBOC-EDC is correct.
HELD:

No. The issue in this case has been decided already in the case of PNOC-EDC
vs Leogardo. It is true that PNOC is a GOCC and that PNOC-EDC, being a
subsidiary of PNOC, is likewise a GOCC. It is also true that under the 1973
Constitution, all GOCCs are under the jurisdiction of the CSC. However, the
1987 Constitution change all this as it now provides:
The Civil Service embraces all branches, subdivisions, instrumentalities and
agencies of the Government, including government-owned or controlled
corporations with original charters. (Article IX-B, Section 2 [1])
Hence, the above provision sets the rule that the mere fact that a
corporation is a GOCC does not automatically place it under the CSC. Under
this provision, the test in determining whether a GOCC is subject to the Civil
Service Law is the manner of its creation such that government corporations
created by special charter are subject to its provisions while those
incorporated under the general Corporation Law are not within its coverage.
In the case at bar, PNOC-EDC, even though it is a GOCC, was incorporated
under the general Corporation Law it does not have its own charter, hence,
it is under the jurisdiction of the MOLE.
Even though the facts of this case occurred while the 1973 Constitution was
still in force, the provisions of the 1987 Constitution regarding the legal
matters [procedural aspect] are applicable because it is the law in force at
the time of the decision.

G.R. No. 101013

February 2, 1993

ABRAHAM B. BLANCAFLOR, ANASTACIO T. MERCADO, LEONARDO


DANTES, ANA B. AGAIN, MARVIN B. VICENTE, ROBERTO Z. CALICA,
MARYLYN M. KARGANILLA and LYDIA S. YUSAY, petitioners,
vs.
NATIONAL LABOR RELATIONS COMMISSION, GREGORIO ARANETA
UNIVERSITY FOUNDATION and ILUMINADO G. VALENCIA,
respondents.
REGALADO, J.:
FACTS:
On March 15, 1983, Cesar A. Mijares, the former president of respondent
Gregorio Araneta University Foundation (GAUF), sent a letter to the then
Minister of Labor and Employment requesting approval of the Reorganization,
Retrenchment and Restructuring (hereinafter referred to as RRR) Program of
the GAUF on the ground of serious business losses and financial reverses
being experienced by the university. 1
In a letter dated March 29, 1983, Minister Blas F. Ople approved the RRR
Program without any serious objection, but with the requirement that the
implementation thereof shall be instituted without prejudice to whatever
benefits may have accrued in favor of the employees concerned. 2
Petitioners in the case at bar are regular members of the faculty of
respondent university and were concurrently holding administrative positions
as dean, department heads and institute secretaries therein. In the
implementation of the RRR Program effective January 1, 1984, herein

petitioners were retired but subsequently rehired. Their appointment to their


administrative positions as dean, department heads and institute secretaries,
respectively, had been extended by private respondent from time to time
until the expiration of their last appointment on May 31, 1988.
Private respondent GAUF appealed the decision to the National Labor
Relations Commission (NLRC) which rendered its decision dated May 24,
1991, reversing the labor arbiter's aforestated decision and dismissing
petitioners' complaint for lack of merit. 4 Petitioners motion for
reconsideration and related reliefs was denied in the resolution of the NLRC
dated July 23, 1991.
ISSUE:
Whether the termination of Blancaflor is valid?
RULING:
Yes, there was no illegal dismissal. Petitioners herein were dismissed by
reason of the expiration of their contracts of employment. Petitioners'
appointments as dean, department heads and institute secretaries were for
fixed terms of definite periods as shown by their respective contracts of
employment, which all expired on the same date, Herein petitioners
voluntarily signed the appointments extended to them as attested by their
signature over the word "conforme" in their contracts of employment. As we
observed in Brent, it is the practice and policy of educational institutions that
appointment to the positions of department heads and other high
administrative offices are held by faculty members only on a temporary or
non-permanent basis either within a specified term or at the pleasure of the
school head or board of regents. There is nothing whatever amiss in said
practice of having teachers serve as administrative officials for a fixed term
or in a non-permanent capacity in order to accord to as many of the teaching
staff as possible the opportunity to serve as dean, principal, or administrative
officer of one type or another.
The alleged lack of notice of termination to petitioners is of no consequence.
Petitioners were lawfully terminated upon the expiration of their contracts
with respondent without the necessity of any notice. When the contract
specifies the period of its duration it terminates on the expiration of such
period. A contract for employment for a definite period terminates by its own

term at the end of such period. 14 The general notice of termination given by
respondent university to petitioners was a mere reminder that their contracts
of employment were due to expire and that the contract would no longer be
renewed. 15 Further, it must be noted that after the employment contracts of
herein petitioners as administrative officers expired, they were retained as
faculty members by private respondent.
On the claims of herein petitioners for separation or retirement pay by
reason of the RRR Program of 1984, the contention of private respondents
that petitioners are not entitled to the same, since they were not separated,
is not well-taken. The right of herein petitioners to claim the said benefits
under the 1984 RRR Program of respondent university is unquestionably
evident.
Philips Semiconductor vs. Fardiquela, G.R. No. 141717, April 14,
2004
Facts:
The petitioner Philips Semiconductors (Phils.), Inc. is a domestic corporation
engaged in the production and assembly of semiconductors such as power
devices, RF modules, CATV modules, RF and metal transistors and glass
diods. It caters to domestic and foreign corporations that manufacture
computers, telecommunications equipment and cars. Aside from contractual
employees, the petitioner employed 1,029 regular workers. The employees
were subjected to periodic performance appraisal based on output, quality,
attendance and work attitude. One was required to obtain a performance
rating of at least 3.0 for the period covered by the performance appraisal to
maintain good standing as an employee. On May 8, 1992, respondent Eloisa
Fadriquela executed a Contract of Employment with the petitioner in which
she was hired as a production operator with a daily salary of P118. Her initial
contract was for a period of three months up to August 8, 1992, but was
extended for two months when she garnered a performance rating of 3.15.
Her contract was again renewed for two months or up to December 16,
1992, when she received a performance rating of 3.8.After the expiration of
her third contract, it was extended anew, for three months, that is, from
January 4, 1993 to April 4, 1993. After garnering a performance rating of 3.4,
the respondents contract was extended for another three months, that is,
from April 5, 1993 to June 4, 1993. She, however, incurred five absences in
the month of April, three absences in the month of May and four absences in
the month of June. Line supervisor Shirley F. Velayo asked the respondent

why she incurred the said absences, but the latter failed to explain her side.
The respondent was warned that if she offered no valid justification for her
absences, Velayo would have no other recourse but to recommend the nonrenewal of her contract. The respondent still failed to respond, as a
consequence of which her performance rating declined to 2.8. Velayo
recommended to the petitioner that the respondents employment be
terminated due to habitual absenteeism, in accordance with the Company
Rules and Regulations. Thus, the respondents contract of employment was
no longer renewed.
Issues and Rulings:
(a) whether or not the respondent was still a contractual employee of the
petitioner as of June 4, 1993;
The two kinds of regular employees under the law are (1) those engaged to
perform activities which are necessary or desirable in the usual business or
trade of the employer; and (2) those casual employees who have rendered at
least one year of service, whether continuous or broken, with respect to the
activities in which they are employed. The primary standard to determine a
regular employment is the reasonable
connection between the particular activity performed by the employee in
relation to the business or trade of the employer. The test is whether the
former is usually necessary or desirable in the usual business or trade of the
employer. If the employee has been performing the job for at least one year,
even if the performance is not continuous or merely intermittent, the law
deems the repeated and continuing need for its performance as sufficient
evidence of the necessity, if not indispensability of that activity to the
business of the employer. Hence, the employment is also considered regular,
but only with respect to such activity and while such activity exists.[22] The
law does not provide the qualification that the employee must first be issued
a regular appointment or must be declared as such before he can acquire a
regular employee status. In this case, the respondent was employed by the
petitioner on May 8, 1992 as production operator. She was assigned to wire
building at the transistor division. There is no dispute that the work of the
respondent was necessary or desirable in the business or trade of the
petitioner.
She remained under the employ of the petitioner without any interruption
since May 8, 1992 to June 4, 1993 or for one (1) year and twenty-eight (28)

days. The original contract of employment had been extended or renewed


for four times, to the same position, with the same chores. Such a continuing
need for the services of the respondent is sufficient evidence of the necessity
and indispensability of her services to the petitioners business. By operation
of law, then, the respondent had attained the regular status of her
employment with the petitioner, and is thus entitled to security of tenure as
provided for in Article 279 of the Labor Code which reads:
(b) whether or not the petitioner dismissed the respondent from her
employment;
if so, whether or not she was accorded the requisite notice and investigation
prior to her dismissal; and
On the second and third issues, we agree with the appellate court that the
respondent was dismissed by the petitioner without the requisite notice and
without any formal investigation. Given the factual milieu in this case, the
respondents dismissal from employment for incurring five (5) absences in
April 1993, three (3) absences in May 1993 and four (4) absences in June
1993, even if true, is too harsh a penalty. We do agree that an employee may
be dismissed for violation of reasonable regulations/rules promulgated by the
employer. Dismissal is the ultimate penalty that can be meted to an
employee. Where a penalty less punitive would suffice, whatever missteps
may have been committed by the worker ought not to be visited with a
consequence so severe such as dismissal from employment. For, the
Constitution guarantees the right of workers to security of tenure. The
misery and pain attendant to the loss of jobs then could be avoided if there
be acceptance of the view that under certain circumstances of the case the
workers should not be deprived of their means of livelihood.

Price, et al., v Innodata Phils., G.R. No. 178505, September 30, 2008

Facts:
INNODATA had since ceased operations due to business losses in June 2002.
Petitioners Cherry J. Price, Stephanie G. Domingo, and Lolita Arbilera were
employed as formatters by INNODATA. The parties executed an employment
contract denominated as a Contract of Employment for a Fixed Period,
stipulating that the contract shall be effective from FEB. 16, 1999 to FEB. 16,
2000 a period of ONE YEAR. On 16 February 2000, the HRAD Manager of
INNODATA wrote petitioners informing them of their last day of work, at the
end of the close of business hours On February 16, 2000. According to
INNODATA, petitioners employment already ceased due to the end of their
contract.
On 22 May 2000, petitioners filed a Complaint for illegal dismissal and
damages against respondents. Petitioners claimed that they should be
considered regular employees since their positions as formatters were
necessary and desirable to the usual business of INNODATA as an encoding,
conversion and data processing company. Petitioners finally argued that they
could not be considered project employees considering that their
employment was not coterminous with any project or undertaking, the
termination of which was predetermined. Respondents asserted that

petitioners were not illegally dismissed, for their employment was terminated
due to the expiration of their terms of employment.
The Labor Arbiter issued its Decision finding petitioners complaint for illegal
dismissal and damages meritorious.
Respondent INNODATA appealed the Labor Arbiters Decision to the NLRC.
The NLRC reversed the Labor Arbiters Decision dated 17 October 2000, and
absolved INNODATA of the charge of illegal dismissal.
On 25 September 2006, the Court of Appeals promulgated its Decision
sustaining the ruling of the NLRC that petitioners were not illegally
dismissed. Hence, this petition.
Issues:
Whether petitioners were illegally dismissed by respondents
Whether petitioners were hired by INNODATA under valid fixed-term
employment contracts
Ruling:
The Court finds merit in the present Petition. There were no valid fixed-term
contracts and petitioners were regular employees of the INNODATA who
could not be dismissed except for just or authorized cause.
The employment status of a person is defined and prescribed by law and not
by what the parties say it should be. Equally important to consider is that a
contract of employment is impressed with public interest such that labor
contracts must yield to the common good. Thus, provisions of applicable
statutes are deemed written into the contract, and the parties are not at
liberty to insulate themselves and their relationships from the impact of labor
laws and regulations by simply contracting with each other. Regular
employment has been defined by Article 280 of the Labor
Code, as amended, which reads: Art. 280. Regular and Casual Employment.
The provisions of written agreement to the contrary notwithstanding and
regardless of the oral agreement of the parties, an employment shall be
deemed to be regular where the employee has been engaged to perform
activities which are usually necessary or desirable in the usual business or
trade of the employer, except where the employment has been fixed for a

specific project or undertaking the completion or termination of which has


been determined at the time of engagement of the employee or where the
work or services to be performed is seasonal in nature and employment is for
the duration of the season.
An employment shall be deemed to be casual if it is not covered by the
preceding paragraph. Provided, That, any employee who has rendered at
least one year of service, whether such service is continuous or broken, shall
be considered a regular employee with respect to the activity in which he is
employed and his employment shall continue while such activity exists.
Based on the afore-quoted provision, the following employees are accorded
regular status: (1) those who are engaged to perform activities which are
necessary or desirable in the usual business or trade of the employer,
regardless of the length of their employment; and (2) those who were initially
hired as casual employees, but have rendered at least one year of service,
whether continuous or broken, with respect to the activity in which they are
employed.
Undoubtedly, petitioners belong to the first type of regular employees.
Under Article 280 of the Labor Code, the applicable test to determine
whether an employment should be considered regular or non-regular is the
reasonable connection between the particular activity performed by the
employee in relation to the usual business or trade of the employer.
In the case at bar, petitioners were employed by INNODATA on 17 February
1999 as formatters. The primary business of INNODATA is data encoding, and
the formatting of the data entered into the computers is an essential part of
the process of data encoding. Formatting organizes the data encoded,
making it easier to understand for the clients and/or the intended end users
thereof. Undeniably, the work performed by petitioners was necessary or
desirable in the business or trade of INNODATA.
However, it is also true that while certain forms of employment require the
performance of usual or desirable functions and exceed one year, these do
not necessarily result in regular employment under Article 280 of the Labor
Code. Under the Civil Code, fixed-term employment contracts are not limited,
as they are under the present Labor Code, to those by nature seasonal or for
specific projects with predetermined dates of completion; they also include

those to which the parties by free choice have assigned a specific date of
termination.
The decisive determinant in term employment is the day certain agreed
upon by the parties for the commencement and termination of their
employment relationship, a day certain being understood to be that which
must necessarily come, although it may not be known when. Seasonal
employment and employment for a particular project are instances of
employment in which a period, where not expressly set down, is necessarily
implied.
While this Court has recognized the validity of fixed-term employment
contracts, it has consistently held that this is the exception rather than the
general rule. More importantly, a fixed-term employment is valid only under
certain circumstances. In Brent, the very same case invoked by respondents,
the Court identified several circumstances wherein a fixed-term is an
essential and natural appurtenance, to wit:
Some familiar examples may be cited of employment contracts which may
be neither for seasonal work nor for specific projects, but to which a fixed
term is an essential and natural appurtenance: overseas employment
contracts, for one, to which, whatever the nature of the engagement, the
concept of regular employment with all that it implies does not appear ever
to have been applied, Article 280 of the Labor Code notwithstanding; also
appointments to the positions of dean, assistant dean, college secretary,
principal, and other administrative offices in educational institutions, which
are by practice or tradition rotated among the faculty members, and where
fixed terms are a necessity without which no reasonable rotation would be
possible. Similarly, despite the provisions of Article 280, Policy Instructions
No. 8 of the Minister of Labor implicitly recognize that certain company
officials may be elected for what would amount to fixed periods, at the
expiration of which they would have to stand down, in providing that these
officials, "x x may lose their jobs as president, executive vice-president or
vice president, etc. because the stockholders or the board of directors for
one reason or another did not reelect them."

[G.R. No. 122178. February 25, 1999]


DANILO DIMABAYAO, petitioner, vs. NATIONAL LABOR RELATIONS
COMMISSION, ISLAND BISCUIT INC. and CHENG SUY EH,
respondents.
BELLOSILLO, J.:
FACTS:
DANILO DIMABAYAO seeks to set aside through this petition for certiorari
under Rule 65 of the 1997 Rules of Civil Procedure the 15 March 1995
Decision and 23 June 1995 Resolution of the National Labor Relations
Commission (NLRC) which modified the Decision of the Labor Arbiter finding
private respondents guilty of having illegally dismissed petitioner from their
employ.
Private respondent Island Biscuit, Inc., is engaged in the manufacture of
biscuits with private respondent Cheng Suy Eh as its General Manager. On 5
April 1983 it employed petitioner with the specific task of operating the
roller, cutting biscuits, sorting out rejects, mashing flour and feeding the flour
mass into its thinning machine.
On 30 July 1992, while petitioner was assigned to sort out rejects, with prior
permission first obtained from his checker, he went to the comfort room to

answer the call of nature and relieve himself, afterwhich he returned to his
work place.
But private respondent Cheng Suy Eh was unhappy seeing
petitioner away from his work station and immediately demanded from him a
written explanation allegedly for abandoning his work. As a matter of policy,
respondent company discourages its employees from going to the comfort
room during working hours for sanitary or hygienic purposes as the company
is engaged in the food business.[1]
The following day, 31 July 1992, Marcela Lok, respondent company's
Personnel Manager, handed petitioner a letter asking him to explain in
writing why he left his work station on 17 and 30 July 1992. Petitioner
verbally explained that he never left his station on 17 July while on 30 July he
only went to the comfort room for a short while to answer the call of nature.
[2] Believing that this denial was enough he did not anymore submit any
written explanation. But, for his inability to submit a written explanation,
petitioner was suspended for fifteen (15) days which he contested before the
Arbitration Branch of the NLRC.
On 20 October 1992 petitioner requested a fellow worker to replace him in
his work station so he could go to the comfort room to relieve himself. Again
private respondent Cheng Suy Eh noticed petitioner's brief absence and so,
upon his return, his manager berated him again and required him to submit
once more a written explanation for allegedly abandoning his work.
Petitioner complied.
Finding petitioner's explanation not satisfactory, respondent company
through its Personnel Officer Marcela Lok served petitioner a notice of
termination.
Petitioner thereafter amended his complaint before the NLRC to include
illegal dismissal among his causes of action in view of his termination from
the service.
On 21 September 1994 the Labor Arbiter declared the suspension of
petitioner valid and legal not because he left his production area to relieve
himself but for his utter disregard of the directive of the manager to submit
his written explanation.
His dismissal however was found illegal, but
because of the strained relationship between the parties, the Labor Arbiter
further held that reinstatement was no longer feasible and thereafter
awarded petitioner a limited back wages for six (6) months without

reinstatement. Thus private respondents were in addition required to pay


petitioner service incentive leave pay of P615.00, proportionate thirteenth
month pay of P2,132.00, separation pay of P14,391.00, and 10% attorney's
fees of P3,632.60.
ISSUE:
Whether or not the acts of Dimabayao constitute willful neglect of duties.
RULING:
No. While it may be true that complainant has been leaving his work area
without permission, this Arbitration Board finds that complainant's habit of
going to the toilet in the morning during production is merely a call of nature
and by force of habit he had to relieve himself. Whether or not the
complainant relieved himself is not the issue. The call of nature is a
reasonable reason for him to leave his work area. Although complainant is
not entirely without fault since he has been leaving his workplace without
permission from his supervisor and his disrespect towards his superiors as
borne out by the reports of his supervisor and guards, the infraction
committed by the complainant is not so grave that would warrant the
ultimate penalty of dismissal . . .
The Labor Arbiter, in effect, opined that a grave injustice would be
committed against the employee if the penalty imposed was grossly
disproportionate to the wrong he committed.[12] At most, a 7-day
suspension without pay - for not asking permission from his supervisor
before answering a call of nature, if that be considered an infraction at all! should have been sufficient penalty for petitioner.
The NLRC also endeavored to justify its decision by taking into account
offenses allegedly committed by petitioner way back in 1990.
These
offenses as enumerated in the NLRC decision were infractions imputed to
petitioner prior to the 17 July, 30 July and 20 October 1992 incidents. As
such, they should have been outrightly ignored by the NLRC in determining
and upholding the validity of petitioner's dismissal since, as may be gleaned
from the termination letter, petitioner's dismissal was based merely on the
17 July, 30 July and 20 October 1992 alleged incidents, without reference to
any infraction committed before then. This only shows that the offenses
attributed to petitioner before 17 July 1992 were mere afterthoughts

conceived in the course of the trial to further justify his dismissal. To refer to
those alleged earlier violations as further grounds for dismissal is
undoubtedly prejudicial to petitioner. Significantly, it would also be doubly
prejudicial to him to penalize him for those committed on 17 and 30 July
1992 as he was already suspended for fifteen (15) days for those infractions.
This, obviously, denied petitioner procedural due process and deprived him
of his right to be heard, to refute and present evidence to controvert such
accusations prior to his actual dismissal from employment.
As a consequence, petitioner is entitled to reinstatement.[13] The postulate
advanced by the Labor Arbiter that there existed "strained relationship"
between the parties, thus barring reinstatement of petitioner, does not hold
water. Strained relationship may be invoked only against employees whose
positions demand trust and confidence, or whose differences with their
employer are of such nature or degree as to preclude reinstatement. In the
instant case, however, the relationship between petitioner, an ordinary
employee, and management was clearly on an impersonal level. Petitioner
did not occupy such a sensitive position as would require complete trust and
confidence, and where personal ill will would foreclose his reinstatement.[14]
But, interestingly, petitioner himself was praying for his reinstatement.

[G.R. No. 144899. February 5, 2004]


ELIZABETH C. BASCON and NOEMI V. COLE, petitioners, vs.
HONORABLE COURT OF APPEALS, METRO CEBU COMMUNITY
HOSPITAL, INC., and GREGORIO IYOY, respondents.
QUISUMBING, J.:
FACTS:
The petitioners in the instant case were employees of private respondent
Metro Cebu Community Hospital, Inc. (MCCH) and members of the
Nagkahiusang Mamumuo sa Metro Cebu Community Hospital (NAMA-MCCH),
a labor union of MCCH employees. Petitioner Elizabeth C. Bascon had been
employed as a nurse by respondent MCCH since May 1984. At the time of her
termination from employment in April 1996, she already held the position of
Head Nurse. The other petitioner, Noemi V. Cole, had been working as a

nursing aide with MCCH since August 1974. Both petitioners were dismissed
by the respondent hospital for allegedly participating in an illegal strike.
The instant controversy arose from an intra-union conflict between the
NAMA-MCCH and the National Labor Federation (NFL), the mother federation
of NAMA-MCCH. In November 1995, NAMA-MCCH asked MCCH to renew their
Collective Bargaining Agreement (CBA), which was set to expire on
December 31, 1995. NFL, however, opposed this move by its local affiliate.
Mindful of the apparent intra-union dispute, MCCH decided to defer the CBA
negotiations until there was a determination as to which of said unions had
the right to negotiate a new CBA.
Believing that their union was the certified collective bargaining agent, the
members and officers of NAMA-MCCH staged a series of mass actions inside
MCCHs premises starting February 27, 1996. They marched around the
hospital putting up streamers, placards and posters.
On March 13 and 19, 1996, the Department of Labor and Employment
(DOLE) office in Region 7 issued two (2) certifications stating that NAMAMCCH was not a registered labor organization. This finding, however, did not
deter NAMA-MCCH from filing a notice of strike with the Region 7 Office of the
National Conciliation and Mediation Board (NCMB). Said notice was, however,
disregarded by the NCMB for want of legal personality of the union.
Meanwhile, the MCCH management received reports that petitioners
participated in NAMA-MCCHs mass actions. Consequently, notices were
served on all union members, petitioners included, asking them to explain in
writing why they were wearing red and black ribbons and roaming around
the hospital with placards. In their collective response dated March 18, 1996,
the union members, including petitioners, explained that wearing armbands
and putting up placards was their answer to MCCHs illegal refusal to
negotiate with NAMA-MCCH.
Subsequently, on March 28, 1996, MCCH notified the petitioners that they
were to be investigated for their activities in the mass actions, with the
hearings being scheduled on March 28, 1996 and April 1, 1996. Petitioners,
however, denied receiving said notices. In a notice dated April 8, 1996,
MCCH ordered petitioners to desist from participating in the mass actions
conducted in the hospital premises with a warning that non-compliance
therewith would result in the imposition of disciplinary measures. Petitioners

again claimed they did not receive said order. Petitioners Bascon and Cole
were then served notices terminating their employment effective April 12,
1996 and April 19, 1996, respectively.
ISSUE:
Whether or not the acts of the Petitioners were just cause for their
termination?
RULING:
No, the acts of the did not constitute petitioners were terminated for
allegedly participating in an illegal strike and gross insubordination to the
order prohibiting them from wearing armbands and putting up placards, not
for ipso facto failing to show up in the scheduled investigation. Thus, the real
issue is whether or not petitioners were validly terminated for (1) allegedly
participating in an illegal strike and/or (2) gross insubordination to the order
to stop wearing armbands and putting up placards.
As to the first ground, Article 264 (a) of the Labor Code provides in part that:
Any union officer who knowingly participates in illegal strike and any
worker or union officer who knowingly participates in the commission of
illegal acts during a strike may be declared to have lost his employment
status (Emphasis ours)
Thus, while a union officer can be terminated for mere participation in an
illegal strike, an ordinary striking employee, like petitioners herein, must
have participated in the commission of illegal acts during the strike
(underscoring supplied). There must be proof that they committed illegal
acts during the strike.[14] But proof beyond reasonable doubt is not required.
Substantial evidence, which may justify the imposition of the penalty of
dismissal, may suffice.
In this case, the Court of Appeals found that petitioners actual participation
in the illegal strike was limited to wearing armbands and putting up placards.
There was no finding that the armbands or the placards contained offensive
words or symbols. Thus, neither such wearing of armbands nor said putting
up of placards can be construed as an illegal act. In fact, per se, they are
within the mantle of constitutional protection under freedom of speech.

Evidence on record shows that various illegal acts were committed by


unidentified union members in the course of the protracted mass action. And
we commiserate with MCCH, patients, and third parties for the damage they
suffered. But we cannot hold petitioners responsible for acts they did not
commit. The law, obviously solicitous of the welfare of the common worker,
requires, before termination may be considered, that an ordinary union
member must have knowingly participated in the commission of illegal acts
during a strike.
As regards the appellate courts finding that petitioners were justly
terminated for gross insubordination or willful disobedience, Article 282 of
the Labor Code provides in part:
An employer may terminate an employment for any of the following causes:
(a) Serious misconduct or willful disobedience by the employee of the lawful
orders of his employer or representative in connection with his work.
However, willful disobedience of the employers lawful orders, as a just cause
for dismissal of an employee, envisages the concurrence of at least two
requisites: (1) the employee's assailed conduct must have been willful, that
is, characterized by a wrongful and perverse attitude; and (2) the order
violated must have been reasonable, lawful, made known to the employee
and must pertain to the duties which he had been engaged to discharge.[15]
In this case, we find lacking the element of willfulness characterized by a
perverse mental attitude on the part of petitioners in disobeying their
employers order as to warrant the ultimate penalty of dismissal. Wearing
armbands and putting up placards to express ones views without violating
the rights of third parties, are legal per se and even constitutionally
protected. Thus, MCCH could have done well to respect petitioners right to
freedom of speech instead of threatening them with disciplinary action and
eventually terminating them.

NAGKAKAISANG LAKAS NG MANGGAGAWA SA KEIHIN (NLMK-OLALIAKMU)


and HELEN VALENZUELA vs. KEIHIN PHILIPPINES CORPORATION,
G.R. No. 171115, August 9, 2010
FACTS:
Petitioner Helen Valenzuela (Helen) was a production associate in
respondent Keihin Philippines Corporation (Keihin), a company engaged in
the production of intake manifold and throttle body used in motor vehicles
manufactured by Honda.
It is a standard operating procedure of Keihin to subject all its
employees to reasonable search before they leave the company premises.[4]
On September 5, 2003, while Helen was about to leave the company

premises, she saw a packing tape near her work area and placed it inside her
bag because it would be useful in her transfer of residence. When the lady
guard on duty inspected Helens bag, she found the packing tape inside her
bag. The guard confiscated it and submitted an incident report[5] dated
September 5, 2003 to the Guard-in-Charge, who, in turn, submitted a
memorandum[6] regarding the incident to the Human Resources and
Administration Department on the same date.
The following day, or on September 6, 2003, respondent company issued a
show cause notice[7] to Helen accusing her of violating F.2 of the companys
Code of Conduct, which says, Any act constituting theft or robbery, or any
attempt to commit theft or robbery, of any company property or other
associates property.
Penalty: D (dismissal).[8]
Paul Cupon, Helens
supervisor, called her to his office and directed her to explain in writing why
no disciplinary action should be taken against her.
Helen, in her explanation,[9] admitted the offense and even manifested that
she would accept whatever penalty would be imposed upon her. She,
however, did not reckon that respondent company would terminate her
services for her admitted offense.[10]
On September 26, 2003, Helen received a notice[11] of disciplinary action
informing her that Keihin has decided to terminate her services.

On October 15, 2003, petitioners filed a complaint[12] against respondent for


illegal dismissal, non-payment of 13th month pay, with a prayer for
reinstatement and payment of full backwages, as well as moral and
exemplary damages. Petitioners alleged that Helens act of taking the
packing tape did not constitute serious misconduct, because the same was
done with no malicious intent.[13] They believed that the tape was not of
great value and of no further use to respondent company since it was
already half used. Although Helen admitted that she took the packing tape,
petitioners claimed that her punishment was disproportionate to her
infraction.
Keihin, on the other hand, maintained that Helen was guilty of serious
misconduct because there was a deliberate act of stealing from the company.
Respondent company also claimed that motive and value of the thing stolen
are irrelevant in this case.

ISSUE:
Whether or not the dismissal of Helen was valid based on serious misconduct
RULING:

It is clear that petitioners failed to include the name of the dismissed


employee Helen Valenzuela in the caption of their petition for certiorari filed
with the CA as well as in the body of the said petition. Instead, they only
indicated the name of the labor union Nagkakaisang Lakas ng Manggagawa
sa Keihin (NLMK-OLALIA) as the party acting on behalf of Helen. As a result,
the CA rightly dismissed the petition based on a formal defect.
Under Section 7, Rule 3 of the Rules of Court, parties in interest without
whom no final determination can be had of an action shall be joined as
plaintiffs or defendants. If there is a failure to implead an indispensable
party, any judgment rendered would have no effectiveness. It is precisely
when an indispensable party is not before the court (that) an action should
be dismissed. The absence of an indispensable party renders all subsequent
actions of the court null and void for want of authority to act, not only as to
the absent parties but even to those present. The purpose of the rules on
joinder of indispensable parties is a complete determination of all issues not
only between the parties themselves, but also as regards other persons who
may be affected by the judgment. A decision valid on its face cannot attain
real finality where there is want of indispensable parties.
At any rate, we are aware that it is the policy of courts to encourage full
adjudication of the merits of an appeal. Dismissal of appeals purely on
technical grounds, especially an appeal by a worker who was terminated and
whose livelihood depends on the speedy disposition of her case, is frowned
upon. Thus, while we affirm the CAs dismissal of the petition for certiorari,
we shall still discuss the substantive aspect of the case and go into the
merits.

[G.R. No. 158232. March 31, 2005]


FUJITSU COMPUTER PRODUCTS CORPORATION OF THE PHILIPPINES
and ERNESTO ESPINOSA, petitioners, vs. THE HONORABLE COURT OF
APPEALS, VICTOR DE GUZMAN and ANTHONY P. ALVAREZ,
respondents.
CALLEJO, SR., J.:
FACTS:
Petitioner Fujitsu Computer Products Corporation of the Philippines (FCPP) is
a corporation organized and existing under Philippine laws with business
address at the Special Export Processing Zone, Carmelray, Canlubang,
Calamba, Laguna. It is engaged in the manufacture of hard disc drives, MR
heads and other computer storage devices for export.[2]

Respondent Victor de Guzman began working for FCPP on September 21,


1997 as Facilities Section Manager. As of 1999, he was also holding in a
concurrent capacity the position of Coordinator ISO 14000 Secretariat and
was receiving a monthly salary of P43,100.00[3]
Respondent Allan Alvarez, on the other hand, was employed as a Senior
Engineer on April 21, 1998. He was assigned at the Facilities Department
under the supervision of respondent De Guzman, and was then earning
P16,800.00.[4]
The garbage and scrap materials of FCPP were collected and bought by the
Saros Trucking Services and Enterprises (Saros). On January 15, 1999,
respondent De Guzman as Facilities Section Manager, for and in behalf of
FCPP, signed a Garbage Collection Agreement[5] with Saros, and the latters
signatory therein was its owner and general manager, Larry Manaig.
Sometime in the third week of July 1999, petitioner Ernesto Espinosa, HRD
and General Affairs Director of FCPP, received a disturbing report from
Manaig. Manaig reported that respondent De Guzman had caused the
anomalous disposal of steel [purlins][6] owned by FCPP.[7] Two of Manaigs
employees, Roberto Pumarez[8] and Ma. Theresa S. Felipe,[9] executed
written statements detailing how respondent De Guzman had ordered the
steel purlins to be brought out.

ISSUE:
Whether or not the acts of Victor De Guzman constitute serious misconduct?
RULING:

After a careful and painstaking study of the records of the case, the Court
rules that the respondents dismissal from employment was not grounded on
any of the just causes enumerated under Article 282 of the Labor Code.
The term trust and confidence is restricted to managerial employees.[33]
In this case, it is undisputed that respondent De Guzman, as the Facilities
Section Manager, occupied a position of responsibility, a position imbued
with trust and confidence. Among others, it was his responsibility to see to it

that the garbage and scrap materials of petitioner FCPP were adequately
managed and disposed of. Thus, respondent De Guzman was entrusted with
the duty of handling or taking care of the property of his employer, i.e., the
steel purlins which the petitioners allege the respondent prematurely
declared as scrap materials.
However, to be valid ground for dismissal, loss of trust and confidence must
be based on a willful breach of trust and founded on clearly established facts.
A breach is willful if it is done intentionally, knowingly and purposely, without
justifiable excuse, as distinguished from an act done carelessly,
thoughtlessly, heedlessly, or inadvertently. It must rest on substantial
grounds and not on the employers arbitrariness, whims, caprices or
suspicion; otherwise, the employee would eternally remain at the mercy of
the employer.[34] Loss of confidence must not be indiscriminately used as a
shield by the employer against a claim that the dismissal of an employee
was arbitrary. And, in order to constitute a just cause for dismissal, the act
complained of must be work-related and shows that the employee concerned
is unfit to continue working for the employer.[35]
The Court had the occasion to reiterate in Nokom v. National Labor Relations
Commission[36] the guidelines for the application of the doctrine of loss of
confidencea. loss of confidence should not be simulated;
b. it should not be used as a subterfuge for causes which are improper,
illegal or unjustified;
c. it may not be arbitrarily asserted in the face of overwhelming evidence to
the contrary; and
d. it must be genuine, not a mere afterthought to justify earlier action taken
in bad faith.[37]
In the case at bar, the grounds relied upon by petitioner FCPP in terminating
the employment of respondent De Guzman are contained in the Inter-Office
Memorandum dated August 23, 1999 which effectively terminated the
latters employment:
We have carefully evaluated your case and we are convinced that you have
committed grave abuse of authority amounting to serious misconduct and
willful breach of trust and confidence.

Based on our findings, as supported by strong and competent evidences, and


contrary to your explanation per your Letter dated July 26, 1999, the
following facts were satisfactorily established:
1.
That sometime in the first week of July 1999, you intimated to Mr.
Roberto Pumarez, Supervisor of Saros Trucking Services, your intention to
buy from Saros the metals which were then piled up and kept inside the Fuji
Electric Philippines compound;
2.
Thereafter, you ordered the metals to be sold to Saros Trucking
Services so that you can buy them (metals) later from Saros at the price of
P3.00 per kg., which price you yourself imposed on them;
3.
However, it turned out later some pieces of metals which you have
earlier declared as scraps and ordered to be sold to Saros were still to be
used in the construction of FCPPs Building B. Thus, on July 10, 1999, while
Saros employees were initially loading the metals, an Engineer of SNK
Philippines, Inc., FCPPs building contractor, stopped them. It was only later
after they were prevented from further loading the metals that you checked
with the SNK personnel if the metals can already be disposed of as scraps
which prove that you have prematurely declared the metals as scrap;
4.
That through Mr. [Adrian] Camcaman, your subordinate Technician, you
instructed the personnel of Saros to deliver the metals to Sta. Rosa Baptist
Church, where you are an active Church member;
5.
That, as of this date, you have not yet settled/paid your obligation to
Saros. That immediately after you were placed under preventive suspension
and to support your explanation that the transaction was between Saros and
Sta. Rosa Baptist Church, you caused, through some people representing to
be members of the Baptist Church and who are unknown to Saros, to issue a
check in favor of Saros. When this failed, another person, representing to be
a member of the Baptist church and who appeared for the first time, went to
the office of Saros and tried to serve a letter addressed to Mr. Larry Manaig,
Saros Proprietor, allegedly inquiring about the total obligation of the Baptist
Church to Saros but, which was again not accepted as, in truth and in fact,
there was really no transaction between Saros and the Sta. Rosa Baptist
Church. All along, it was you and Mr. Camcaman who dealt directly with
Saros.

6.
That in previous occasions, it was reported by Mr. Manaig that you
solicited from him empty drums, pails and corrugated cartons which were all
part of those scraps picked up from FCPP and you never paid any of them, a
fact which you never denied in your explanation which is tantamount to
admission.
Based on the foregoing, it is our well-discerned view that the transaction was
exclusively limited between you and Saros. Except for your self-serving
explanation, you failed miserably to present direct evidence that it was the
Sta. Rosa Baptist church which bought the subject metals from Saros, as
what you want us to believe. At best, your explanation is a mere
afterthought desperately concocted to exculpate yourself.
As Facilities Manager, a very sensitive and confidential position, the nature of
your work demands of you that your actions should not be tainted with any
suspicion or impropriety. However, you failed in this regard and abused your
position to advance your self-interest.

G.R. No. 166039

June 26, 2006

DIGITEL* TELECOMMUNICATIONS PHILIPPINES, INC., JOHNSON


ROBERT L. GO** and ERIC J. SEVERINO,*** Petitioners,
vs.
MARIQUIT SORIANO, Respondent.
CARPIO MORALES, J.:
FACTS:
Mariquit Soriano (Soriano) was hired as Director of Marketing by Digitel
Telecommunications Philippines, Inc. (Digitel). Soriano worked under Vice
President for Business Division Eric J. Severino (Severino) and Senior
Executive Vice President Johnson Robert L. Go (Go). Following a professional
dispute against Severino and Go, Soriano filed a resignation letter which was
accepted by her superiors.

After her resignation, Soriano filed a suit for illegal termination alleging that
she was forced to resign due to professional and sexual harassment. She
alleged that her superiors are preventing her former colleagues in testifying
to the sexual harassment. She produced an affidavit by one of the persons
involved with Digitel stating that the employees of the company were being
forced not to testify against Go and Severino. In defense, Go and Severino
provided witnesses that testified that the acts alleged by Soriano din not
happen.
The Labor Arbiter held that Mariquit voluntarily resigned, thus dismissing the
complaint. On appeal, the NLRC affirmed the findings of the Labor Arbiter.
The Court of Appeals reversed the decision of NLRC. Hence,this petition.
ISSUE:
Whether or not the Soriano was forced to resign, due to professional and
sexual harassment, thus amounting to constructive dismissal.
HELD:
Sorianos own allegation, although they are so detailed, appear incredible if
not downright puny. An analysis of her statements shows that her own
conclusion that she was being sexually and professionally harassed was on
the basis of her own suppositions, conjectures, and surmises.
She could not satisfactorily explain her allegation that she was consistently
professionally harassed by respondent Severino. The latters alleged words:
How come you claim you know so much yet nothing ever gets done in your
department? do not jurisprudentially constitute nor clearly establish
professional harassment. Aside from these words, the complainant could
only venture to allege instances in general and vague terms. As to the facts
allegedly constituting sexual harassment advanced by Go and Severino,
after an objective analysis over their assertions as stated in their respective
counter-affidavits and further considering the other supporting documents
attached to the respondents pleadings, it is found that these far out weigh
the Sorianos own evidence
A reading of the affidavit of the witness, who was never an employee nor
present at the party of Digitel, reveals, however, that she merely
concluded that the employees of Digitel were instructed or harassed not to
testify in favor of Soriano when they failed to meet one Matet Ruiz, a Digitel

employee who kept avoiding to meet with such tendency to threaten


resignation every time higher management would refuse her demand to
transfer subordinates who had administrative differences with her, we
therefore have no doubt that complainant voluntarily resigned when
respondent Severino refused to heed her demand that Ms. Arnedo and Ms.
Inductivo, her subordinates, be transferred to other departments. We also
have no doubt that such resignation does not constitute constructive
dismissal, much less an illegal one.

[G.R. No. 124617. April 28, 2000]


PHILIPPINE AEOLUS AUTOMOTIVE UNITED CORPORATION and/or
FRANCIS CHUA, petitioners, vs. NATIONAL LABOR RELATIONS
COMMISSION and ROSALINDA C. CORTEZ, respondents.
BELLOSILLO, J.:
FACTS:
Petitioner Philippine Aeolus Automotive United Corporation (PAAUC) is a
corporation duly organized and existing under Philippine laws, petitioner
Francis Chua is its President while private respondent Rosalinda C. Cortez
was a company nurse[1] of petitioner corporation until her termination on 7
November 1994. Jlexj

On 5 October 1994 a memorandum was issued by Ms. Myrna Palomares,


Personnel Manager of petitioner corporation, addressed to private
respondent Rosalinda C. Cortez requiring her to explain within forty-eight
(48) hours why no disciplinary action should be taken against her (a) for
throwing a stapler at Plant Manager William Chua, her superior, and uttering
invectives against him on 2 August 1994; (b) for losing the amount of
P1,488.00 entrusted to her by Plant Manager Chua to be given to Mr. Fang of
the CLMC Department on 23 August 1994; and, (c) for asking a co-employee
to punch-in her time card thus making it appear that she was in the office in
the morning of 6 September 1994 when in fact she was not. The
memorandum however was refused by private respondent although it was
read to her and discussed with her by a co-employee. She did not also
submit the required explanation, so that while her case was pending
investigation the company placed her under preventive suspension for thirty
(30) days effective 9 October 1994 to 7 November 1994. Lexjuris
On 20 October 1994, while Cortez was still under preventive suspension,
another memorandum was issued by petitioner corporation giving her
seventy-two (72) hours to explain why no disciplinary action should be taken
against her for allegedly failing to process the ATM applications of her nine
(9) co-employees with the Allied Banking Corporation. On 21 October 1994
private respondent also refused to receive the second memorandum
although it was read to her by a co-employee. A copy of the memorandum
was also sent by the Personnel Manager to private respondent at her last
known address by registered mail. Jurismis
Meanwhile, private respondent submitted a written explanation with respect
to the loss of the P1,488.00 and the punching-in of her time card by a coemployee.
On 3 November 1994 a third memorandum was issued to private
respondent, this time informing her of her termination from the service
effective 7 November 1994 on grounds of gross and habitual neglect of
duties, serious misconduct and fraud or willful breach of trust.
ISSUE:
Whether or not Rosalinda Cortez was sexually harassed?
RULING:

We are not persuaded. The gravamen of the offense in sexual harassment is


not the violation of the employee's sexuality but the abuse of power by the
employer. Any employee, male or female, may rightfully cry "foul" provided
the claim is well substantiated. Strictly speaking, there is no time period
within which he or she is expected to complain through the proper channels.
The time to do so may vary depending upon the needs, circumstances, and
more importantly, the emotional threshold of the employee. Esmso
Private respondent admittedly allowed four (4) years to pass before finally
coming out with her employer's sexual impositions. Not many women,
especially in this country, are made of the stuff that can endure the agony
and trauma of a public, even corporate, scandal. If petitioner corporation had
not issued the third memorandum that terminated the services of private
respondent, we could only speculate how much longer she would keep her
silence. Moreover, few persons are privileged indeed to transfer from one
employer to another. The dearth of quality employment has become a daily
"monster" roaming the streets that one may not be expected to give up
one's employment easily but to hang on to it, so to speak, by all tolerable
means. Perhaps, to private respondent's mind, for as long as she could
outwit her employer's ploys she would continue on her job and consider
them as mere occupational hazards. This uneasiness in her place of work
thrived in an atmosphere of tolerance for four (4) years, and one could only
imagine the prevailing anxiety and resentment, if not bitterness, that beset
her all that time. But William Chua faced reality soon enough. Since he had
no place in private respondent's heart, so must she have no place in his
office. So, he provoked her, harassed her, and finally dislodged her; and for
finally venting her pent-up anger for years, he "found" the perfect reason to
terminate her. Mse sm
In determining entitlement to moral and exemplary damages, we restate the
bases therefor. In moral damages, it suffices to prove that the claimant has
suffered anxiety, sleepless nights, besmirched reputation and social
humiliation by reason of the act complained of.[22] Exemplary damages, on
the other hand, are granted in addition to, inter alia, moral damages "by way
of example or correction for the public good"[23] if the employer "acted in a
wanton, fraudulent, reckless, oppressive or malevolent manner."

[A.M. No. CTA-01-1. April 2, 2002]


ATTY. SUSAN M. AQUINO, complainant, vs. HON. ERNESTO D.
ACOSTA, Presiding Judge, Court of Tax Appeals, respondent.
SANDOVAL-GUTIERREZ, J.:
Facts: On November 21, 2000, she reported for work after her vacation in the
U.S., bringing giftsfor the three judges of the CTA, including respondent. In
the afternoon of the same day, heentered her room and greeted her by
shaking her hand. Suddenly, he pulled her towards him andkissed her on her
cheek.On December 28, 2000, while respondent was on official leave, he
called complainant by phone,saying he will get something in her office.
Shortly thereafter, he entered her room, shook her hand and greeted her,
"Merry Christmas." Thereupon, he embraced her and kissed her. She wasable
to free herself by slightly pushing him away.On the first working day in

January, 2001, respondent phoned complainant, asking if she couldsee him


in his chambers in order to discuss some matters. When complainant arrived
there,respondent tried to kiss her but she was able to evade his sexual
attempt.Weeks later, after the Senate approved the proposed bill expanding
the jurisdiction of the CTA,while complainant and her companions were
congratulating and kissing each other, respondentsuddenly placed his arms
around her shoulders and kissed her.In the morning of February 14, 2001,
respondent called complainant, requesting her to go to hisoffice. She then
asked Ruby Lanuza, a clerk in the Records Section, to accompany
her.Fortunately, when they reached his chambers, respondent had left.The
last incident happened the next day. At around 8:30 a.m., respondent called
complainant andasked her to see him in his office to discuss the Senate bill
on the CTA. She again requestedRuby to accompany her. The latter agreed
but suggested that they should act as if they met byaccident in respondents
office. Ruby then approached the secretarys table which was separatedfrom
respondents office by a transparent glass. For her part, complainant sat in
front of respondent's table and asked him what he wanted to know about the
Senate bill. Respondentseemed to be at a loss for words and kept glancing at
Ruby who was searching for something atthe secretary's desk. Forthwith,
respondent approached Ruby, asked her what she was looking for and
stepped out of the office. When he returned, Ruby said she found what she
was looking for and left. Respondent then approached complainant saying,
me gusto akong gawin sa iyo kahapon pa. Thereupon, he tried to grab her.
Complainant instinctively raised her hands to protect herself but respondent
held her arms tightly, pulled her towards him and kissed her. She pushed
himaway, then slumped on a chair trembling. Meantime, respondent sat on
his chair and covered hisface with his hands. Thereafter, complainant left
crying and locked herself inside a comfortroom. After that incident,
respondent went to her office and tossed a note stating, sorry, it wonthappen
again.Issue: Whether or not Judge Acosta is guilty of sexually
harassment.Held: No, Judge Acosta is not guilty of sexual harassment. He is
exonerated of the chargesagainst him and is advised to be more circumspect
in his deportment. A mere casual buss on the cheek is not a sexual conduct
or favor and does not fallwithin the purview of sexual harassment under R.A.
No. 7877. Section 3 (a) thereof provides, towit:'Sec. 3.
Work, Education or Training - related Sexual Harassment Defined
. - Work, education or training-related sexual harassment is committed by an
employer, employee, manager, supervisor,agent of the employer, teacher,
instructor, professor, coach, trainor, or any other person who,having
authority, influence or moral ascendancy over another in a work or training

or educationenvironment, demands, requests or otherwise requires any


sexual favor from the other, regardlessof whether the demand, request or
requirement for submission is accepted by the object of saidAct.a)In a workrelated or employment environment, sexual harassment is committed
when:1)The sexual favor is made as a condition in the hiring or in the
employment, re-employment or continued employment of said individual, or
in granting said individualfavorable compensation, terms, conditions,
promotions or privileges; or the refusal togrant sexual favor results in
limiting, segregating or classifying the employee which inanyway would
discriminate,
deprive
or
diminish
employment
opportunities
or
otherwiseadversely affect said employees;2) The above acts would impair
the employee's right or privileges under existing labor laws;or 3) The above
acts would result in an intimidating, hostile, or offensive environment for
theemployee.'"Clearly, under the foregoing provisions, the elements of
sexual harassment are as follows:1)The employer, employee, manager,
supervisor, agent of the employer, teacher, instructor, professor, coach,
trainor, or any other person has authority, influence or moralascendancy
over another;2)The authority, influence or moral ascendancy exists in a
working environment;
3)
The employer, employee, manager, supervisor, agent of the employer,
teacher, instructor, professor, coach, or any other person having authority,
influence or moral ascendancymakes a demand, request or requirement of a
sexual favor.Indeed, from the records on hand, there is no showing that
respondent judge demanded,requested or required any sexual favor from
complainant in exchange for favorablecompensation, terms, conditions,
promotion or privileges specified under Section 3 of R.A. 7877.
G.R. No. 155831

February 18, 2008

MA. LOURDES T. DOMINGO, petitioner,


vs.
ROGELIO I. RAYALA, respondent.
NACHURA, J.:
FACTS:
On November 16, 1998, Ma. Lourdes T. Domingo (Domingo), then
Stenographic Reporter III at the NLRC, filed a Complaint for sexual

harassment against Rayala before Secretary Bienvenido Laguesma of the


Department of Labor and Employment (DOLE).The committee constituted
found Rayala guilty of the offense charged. Secretary Laguesma submitted a
copy of the Committee Report and Recommendation to the OP, but with the
recommendation that the penalty should be suspension for six (6) months
and one (1) day, in accordance with AO 250.On May 8, 2000, the OP issued
AO 119, disagreeing with there commendation that respondent be meted
only the penalty of suspension for six (6) months and one (1) day considering
the circumstances of the case because of the nature of the position of Reyala
as occupying the highest position in the NLRC, being its Chairman. Long
digest by Ernani Tadili.It was ordered that Rayala be dismissed from service
for being found guilty of grave offense of disgraceful and immoral conduct.
Rayala filed Motions for Reconsideration until the case was finally referred to
the Court of Appeals for appropriate action. The CA found Reyala guilty and
imposed the penalty of suspension of service for th emaximum period of one
(1) year. Domingo filed a Petition for Review before the SC. Rayala likewise
filed a Petition for Review with this Court essentially arguing that he is not
guilty of any act of sexual harassment. The Republic then filed its own
Petition for Review
ISSUE:
Whether or not there was sexual harassment?
RULING:
Rayala asserts that Domingo has failed to allege and establish any sexual
favor, demand, or request from petitioner in exchange for her continued
employment or for her promotion. According to Rayala, the acts imputed to
him are without malice or ulterior motive. It was merely Domingos
perception of malice in his alleged acts a "product of her own imagination"
that led her to file the sexual harassment complaint .Likewise, Rayala assails
the OPs interpretation, as upheld by the CA, that RA 7877 is
malum prohibitum such that the defense of absence of malice is unavailing.
He argues that sexual harassment is considered an offense against a
particular person, not against society as a whole. Rayala next argues that AO
250 expands the acts proscribed in RA7877. In particular, he assails the
definition of the forms of sexual harassment:
FORMS OFSEXUAL HARASSMENT
Section 1.

Forms of Sexual Harassment.


Sexual harassment may be committed in any of the following forms: a) Overt
sexual advances ;b) Unwelcome or improper gestures of affection; c) Request
or demand for sexual favors including but not limited to going out on dates,
outings or the like for the same purpose; d) Any other act or conduct of a
sexual nature or for purposes of sexual gratification which is generally
annoying, disgusting or offensive to the victim.
He posits that these acts alone without corresponding demand, request, or
requirement do not constitute sexual harassment as contemplated by the
law.
He alleges that the rule-making power granted to the employer in Section
4(a) of RA 7877 is limited only to procedural matters. The law did not
delegate to the employer the power to promulgate rules which would provide
other or additional forms of sexual harassment, or to come up with its own
definition of sexual harassment.

[G.R. No. 132164. October 19, 2004]


CIVIL SERVICE COMMISSION, petitioner, vs. ALLYSON BELAGAN,
respondent.
SANDOVAL-GUTIERREZ, J.:
FACTS:
2 separate complaints for sexual harassment and various malfeasances were
filed against Dr. Belagan, the Superintendent of DECS

1st (MAGDALENAs): She was applying for a permit to operate a pre


-school and during the inspection of the pre-school, Belagan placed his arms
around her shoulders and kissed her cheeks. When she followed up her
application, Belagan replied, Magdate muna tayo.
2nd (LIGAYA ANNAWI): She alleged in her complaint that on four separate
occasions, respondent touched her breasts, kissed her cheek, touched her
groins, embraced her from behind and pulled her close to him, his organ
pressing the lower part of her back.
DECS joint investigation: Belagan denied sexual harassment accusations.
Presented evidence against admin acts.
DECS Sec: GUILTY of 4 counts of sexual indignities or harassments
committed against Ligaya; and two (2)counts of sexual advances or
indignities against Magdalena; DISMISSED from service. Absolved of admin
malfeasance and dereliction of duty.
CSC: affirm DECS Sec but dismissed complaint of Ligaya. Transgression
against Magdalena constitutes grave misconduct.
CA: dismissed Magdalenas complaint, reversed CSC Resolutions. Magdalena
is an unreliable witness, her character being questionable. Magdalena was
previously charged with 22 offenses before MTC Baguio and 23 complaints
before brgy captains of Brgy Silang and Hillside in Baguio. Given her
aggressiveness and propensity for trouble, she is not one whom any male
would attempt to steal a kiss.

ISSUE1.
WON Magdalena is a credible witness
WON Belegan is guilty of grave misconduct.
HELD1.
YES. Rules on character evidence provision pertain only to criminal cases,
not to administrative offenses. Even if it is applicable to admin cases, only
character evidence that would establish the probability or improbability of
the offense charged may be proved. Character evidence must be limited to

the traits and characteristics involved in the type of offense charged. In this
case, no evidence bearing on Magdalenas chastity. What were presented
were charges for grave oral defamation, grave threats, unjust vexation,
physical injuries, malicious mischief, etc. filed against her.
Regarding Magdalenas credibility as a witness, the charges and complaints
against her happened way back in the70s and 80s while the act complained
of happened in 1994, thus, the said charges are no longer reliable proofs of
Magdalenas character or reputation. Evidence of ones character or
reputation must be confined to a time not too remote from the time in
question. In other words, what is to be determined is the character or
reputation of the person at the time of the trial and prior thereto, but not at a
period remote from the commencement of the suit.
It is unfair to presume that a person who has wandered from the path of
moral righteousness can never retrace his steps again. Certainly, every
person is capable to change or reform.
The general rule prevailing in a great majority of jurisdictions is that it is not
permissible to show that a witness has been arrested or that he has been
charged with or prosecuted for a criminal offense, or confined in jail for the
purpose of impairing his credibility. This view has usually been based upon
one or more of the following grounds or theories: (a) that a mere unproven
charge against the witness does not logically tend to affect his credibility,
(b)that innocent persons are often arrested or accused of a crime, (c) that
one accused of a crime is presumed to be innocent until his guilt is legally
established, and (d) that a witness may not be impeached or discredited by
evidence of particular acts of misconduct.

G.R. No. 165565

July 14, 2008

SCHOOL OF THE HOLY SPIRIT OF QUEZON CITY and/or SR. CRISPINA


A. TOLENTINO, S.Sp.S., Petitioners,
vs.
CORAZON P. TAGUIAM, Respondent.
QUISUMBING, J.:

Facts:
Corazon P. Taguiam was the Class Adviser of Grade 5-Esmeralda of the
petitioner, School of the Holy Spirit of Quezon City. On March 10, 2000, the
class president, wrote a letter to the grade school principal requesting
permission to hold a year-end celebration at the school grounds. The
principal authorized the activity and allowed the pupils to use the swimming
pool. In this connection, respondent distributed the parent's/guardian's
permit forms to the pupils. Corazon P. Taguiam admitted that Chiara Mae
Federico's permit form was unsigned.
Nevertheless, she concluded that Chiara Mae was allowed by her mother to
join the activity since her mother personally brought her to the school with
her packed lunch and swimsuit. Before the activity started, she warned the
pupils who did not know how to swim to avoid the deeper area. However,
while the pupils were swimming, two of them sneaked out. Respondent went
after them to verify where they were going.
Unfortunately, while respondent was away, Chiara Mae drowned. When she
returned, the maintenance man was already administering cardiopulmonary
resuscitation on Chiara Mae. The child was still alive when respondent rushed
her to the General Malvar Hospital where she was pronounced dead on
arrival. Corazon P. Taguiam was dismissed for gross negligence resulting to
loss of confidence.
Issue:
Was the dismissal based on the ground as stated valid?
Ruling:
Yes the dismissal was valid. Under Article 282 of the Labor Code, gross and
habitual neglect of duties is a valid ground for an employer to terminate an
employee. Gross negligence implies a want or absence of or a failure to
exercise slight care or diligence, or the entire absence of care. It evinces a
thoughtless disregard of consequences without exerting any effort to avoid
them. Habitual neglect implies repeated failure to perform one's duties for a
period of time, depending upon the circumstances. Respondent had been
grossly negligent. First , it is undisputed that Chiara Mae's permit form was
unsigned. Yet, respondent allowed her to join the activity because she
assumed that Chiara Mae's mother has allowed her to join it by personally

bringing her to the school with her packed lunch and swimsuit. The purpose
of a permit form is precisely to ensure that the parents have allowed their
child to join the school activity involved. Respondent cannot simply ignore
this by resorting to assumptions. Respondent admitted that she was around
when Chiara Mae and her mother arrived. She could have requested the
mother to sign the permit form before she left the school or at least called
her up to obtain her conformity. Second, it was respondent's responsibility as
Class Adviser to supervise her class in all activities sanctioned by the school.
Thus, she should have coordinated with the school to ensure that proper
safeguards, such as adequate first aid and sufficient adult personnel, were
present during their activity. She should have been mindful of the fact that
with the number of pupils involved, it would be impossible for her by herself
alone to keep an eye on each one of them. Notably, respondent's negligence,
although gross, was not habitual. In view of the considerable resultant
damage, however, the cause is sufficient to dismiss respondent.
This is not the first time that the SC have departed from the requirements
laid down by the law that neglect of duties must be both gross and habitual.
In Philippine Airlines, Inc. v. NLRC, we ruled that Philippine Airlines (PAL)
cannot be legally compelled to continue with the employment of a person
admittedly guilty of gross negligence in the performance of his duties
although it was his first offense. In that case, we noted that a
mere delay on PAL's flight schedule due to aircraft damage entails problems
like hotel accommodations for its passengers, re-booking, the possibility of
law suits, and payment of special landing fees not to mention the soaring
costs of replacing aircraft parts. In another case, Fuentes v. National Labor
Relations Commission, we held that it would be unfair to compel Philippine
Banking Corporation to continue employing its bank teller. In that case, we
observed that although the teller's infraction was not habitual, a substantial
amount of money was lost. The deposit slip had already been validated prior
to its loss and the amount reflected thereon is already considered as current
liabilities in the bank's balance sheet. Indeed, the sufficiency of the evidence
as well as the resultant damage to the employer should be considered in the
dismissal of the employee. In this case, the damage went as far as claiming
the life of a child. As a result of gross negligence in the present case,
petitioners lost its trust and confidence in respondent. Loss of trust and
confidence to be a valid ground for dismissal must be based on a willful
breach of trust and founded on clearly established facts. A breach is willful if
it is done intentionally, knowingly and purposely, without justifiable excuse,

as distinguished from an act done carelessly, thoughtlessly, heedlessly or


inadvertently.
Otherwise stated, it must rest on substantial grounds and not on the
employer's arbitrariness, whims, caprices or suspicion; otherwise, the
employee would eternally remain at the mercy of the employer. It should be
genuine and not simulated; nor should it appear as a mere afterthought to
justify earlier action taken in bad faith or a subterfuge for causes which are
improper, illegal or unjustified. It has never been intended to afford an
occasion for abuse because of its subjective nature.
There must, therefore, be an actual breach of duty committed by the
employee which must be established by substantial evidence. All told, there
being a clear showing that respondent was culpable for gross negligence
resulting to loss of trust and confidence, her dismissal was valid and legal.

G.R. No. 82471 February 18, 1991


PHILIPPINE AIRLINES, INC., petitioner,
vs.

NATIONAL LABOR RELATIONS COMMISSION, NATHANIEL PINUELA and


PHILIPPINE AIRLINES EMPLOYEES ASSOCIATION, respondents.
FERNAN, C.J.:p
FACTS:
Private respondent Nathaniel Pinuela was ground equipment and tug
operator of petitioner Philippine Air Lines (PAL), responsible for towing
aircraft from the hangar to the ramp area and vice-versa. In line with his
function, he held a professional driver's license and an operator's permit
issued by PAL after he had passed corresponding theoretical and practical
tests to operate PAL vehicles used for towing aircraft. He was already five (5)
years in this particular line of work when the incident in question arose.
On May 31, 1985, Pinuela was assigned as a member of the group tasked to
park and position the Boeing 747 aircraft registered as N-745 at the Manila
International Airport (now Ninoy Aquino International Airport, [NAIA]) for a
flight to the United States. The group members and their respective
assignments were: Pinuela tow tug operator, Rolando Manalaysay
leadman and wingtip guide; Rodrigo Camina-headsetman; Arturo Balagat
breakman, and a certain Taada guide. The head of the group in charge of
the parking/positioning of said aircraft was Nolie Domingo.
On or about 12:55 in the afternoon, the aircraft was towed from the PAL
technical center to Bay 16 area at the NAIA. While the Boeing 747 was being
towed, the airplane collided with the bridge at Bay 16 causing damage to the
plane's left landing light and the left wing flop and scratching its No. 2
engine. Consequently, on June 1, 1985, Pinuela was placed under preventive
suspension and was charged administratively. After investigation by the PAL
Administrative Board, he was dismissed from the service effective July 1,
1985.
Thereafter, Pinuela filed a complaint for illegal dismissal and unfair labor
practice against PAL. On July 21, 1987, the Labor Arbiter ruled in favor of
Pinuela and ordered PAL to reinstate him to his former position without loss
of seniority rights with full backwages until actual reinstatement. 1
On appeal, the National Labor Relations Commission (NLRC) on January 29,
1988 affirmed the decision of the Labor Arbiter but limited the award of

backwages to two (2) years only. 2 The Labor Tribunal opined that "Pinuela
could not be blamed for the accident as he relied on the signal of the
headsetman (Camina) who still signaled to him despite the fact that the nose
of the aircraft being towed was about to overshoot the yellow line and the
aircraft wing was about to hit the airbridge." 3
Hence, this recourse, the issue being whether or not the NLRC gravely
abused its discretion in appreciating the facts of the case. Petitioner PAL
contends that the records do not reveal that a tow operator can rely only on
a headsetman. According to the Engineering Manual of petitioner, a tug
operator must tipguide positive visual contact with the wing tipguide when
towing aircraft in congested areas. Thus, PAL avers that Pinuela should have
relied on the signal of Manalaysay, then wing tipguide, and not on Camina,
the headsetman.
ISSUE:
Whether or not the acts of Nathaniel Pinuela constitute gross and habitual
neglect of duties
RULING:
One must admit that towing an aircraft is a group activity necessitating
group coordination. This is explicit in petitioner's Engineering and
Maintenance Manual which states, "that the tug operator must undertake
and/or continue on towing/pushing procedure only when positive visual
contact with all guidemen is possible." The use of, "all necessary guidemen"
indicates plurality or group coordination. Thus, instead of relying solely on
the signals of Camina, Pinuela should have also checked with the other
ground crew personnel.
Particularly, Pinuela should have relied on Manalaysay, Exhibit 2 distinctly
shows the relative position of the plane, ground crew personnel and the
airport's aerobridge when the incident happened. Manalaysay, who was near
the marked line and the nearest obstruction which was the aerobridge and
the parked service vehicle, was strategically located. For Pinuela to claim
that he relied on Camina for signals is not credible, for he demonstrated
before the Labor Arbiter that he had to turn 180 degrees to see Camina who
was directly at his back. 9

In contradiction to the Labor Arbiter's opinion that petitioner's stance is weak


because PAL failed to tipguide normal towing speed, we say that there is no
necessity of determining the exact numerical towing speed. From daily
operations, we can safely assume that the ground crew knows what
"creeping" speed means. Through their working experience, they must
certainly have gained a sufficient degree of competence to determine
whether a Boeing 747, weighing tons, was being towed beyond normal speed
or not. Indeed, the Labor Arbiter acted with grave abuse of discretion in
ignoring this vital piece of evidence for the petitioner.
Pinuela's act of towing beyond normal speed, his failure to observe proper
parking procedure as provided in the Engineering and Maintenance Manual,
and the unanimous statement of the members of the towing crew that he
completely disregarded their warning shouts indicate that Pinuela is grossly
negligent of his responsibilities as a tug operator. Pinuela's dismissal must
therefore follow for a company has the right to dismiss its erring employees
if only as a measure of self-protection against acts inimical to its interest. 10
Philippine Airlines, as employer, cannot be legally compelled to continue with
the employment of a person admittedly guilty of gross negligence in the
performance of his duties. 11
Lastly, Pinuela should not compare the penalty of dismissal imposed on him
in relation to lesser sanctions previously meted by PAL on its other
employees. We are solely concerned here with the sufficiency of the
evidence surrounding Pinuela's dismissal. Besides, Pinuela's examples do not
involve a plane with a scheduled flight. A mere delay on petitioner's flight
schedule due to aircraft damage entails problems like hotel accommodations
for its passengers, re-booking, the possibility of law suits, and payment of
special landing fees not to mention the soaring costs of replacing aircraft.
parts. All told, Pinuela's gross negligence which called for dismissal is evident
and it was grave abuse of discretion on the part of the labor tribunal to have
ruled otherwise.

G.R. No. 75955 October 28, 1988

MARIA LINDA FUENTES, petitioner,


vs.
NATIONAL LABOR RELATIONS COMMISSION (NLRC), PHILIPPINE
BANKING CORPORATION and JOSE LAUREL IV, as its President,
respondents.
FERNAN, C.J.:
FACTS:
Petitioner was employed as a teller at the Philbanking's office at Ayala
Avenue, Makati, Metro Manila. On May 28, 1982, at about 10:30 a.m.,
petitioner, who was acting as an overnight teller, received a cash deposit of
P200,000.00. She counted the money with the assistance of a co-teller,
finishing the task at 10:40 a.m. or ten (10) minutes after her closing time.
Before she could start balancing her transactions, the Chief Teller handed her
several payroll checks for validation. Finding the checks to be incomplete,
petitioner left her cage to get other checks, without, however, bothering to
put the P200,000.00 cash on her counter inside her drawer. When she
returned to her cubicle after three (3) to five (5) minutes, she found that the
checks for validation were still lacking, so she went out of her cubicle again
to get the rest of the checks. On her way to a co-teller's cubicle, she noticed
that the P200,000.00 pile on her counter had been re-arranged. She thus
returned to her cage, counted the money and discovered that one (1) big
bundle worth P50,000.00 was missing therefrom. She immediately asked her
co-teller about it and getting a negative reply, she reported the matter to the
Chief Teller. A search for the P50,000.00 having proved unavailing, petitioner
was asked to explain why she should not be held liable for the loss. She
submitted her explanation on June 24, 1982.
Subsequently, on June 3, 1983, petitioner was dismissed for gross
negligence. On June 21, 1983, she filed a complaint for illegal dismissal with
reinstatement and backwages.
Private respondent bank seasonably filed an answer with counterclaim that
petitioner be ordered to restitute the amount of P50,000.
ISSUE:
Whether or not the dismissal of Fuentes on the ground of gross and habitual
neglect of duties valid?

RULING:
In the case at bar, the bank's inaction merely created a condition under
which the loss was sustained. Regardless of whether there was a failure to
investigate, the fact is that the money was lost in the first place due to
petitioner's gross negligence. Such gross negligence was the immediate and
determining factor in the loss.
Besides, the petitioner's position is anathema to banking operations. By
conducting an instant search on its depositors for every loss that occurs,
management holds suspect each depositor within its premises. Considering
that currency in the form of money bills bears no distinct earmarks which
would distinguish it from other similar bills of similar denominations except
as to its serial numbers, any innocent depositor with P50,000 in his
possession would be a likely suspect. Such act would do violence to the
fiduciary relationship between a bank and its depositors. Ultimately it will
result in the loss of valued depositors.
Petitioner argues further that the NLRC failed to consider that petitioner left
her cage at the instance of the Chief Teller. Again we are not persuaded. The
findings of the NLRC are clear. Petitioner left at her own volition to approach
her Chief Teller to ask for the remaining checks to ascertain their authenticity
and completeness. Besides, irrespective of who summoned her, her
responsibility over the cash entrusted to her remained.
Although petitioner's infraction was not habitual, we took into account the
substantial amount lost. Since the deposit slip for P200,000.00 had already
been validated prior to the loss, the act of depositing had already been
complete and from thereon, the bank had already assumed the deposit as a
liability to its depositors. Cash deposits are not assets to banks but are
recognized as current liabilities in its balance sheet.

G.R. No. 139847

March 5, 2004

PROCTER AND GAMBLE PHILIPPINES, petitioner,


vs.
EDGARDO BONDESTO, respondent.
TINGA, J.:
FACTS:
On July 18, 1975, respondent Edgardo Bondesto started work in the employ
of petitioner Procter and Gamble Philippines, Inc. Nineteen (19) years later,
the events which preceded the respondents dismissal from work unfolded.
At that time, he was working as production technician at the companys
Tondo Plant in Tondo, Manila.
On June 13, 1994, the respondent received a letter2 dated June 3, 1994,
asking him to explain why his absences consisting of 35 days3 should not be
classified as "unauthorized absence." Unauthorized absence, as a company
policy, is a ground for termination of employment.4
The respondent presented his explanation in two (2) separate letters5, both
dated June 16, 1994. However, on June 22, 1994, he received another letter,
this time informing him that his employment in the company was to be
terminated effective June 23, 1994 on the ground of "unauthorized
absences."
Claiming that his dismissal was without just cause, the respondent,
represented by the United Employees Union of Procter and Gamble Phils.,
Inc., filed a complaint for illegal dismissal before the National Labor Relations
Commission (NLRC). The respondent contended that his absences were
justified.
Sometime in November 1993, the respondent alleged, the petitioner directed
him to go to Mindanao for field assignment. Except for the plane fare which
the petitioner paid prior to his departure, the respondent advanced all the
other work-related expenses incurred during the assignment. One of the
petitioners Staff District Managers issued a check in the amount of Ten

Thousand Pesos (P10,000.00) supposedly to cover respondents traveling


expenses, but it bounced after he presented it to the bank.
On January 31, 1994, the respondent was re-assigned in Manila. He
immediately worked on the reimbursement of his advances. But as the
reimbursements were not immediately released, he was constrained to go to
the petitioners General Office located in Makati to follow-up the
reimbursement.
Meanwhile, the children of the respondent became sick. He spent time
attending to them. And as he needed money, he also went to the companys
Makati office to follow-up the reimbursement process. The delay in the
release of his reimbursement even forced him to apply for wage advances
under the collective bargaining agreement between the company and the
union.
On April 6, 1994, or after more than two months, the petitioner finally
released the respondents reimbursements.
One week later, or on April 13, 1994, the respondent received a letter asking
him to explain his "excessive absences"7, which according to him, included
the days he worked on his reimbursements. The respondent demurred. He
claimed that the seventeen (17) days should be considered as compensable
working time since he was then at the Makati office working on the
reimbursement of his money.
ISSUE:
Whether or not habitual absences constitute gross and habitual neglect of
duties
RULING
The Court agrees, however, with the petitioner that the respondent failed to
justify his prolonged absences during the months of May to June. While his
intention to go back to work was manifest, he regrettably failed to show that
he exerted any effort to locate his physician. Nevertheless, the failure to
locate the physician cannot amount to "serious misconduct or willful
disobedience," as the petitioner would like this Court to believe.
"Misconduct" has been defined as "the transgression of some established

and definite rule of action, a forbidden act, a dereliction of duty, willful in


character, and implies wrongful intent and not mere error in judgment."22
On the other hand, "willful disobedience" envisages the concurrence of at
least two (2) requisites: the employees assailed conduct has been willful or
intentional, the willfulness being characterized by a "wrongful and perverse
attitude;" and the order violated must have been reasonable, lawful, made
known to the employee and must pertain to the duties which he had been
engaged to discharge.
Even assuming that the respondents absenteeism constitutes willful
disobedience, such offense does not warrant the respondents dismissal. Not
every case of insubordination or willful disobedience by an employee
reasonably deserves the penalty of dismissal. There must be a reasonable
proportionality between the offense and the penalty.
At the time of the filing of the complaint, the respondent had worked with the
petitioner for nineteen (19) years. It has not been shown that the respondent
committed any infraction of company rules during his two (2) decade stint
in the company. Undoubtedly, dismissal is too harsh a sanction. Dismissal
has always been regarded as the ultimate penalty.
While the Court recognizes the rights of an employer to terminate the
services of an employee for a just or authorized cause, the dismissal of an
employee must be made within the parameters of law and pursuant to the
tenets of equity and fair play. Truly, the employers power to discipline its
workers may not be exercised in such an arbitrary manner as to erode the
constitutional guarantee of security of tenure.The Constitution mandates the
protection of labor. This command the Court has to heed and cannot
disregard.
In sum, the Court is convinced that the respondent has been illegally
terminated from employment. The normal consequences of illegal dismissal
are reinstatement without loss of seniority rights and the payment of back
wages computed from the time the employees compensation was withheld
from him. However, in view of the Courts finding that some of the
respondents absences were not wholly justified, the Court agrees with the
NLRC and the Court of Appeals that backwages should be limited to one (1)
year.
The petitioner claims that the existence of strained relationship between the
parties militates against the reinstatement of the respondent. While the

Court agrees that human nature engenders, in the normal course of things, a
certain degree of hostility as a result of litigation, the strained relations are
not necessarily sufficient to rule out reinstatement. As aptly put by the Court
of Appeals, "if petitioners contention should be sustained, reinstatement
would thus become the exception rather than the rule in cases of illegal
dismissal."

[G.R. No. 153477 : July 04, 2007]


DEL MONTE PHILIPPINES, INC. V. LOLITA VELASCO
Facts:
Respondent was a regular employee of petitioner, Del Monte Philippines. On
17 September 1994, a notice of hearing was sent to respondent notifying her
of the charges filed against her by the company, more specifically, for
violating the Absence Without Official Leave Rule. She was subsequently
dismissed. The respondent filed a case for illegal dismissal, asserting that her
dismissal was illegal because she was suffering from a urinary tract infection
related to her pregnancy and that her doctor had advised her to stay home
for a few days. She declared that she did not file the adequate leave of
absence because a medical certificate was already sufficient per company
policy. The Labor Arbiter dismissed the complaint. On appeal, both the NLRC
and the CA declared the dismissal of complainant as illegal for being contrary
to Art. 137 (2) of the Labor Code, prohibiting the discharge of a woman on
account of her pregnancy.

ISSUE:
Whether the absences of the respondent constitute gross and habitual
neglect of duties
Held:
The Court affirmed the ruling of the Court of Appeals. Respondents rule
penalizing with discharge any employee incurring six (6) or more absences
without permission or subsequent justification is admittedly within the

purview of the labor standard. However, while it is not disputed that


complainant incurred absences exceeding 6 days, respondent was able to
subsequently justify her absences in accordance with company rules and
policy: that she was pregnant at the time she incurred the absences; that the
fact of pregnancy and its related illnesses had been duly proven through
substantial evidence; that the respondent attempted to file leaves of
absence but the petitioners supervisor refused to received them; that she
could not have filed prior leaves due to her continuing condition, and that
petitioner dismissed the respondent on account of her pregnancy, a
prohibited act. Art. 137 of the Labor Code provides: It shall be unlawful for
any employer: x x x (2.) To discharge such woman on account of her
pregnancy, while on leave or in confinement due to her pregnancy; x x x.
WPP Marketing Communications Inc., et al., vs. Galera,
G.R. No. 169207, March 25, 2010
Facts:
Petitioner is Jocelyn Galera, an American citizen who was recruited from the
US by private respondent John Steedman, Chairman-WPP Worldwide and
Chief Executive Officer of Mindshare, Co., a corporation based in Hong Kong,
China, to work in the Philippines for private respondent WPP Marketing
Communications, Inc. (WPP). On December 14, 2000, GALERA alleged she
was verbally notified by private
STEEDMAN that her services had been terminated from private respondent
WPP. Atermination letter followed the next day. On 3 January 2001, Galera
filed a complaint for illegal dismissal, holiday pay, service incentive leave
pay, 13th month pay, incentive plan, actual and moral damages, and
attorney's fees against WPP and/or John Steedman (Steedman), Mark
Webster (Webster) and Nominada Lansang (Lansang). The Labor Arbiter's
Ruling for illegal dismissal and damages in favor of GALERA. The First
Division of the NLRC reversed the ruling of Arbiter Madriaga. Yet it was
reversed again by CA.
Issue:
Whether Galera is an Employee or a Corporate Officer.
Whether WPP illegally dismissed Galera.
Ruling:

Employee. Galera, on the belief that she is an employee, filed her complaint
before the Labor Arbiter. On the other hand, WPP, Steedman, Webster and
Lansang contend that Galera is a corporate officer; hence, any controversy
regarding her dismissal is under the jurisdiction of the Regional Trial Court.
We agree with Galera. Corporate officers are given such character either by
the Corporation Code or by the corporation's by-laws. Galera's appointment
as a corporate officer (Vice-President with the operational title of Managing
Director of Mindshare) during a special meeting of WPP's Board of Directors
is an appointment to a non-existent corporate office. At the time of Galera's
appointment, WPP already had one Vice-President in the person of Webster
and all five directorship positions provided in the by-laws are already
occupied. Another
indicator that she was a regular employee and not a corporate officer is
Section 14 of the contract, which clearly states that she is a permanent
employee not a Vice-President or a member of the Board of Directors.
Another convincing indication that she was only a regular employee and not
a corporate officer is the disciplinary procedure, which states that her right of
redress is through Mindshare's Chief Executive Officer for the Asia-Pacific.
This implies that she was not under the disciplinary control of private
respondent WPP's Board of Directors (BOD), which should have been the
case if in fact she was a corporate officer because only the Board of Directors
could appoint and terminate such a corporate officer.
WPP's dismissal of Galera lacked both substantive and procedural due
process. Apart from Steedman's letter dated 15 December 2000 to Galera,
WPP failed to prove any just or authorized cause for Galera's dismissal.
Steedman's letter to Galera reads: The operations are currently in a shamble.
There is lack of leadership and confidence in your abilities from within, our
agency partners and some clients. Most of the staff I spoke with felt they got
more guidance and direction from Minda than yourself. In your role as
Managing Director, that is just not acceptable. I believe your priorities are
mismanaged. The recent situation where you felt an internal strategy
meeting was more important than a new business pitch is a good example.
You failed to lead and advise on the two new business pitches. In both cases,
those involved sort (sic) Minda's input.
As I discussed with you back in July, my directive was for you to lead and
review all business pitches. It is obvious [that] confusion existed internally
right up until the day of the pitch. The quality output is still not to an
acceptable standard, which was also part of my directive that you needed to

focus on back in July. I do not believe you understand the basic skills and
industry knowledge required to run a media special operation.
WPP, Steedman, Webster, and Lansang, however, failed to substantiate the
allegations in Steedman's letter. Galera, on the other hand, presented
documentary evidence 22 in the form of congratulatory letters, including one
from Steedman, which contents are diametrically opposed to the 15
December 2000 letter. The law further requires that the employer must
furnish the worker sought to be dismissed with two written notices before
termination of employment can be legally effected: (1) notice which apprises
the employee of the particular acts or omissions for which his dismissal is
sought; and (2) the subsequent notice which informs the employee of the
employer's decision to dismiss him. Failure to comply with the requirements
taints the dismissal with illegality. 23 WPP's acts clearly show that Galera's
dismissal did not comply with the two-notice rule.

ARSENIO S. QUIAMBAO, vs.MANILA ELECTRIC COMPANY


G.R. No. 171023 December 18, 2009
FACTS:
Petitioner was employed as branch teller by respondent Manila Electric
Company-Mandaluyong Office for the handling and processing of payments.
Petitioner has repeatedly violated the Company Code of Employee Discipline
specifically excessive unauthorized absences. Through a Notice of Dismissal,
petitioners employment was terminated effective March 29, 2000.
On July 3, 2001, petitioner filed a complaint before the Arbitration Branch of
the NLRC against respondent assailing the legality of his dismissal. He also
claimed that he was denied due process. The Labor Arbiter dismissed the
complaint for a lack of merit.
Petitioner appealed to the NLRC which affirmed the legality of his dismissal
due to habitual absenteeism.
On appeal to the Court of Appeals, the CA nullified the NLRCs Decision and
reinstated the Labor Arbiters Decision dismissing the complaint. It ruled that
the award of separation pay is neither justified nor warranted under the
circumstances

The Motion for Reconsideration was denied, hence this petition for review on
certiorari.
ISSUE:
Whether or not a validly dismissed employee may be entitled to separation
pay.
RULING:
The Supreme Court DENIED petition for lack of merit.
The Labor Arbiter, the NLRC and the Court of Appeals found that petitioners
unauthorized absences and repeated infractions of company rules on
employee discipline manifest gross and habitual neglect of duty that merited
the imposition of the supreme penalty of dismissal from work. Serious
misconduct is said to be a transgression of some established and definite
rule of action, a forbidden act, a dereliction of duty, willful in character, and
indicative of wrongful intent and not mere error of judgment. Oddly,
petitioner never advanced any valid reason to justify his absences.
Following jurisprudence, it is held that a series of irregularities when put
together may constitute serious misconduct. Hence, the petitioner is not
entitled to separation pay. The liberality of the law can never be extended to
the unworthy and undeserving.

G.R. No. 112206

December 11, 1995

GONZALO D. LABUDAHON, KAPATIRANG ANAK-PAWIS SA PIONEER


TEXTURIZING, petitioners,
vs.
NATIONAL LABOR RELATIONS COMMISSION, PIONEER TEXTURIZING
CORPORATION, and MR. JULIANO LIM, respondents.
PADILLA, J.:

FACTS:

On 27 April 1990, petitioner applied for a 13-day paternity leave, but


respondent company allowed him only five (5) days of leave effective on 30
April 1990 and until 5 May 1990 because of lack of manpower at the
Texturizing Preparatory Section. From 7 May 1990 to 12 May 1990, petitioner
absented himself from work without respondent company's approval.
On 11 May 1990, Venus Dy, the personnel manager of respondent company,
wrote petitioner a letter directing him to report to work lest his absences be
considered as abandonment of duty.

On 12 May 1990, petitioner asked for two (2) weeks of vacation leave from 7
May to 20 May 1990, but the same was disapproved except for two (2) days
of leave on 14 and 15 of May 1990 to enable petitioner to attend to family
problems. Petitioner did not report for work from 16 to 19 of May 1990. Upon
orders of respondent company, petitioner submitted a written explanation
citing his wife's childbirth and family problems as reasons for his absences.
Petitioner was meted five (5) days suspension for unexcused absences and
for insubordination.
In spite of his previous absences, petitioner, as union officer, asked for fiftyfour (54) days of leave from 9 July 1990 to 31 August 1990 to prepare for
CBA negotiations and union activities. The request was denied and instead
the management advised petitioner to file his leave on a weekly basis, as
approval thereof was contingent on the necessity of his presence in the
operations of the Texturizing Department of respondent company. Petitioner
completely ignored this directive and absented himself from work starting 21
July 1990 until 16 August 1990. In a memorandum of the personnel
department dated 3 August 1990, petitioner was asked to submit a written
explanation for his absences. Respondent company never received any letter
of explanation. In a memorandum dated 28 August 1990, the company,
through its personnel manager, terminated petitioner's services to take
effect on 29 August 1990 "for excessive absences, insubordination, and
violation of existing company rules and regulations".

ISSUE:
Whether or not the excessive absences of the petitioner a valid ground for
his termination.
RULING:
The NLRC found that petitioner had no regard for his work. His applications
for a series of leaves of absence attest to his unconcern for his duties in
respondent company. On the other hand, respondent company has to protect
its interests in order to have an efficient and productive enterprise. It is in
this light that the law recognizes what are clearly "management
prerogatives", or the right of the employer to hire, fire, transfer, demote or
promote employees. Doubtless, what respondent did in this case was a
management prerogative. The need for petitioner's presence in the

company's Texturizing Department cannot be denied. Therefore, the


continuous and unauthorized absences of petitioner adversely affected the
operations of respondent company. The petitioner left the company with no
other choice but to terminate his employment.
The NLRC decision to indemnify petitioner is also affirmed, as there is no
evidence in the records to show that respondent company observed the twonotice requirement and hearing before dismissing petitioner. Applying the
Omnibus Rules Implementing the Labor Code 4 on the requirements of notice
and hearing, this Court in the case of Tiu vs. NLRC 5 ruled:
It is evident from the said provisions that the employer is required to furnish
an employee who is to be dismissed two (2) written notices before such
termination. The first is the notice to apprise the employee of the particular
acts or omissions for which his dismissal is sought. This may be loosely
considered as the proper charge. The second is the notice informing the
employee of the employer's decision to dismiss him. This decision, however,
must come only after the employee is given a reasonable period from receipt
of the first notice within which to answer the charge, and ample opportunity
to be heard and defend himself with the assistance of his representative, if
he so desires.
In the case at bar, petitioner was given only a letter of dismissal without
earlier informing him of the charges against him and without giving him the
opportunity to defend himself. Non-compliance by private respondent with
these requirements is a violation of the petitioner's right to due process.

BREW MASTER INTERNATIONAL INC. V NATIONAL FEDERATION OF


LABOR UNIONS (NAFLU)
DAVIDE, JR; April 17, 1997
FACTS
- Private respondent NAFLU, a co-complainant in the labor case, is a labor
union of which complainant is amember.- Complainant was first employed by
Brew Master on 16 September 1991 as route helper with the latest daily
wageof P119.00.- From 19 April 1993 up to 19 May 1993, for a period of 1
month, complainant went on absent without permission(AWOP).- On 20 May
1993, Brew master sent him a Memo: Please explain in writing within 24
hours of your receipt of thismemo why no disciplinary action should be taken
against you for the following offense: You were absent sinceApril 19, 1993 up
to May 19, 1993.- In answer to the aforesaid memo, complainant
explained:Sa dahilan po na ako ay hindi nakapagpaalam sainyo dahil inuwi
ko ang mga anak ko sa Samar dahil ang asawa koay lumayas at walang magaalaga sa mga anak ko. Kaya naman hindi ako naka long distance or
telegrama dahilwala akong pera at ibinili ko ng gamot ay puro utang pa.Finding said explanation unsatisfactory, the company issued a Notice of
Termination: ...we regret to inform youthat we do not consider it valid. You
are aware of the company Rules and Regulations that absence without
permission for 6 consecutive working days is considered abandonment of
work...- Complainants contend that individual complainants dismissal was
done without just cause; that it was notsufficiently established that individual
complainants absence from April 19, 1993 to June 16, 1993 are
unjustified;that the penalty of dismissal for such violation is too severe; that

in imposing such penalty, respondent should havetaken into consideration


complainants length of service and as a first offender, a penalty less
punitive will sufficesuch as suspension for a definite period.- Upon the other
hand, respondent contends that individual complainant was dismissed for
cause allowed by thecompany Rules and Regulations and the Labor Code;
that the act of complainant in absenting from work for 1month without
official leave is deleterious to the business of respondent; that it will result to
stoppage of productionwhich will not only destructive to respondents
interests but also to the interest of its employees in general; that
thedismissal of complainant from the service is legal.- The Labor Arbiter
dismissed the complaint for lack of merit, citing the principle of managerial
control, whichrecognizes the employers prerogative to prescribe reasonable
rules and regulations to govern the conduct of hisemployees. He relied on
Shoemart, Inc. vs. NLRC
: ...that individual complainant has indeed abandoned his work...therefore,
under the law and jurisprudence which upholds the right of an employer to
discharge an employee whoincurs frequent, prolonged and unexplained
absences as being grossly remiss in his duties to the employer and
istherefore, dismissed for cause. An employee is deemed to have abandoned
his position or to have resigned from thesame, whenever he has been absent
therefrom without previous permission of the employer for three
consecutivedays or more. - the NLRC modified the Labor Arbiter's decision
and held that complainants dismissal was invalid for thefollowing
reasons:Complainant-appellants
prolonged
absences,
although
unauthorized, may not amount to gross neglect or abandonment of work to
warrant outright termination of employment. Dismissal is too severe a
penalty...Relianceon the ruling enunciated in the cited case of Shoemart is
quite misplaced because of the obvious dissimilarities--complainant in the
Shoemart Case was an inveterate absentee who does not deserve
reinstatement compared toherein complainant-appellant who is a first
offender

ISSUE
WON the NLRC committed grave abuse of discretion in modifying the
decision of the Labor Arbiter

HELD

NO. Petitioners finding that complainant was guilty of abandonment is


misplaced. Abandonment as a just andvalid ground for dismissal requires the
deliberate, unjustified refusal of the employee to resume his
employment.Two elements must then be satisfied: (1) the failure to report for
work or absence without valid or justifiable reason;and (2) a clear intention
to sever the employer-employee relationship. b) Verily, relations between
capital and labor are not merely contractual. They are impressed with public
interestand labor contracts must, perforce, yield to the common good.While
the employer is not precluded from prescribing rules and regulations to
govern the conduct of his employees,these rules and their implementation
must be fair, just and reasonable.
Reasoning
- complainants absence was precipitated by a grave family problem as his
wife unexpectedly deserted him andabandoned the family. Considering that
he had a full-time job, there was no one to whom he could entrust
thechildren and he was thus compelled to bring them to the province. He
was then under emotional, psychological,spiritual and physical stress and
strain. The reason for his absence is, under these circumstances, justified.
Whilehis failure to inform and seek petitioner's approval was an omission
which must be corrected and chastised, he didnot merit the severest penalty
of dismissal from the service.- the elements of abandonment are not present
here. First, as held above, complainant's absence was justified under the
circumstances. As to the second requisite, complainant immediately
complied with the memo requiring him toexplain his absence, and upon
knowledge of his termination, immediately sued for illegal dismissal. These
plainlyrefuted any claim that he was no longer interested in returning to
work.- our Constitution looks with compassion on the workingman and
protects his rights not only under a generalstatement of a state policy, but
under the Article on Social Justice and Human Rights, thus placing labor
contracts ona higher plane and with greater safeguards.- While we do not
decide here the validity of petitioner's Rules and Regulations on continuous,
unauthorizedabsences, what is plain is that it was wielded with undue haste
resulting in a deprivation of due process, thus notallowing for a
determination of just cause or abandonment. In this light, petitioner's
dismissal was illegal. This isnot to say that his absence should go
unpunished, as impliedly noted by the NLRC in declining to award back
wages.

G.R. No. 168931 September 12, 2006


PAULINO ALITEN,
Petitioner,- versus U-NEED LUMBER & HARDWARE, and COURT OF APPEALS,
Respondents.
FACTS:
Petitioner, a native of Calanasan, Kalinga-Apayao, was employed on January
3, 1988[2] as helper in U-Need, private respondents business in Baguio City.
On October 22, 1990, petitioner was promoted as regular driver. Petitioner
was registered as a member of the Social Security System (SSS).
On April 30, 1992, petitioner asked permission from U-Need Manager Virginia
Tan for a 15-day leave of absence. He planned to go to his hometown to visit
his parents and vote for his uncle Elias Balot who was running for
congressman in the May 11, 1992 national and local elections. He needed a
15-day leave because his hometown was a remote area; and considering the
mountains and rivers he had to cross, travel could be very difficult and could
take roughly 12 days.[3] Petitioner signed a typewritten application for a
one-week vacation leave starting May 4, 1992,[4] a Monday, wherein he

declared that should he fail to report back for work at the end of his vacation,
it is understood that he would be automatically terminated by his employer.
He signed the typewritten application without reading its contents because
he was being scolded by Virginia Tan, who likewise forced him to sign.
Thereafter, he took some of his belongings from the lumbers bunkhouse and
left with his brother using a government vehicle.[5] When he left U-Need on
May 4, 1992, he had an outstanding account. However, it was not in the sum
of P1,750.00 as alleged by private respondent because as far as he knew,
amounts had been automatically deducted from his wages.[6]
It turned out that, as gleaned from the records of the Election Registrar,
Baguio City, he was a registered voter of Middle Rock Quarry, Baguio City,
not Calanasan.[7] He claimed that he only registered therein so that he
could cast his vote in Baguio should he fail to return to his domicile.[8] When
he reported for work on May 14, 1992, he was told that he had already been
dismissed from employment.

[G.R. No. 149180. February 14, 2005]


HODIENG CONCRETE PRODUCTS and/or HENRY GO and ERIC B. GO,
petitioners, vs. DANTE EMILIA, respondent.
SANDOVAL-GUTIERREZ, J.:
FACTS:
Respondent, in his complaint, alleged that sometime in January 1985, he
was employed as a truck helper by petitioner with a daily salary of P40.00.
Eventually, he became a regular truck driver with a salary of P76.00 per trip.
Feeling that he was underpaid, he asked petitioners secretary if he is
entitled to 13th month pay and other benefits. Upon receiving a negative
response, he reported the matter to the Department of Labor and
Employment. This prompted petitioners to terminate his services effective
January 2, 1997.
Petitioners denied respondents allegations in his complaint. They claimed
that sometime in 1987, respondent was employed as a truck helper with a
salary of P40.00 per trip. Later, in 1991, he recklessly drove and damaged

petitioners truck. As a consequence, he failed to report for work. But, in


1995, he was re-hired as a truck driver with a salary of P76.00 per trip. In
1997, he abandoned his work. Petitioners then came to know that he was
employed by Vortex, another company.
On March 30, 1998, the Labor Arbiter rendered a Decision holding that
respondent was illegally dismissed from employment and ordering
petitioners, jointly and severally, to pay his backwages and other benefits,
separation pay, and attorneys fee equivalent to 10% of the monetary
awards,
ISSUE:
Whether or not the dismissal was valid?
RULING:
On the alleged abandonment of employment by private respondent, the
contention is not meritorious. x x x. The burden of proof is on the employer
to show unequivocal intent on the part of the employee to discontinue
employment. Other than the self-serving declarations in the affidavits of
their two witnesses (pp. 59-61, ibid), petitioners failed to adduce other
evidence on any overt act of private respondent showing an actual intent to
abandon his employment. Moreover, private respondent filed an illegal
dismissal case against petitioners, an act which negates any intention on the
part of the employee to forsake his work. x x x.
Considering our finding that private respondent is a regular employee of
petitioners, and considering that there was no legal cause for private
respondents termination from employment, We so hold that petitioners
violated the constitutional right of private respondent to security of tenure
and due process. As a consequence of the illegal dismissal of the private
respondent, he is entitled under Articles 279 of the Labor Code, to his full
backwages, without deduction of earnings derived elsewhere from the time
his compensation was withheld from him up to the time of his actual
reinstatement. By virtue however of private respondents refusal to be
reinstated, the harmonious relationship between him and petitioners had
been severed such that reinstatement would no longer be beneficial to either
party in which case, a separation pay equivalent to one (1) month pay for
every year of service be paid. For failure of petitioners to submit the

corresponding employment records such as the payroll to controvert the


private respondents claim for service incentive leave pay and holiday pay,
We find the award for the said claim proper.

[G.R. No. 133259. February 10, 2000]


WENIFREDO FARROL, petitioner, vs. The HONORABLE COURT OF
APPEALS and RADIO COMUNICATIONS of the PHILIPPINES INC.
(RCPI), respondents.
YNARES_SANTIAGO, J.:
FACTS:
Petitioner Wenifredo Farrol was employed as station cashier at respondent
RCPIs Cotabato City station. On June 18, 1993, respondent RCPIs district
manager in Cotabato City informed their main office that "Peragram
funds"[1] from said branch were used for the payment of retirement benefits
of five employees. On October 1, 1993, petitioner verified as correct RCPIs
Field Auditors report that there was a shortage of P50,985.37 in their
branchs Peragram, Petty and General Cash Funds. Consequently, petitioner
was required by the Field Auditor to explain the cash shortage within 24
hours from notice.[2] The next day, petitioner paid to RCPI P25,000.00 of the
cash shortage.

On October 16, 1993, RCPI required petitioner to explain why he should not
be dismissed from employment.[3] Two days thereafter, petitioner wrote a
letter to the Field Auditor stating that the missing funds were used for the
payment of the retirement benefits earlier referred to by the branch manager
and that he had already paid P25,000.00 to RCPI. After making two more
payments of the cash shortage to RCPI, petitioner was informed by the
district manager that he is being placed under preventive suspension.[4]
Thereafter, he again paid two more sums on different dates to RCPI leaving a
balance of P6,995.37 of the shortage.
Respondent RCPI claims that it sent a letter to petitioner on November 22,
1993 informing him of the termination of his services as of November 20,
1993 due to the following reasons:
"a) Your allegation that part of your cash shortages was used for payment of
salaries/wages and retirement benefits is not true because these have been
accounted previously per auditors report;
"b) As Station Cashier you must be aware of our company Circular No. 63
which strictly requires the daily and up-to-date preparation of Statistical
Report and depositing of cash collections twice a day. But these procedures more particularly on depositing of cash collections twice a day - was
completely disregarded by you;
"c) Deliberate withholding of collections to hide shortages/malversation or
misappropriation in any form, as emphasized under Section No. 20 of our
Rules and Regulations, is penalized by immediate dismissal;
"d) The position of Station Cashier is one which requires utmost trust and
confidence.[5]
Unaware of the termination letter, petitioner requested that he be reinstated
considering that the period of his preventive suspension had expired.
Sometime in September 1995, petitioner manifested to RCPI his willingness
to settle his case provided he is given his retirement benefits. However, RCPI
informed petitioner that his employment had already been terminated earlier
as contained in the letter dated November 22, 1993. The conflict was
submitted to the grievance committee. Despite the lapse of more than two

years, the case remained unresolved before the grievance committee,


hence, it was submitted for voluntary arbitration.
ISSUE:
Whether or not the acts committed by Farrol constituted fraud or willful
breach of trust
RULING:
A perusal of RCPIs dismissal notice reveals that it merely stated a conclusion
to the effect that the withholding was deliberately done to hide alleged
malversation or misappropriation without, however, stating the facts and
circumstances in support thereof. It further mentioned that the position of
cashier requires utmost trust and confidence but failed to allege the breach
of trust on the part of petitioner and how the alleged breach was committed.
On the assumption that there was indeed a breach, there is no evidence that
petitioner was a managerial employee of respondent RCPI. It should be noted
that the term "trust and confidence" is restricted to managerial employees.
[15] It may not even be presumed that when there is a shortage, there is
also a corresponding breach of trust. Cash shortages in a cashiers work may
happen, and when there is no proof that the same was deliberately done for
a fraudulent or wrongful purpose, it cannot constitute breach of trust so as to
render the dismissal from work invalid.
Assuming further that there was breach of trust and confidence, it appears
that this is the first infraction committed by petitioner. Although the
employer has the prerogative to discipline or dismiss its employee, such
prerogative cannot be exercised wantonly, but must be controlled by
substantive due process and tempered by the fundamental policy of
protection to labor enshrined in the Constitution.[16] Infractions committed
by an employee should merit only the corresponding sanction demanded by
the circumstances. The penalty must be commensurate with the act, conduct
or omission imputed to the employee[17] and imposed in connection with
the employers disciplinary authority.
RCPI alleged that under its rules, petitioners infraction is punishable by
dismissal. However, employers rules cannot preclude the State from
inquiring whether the strict and rigid application or interpretation thereof
would be harsh to the employee. Petitioner has no previous record in his

twenty-four long years of service - this would have been his first offense. The
Court thus holds that the dismissal imposed on petitioner is unduly harsh and
grossly disproportionate to the infraction which led to the termination of his
services. A lighter penalty would have been more just, if not humane. In any
case, petitioner paid back the cash shortage in his accounts. Considering,
however, that the latter is about to retire or may have retired from work, it
would no longer be practical to order his reinstatement.

[G.R. No. 158758. April 29, 2005]


P.J. LHUILLIER INC. and PHILIPPE J. LHUILLIER, petitioners, vs.
NATIONAL LABOR RELATIONS COMMISSION and HERMINIA
MONTENEGRO, respondents.
CHICO-NAZARIO, J.:
FACTS:
Vincent Vicente Montenegro was the appraiser/manager of the petitioner
corporations Bauan Branch in Batangas. Lailanie Palma, a trainee of the
company, charged him with sexual harassment. A committee was formed to
investigate him for the alleged sexual harassment. On the basis of the
formal investigation conducted, the Chairman of the Investigation Committee
issued a Notice of Disciplinary Action dated 06 September 1997 wherein said
employee was meted a ten (10)-day suspension and transfer of assignment

to the CLH-Zobel Branch, Makati City, effective the next working day from
receipt thereof for violation of Section 9 of the Handbook on Company
Policies and Guidelines and Employees Code of Conduct, with a warning that
a repetition of said violation will be penalized with the supreme sanction of
dismissal. Vincent Montenegro claims that for the sexual harassment case,
he was meted 35 days suspension which he contends is a violation of the 30day suspension. Thereafter, he was transferred to Makati.
Herminia Montenegro was charged with dishonest acts committed by causing
the redemption of two (2) pieces of jewelry specifically described in pawn
tickets 008664 and 008665, allegedly, through the use of falsified affidavit of
loss. A formal administrative investigation was conducted on 15 October
1997. Findings of said investigation showed that respondent Herminia
Montenegro committed dishonesty and misconduct violative of Rule 22,
Section 2 of the Handbook on Company Policies, hence, she was dismissed
from employment. Herminia Montenegro averred, however, that her only
participation was the approval of the redemption of the pawned items by a
certain Agnes Moradas who submitted an affidavit of loss of pawnshop
tickets.
Carlos Pedro Sara was charged with incompetence and dishonesty. During
the administrative investigation conducted on 05 December 1997, the
investigating committee reported that Sara admitted having intentionally
overweighed an item in favor of a customer but the report about which he
refused to sign. It was also discovered that Sara was directly responsible for
the loss of certain jewelry as disclosed in an audit report.
Marites Noble was charged with having involved in the over-appraisal of an
item and having accepted a gold plated item. She claims that she had to
accept the over-appraised item to attract customers as the branch has just
opened. As for the fake item she accepted, Noble avers that the item is so
thickly plated that it could not be detected by merely applying the usual
procedure. During the formal investigation conducted on 05 December
1997, it was discovered and admitted by Noble that she intentionally overappraised the subject pawned fake item by increasing their true weights.
Later, it turned out that the fake items belong to Noble herself.
ISSUE:
Whether or not the termination of the respondents were with just cause?

RULING:
At the onset, it is pertinent to note that the second issue raised in the instant
petition inquires into the factual findings of the court a quo. The petitioners
are fundamentally raising a question of fact regarding the appellate courts
finding that the charge of falsification was not substantially proved. The
petitioner would have us sift through the evidence on record and pass upon
whether the signatures found on the Affidavit of Loss vis--vis the pawn
tickets are similar or not. This clearly involves a factual inquiry, the
determination of which is the statutory function of the NLRC.[20]
Elementary is the principle that this court is not a trier of facts. Judicial
review of labor cases does not go beyond the evaluation of the sufficiency of
the evidence upon which its labor officials findings rest.[21] As such, the
findings of facts and conclusion of the NLRC are generally accorded not only
great weight and respect but even clothed with finality and deemed binding
on this Court as long as they are supported by substantial evidence.[22] We
find no basis for deviating from the aforestated doctrine without any clear
showing that the findings of the labor arbiter, as affirmed by the NLRC and
the Court of Appeals, are bereft of sufficient substantiation. Well-settled is
the rule that the jurisdiction of this Court in a petition for review on certiorari
under Rule 45 of the Revised Rules of Court is limited to reviewing only errors
of law, not of fact, unless the factual findings complained of are completely
devoid of support from the evidence on record or the assailed judgment is
based on a gross misapprehension of facts.[23] What is more, factual
findings of quasi-judicial agencies like the NLRC, when affirmed by the Court
of Appeals, are conclusive upon the parties and binding on this Court.[24]
In the case at bar, the issue of the veracity of the signature appearing on the
questioned Affidavit of Loss has been undoubtedly passed upon by, not one,
not two, but three tribunals all having the same findings that there is no
evident showing that the said document is indeed falsified.
Be that as it may, we believe it proper to address and clarify petitioners
postulation that the Court of Appeals adopted the view that the degree of
proof required in Labor cases is proof beyond reasonable doubt instead of
merely substantial evidence.

Petitioners allege that they have lost trust and confidence in the respondent
due to the latters actions. The Labor Arbiter, however, found it hard to see
the basis of the loss of trust and confidence in the light of the insufficiency of
evidence presented by the petitioners, succinctly put thus:
. . . There is no showing that the affidavit of loss was a falsified one. Neither
is there any competent evidence submitted by the respondents to prove that
Mrs. Montenegro was the one who caused the execution thereof, granting
that the same is a falsified one. Henceforth, her dismissal from employment
based on the charge against her is illegal.[25]
The NLRC, affirming the aforequoted pronouncement of the Labor Arbiter,
added that:
The Affidavit of Loss not having been repudiated by the one who executed
the same, said affidavit stands and cannot be said to have been forged or a
fake one. Hence, we sustain the findings and ruling of the Labor Arbiter
relative to complainant Ms. Montenegro.

PHILIPPINE PIZZA, INC., formerly


G.R. No. 154315
PHILIPPINE FRANCHISE
PROGRESSIVE DEVELOPMENT
Present:
CORP., PIZZA HUT DIVISION,
and JANET RUTH M. SOLSOLOY,
Davide, Jr., C.J.,
Petitioners,
(Chairman),
Quisumbing,
Ynares-Santiago,
- versus Carpio, and
Azcuna, JJ.
KIM M. BUNGABONG,

Promulgated:
Respondent.
May 9, 2005

x--------------------------------------------------x
DECISION
QUISUMBING, J.:
FACTS:
Respondent Bungabong had been working for five years as a food attendant
in petitioners Ermita outlet. On December 6, 1997 at around 1:30 a.m., the
Duty Manager Alvin Biscocho, allegedly caught one Felix Sabado, another
employee, consuming some beer from the establishments beer dispenser.
While the duty manager did not actually see respondent, he concluded that
respondent was involved too, because earlier that night, a driver, Jonathan
Andra, reported that he saw respondent with Sabado drinking beer from the
dispenser. The next day, the duty manager called respondent, inquiring
about the latters involvement, and showed him a letter of Sabado admitting
to the offense of drinking beer, and then told him to file an incident report.
Thereafter, Criselda Cusi, the outlets unit manager, issued an offense notice.
Respondent denied any involvement in the theft of beer, asserting that only
Sabado was involved and was caught. Cusi reported the incident to the head
office of Pizza Hut.
On December 15, respondent was informed of his preventive suspension. He
was told to report to the Human Resources Department (HRD) of the
company for investigation. During the investigation, respondent stated,
driver Andra was with the Vice-President for HRD, co-petitioner Janet Ruth M.
Solsoloy. Andra then pointed to respondent, and stated that respondent was
with Sabado in drinking the companys beer on December 5, 1997, at around
11:30 to 12:00 p.m. A guard on duty, Rossman Manaloto, also stated that on
the evening of said date, he confronted respondent and asked why
respondent smelled of beer, but respondent ignored the inquiry and hurriedly
left. A crew member of the outlet, Daniel Gatdula, also reported on how the
respondent bragged how much beer he could drink on his way passing out of
the beer dispenser area.
After the investigation, a certain Ms. Ellen of the HRD explained to the
respondent the penalty for his alleged offense. She told respondent he
should no longer report for work. Respondent was advised to go home. He
then refused to receive his letter of termination, which followed after the
investigation.

ISSUE:
Whether or not respondent held a position entrusted with trust and
confidence?
RULING:
Contrary to the ruling of the Labor Arbiter and the NLRC, which eventually the Court of Appeals affirmed, we find that petitioner Philippine Pizza,
Inc. established the existence of just cause to terminate respondent on the
ground of loss of trust and confidence.
Where the employee has access to the employers property in the form
of merchandise and articles for sale, the relationship of the employer and the
employee necessarily involves trust and confidence.[24] Hence, when
respondent drank stolen beer from the dispenser of Pizza Hut-Ermita on
December 6, 1997, he gave cause for his termination and his termination
was within the ambit of Article 282 of the Labor Code.
Now, however, as regards violations of the procedural requirement for
valid dismissal, the petitioners could be justly faulted. Book V, Rule XIV of the
Omnibus Rules Implementing Batas Pambansa Blg. 130 in effect at the time
respondent was terminated, outlines the procedure for termination of employment.[25] It provides as follows:
Sec. 1. Security of tenure and due process. No worker shall be dis missed except for a just or authorized cause provided by law and after due
process.
Sec. 2. Notice of Dismissal. Any employer who seeks to dismiss a
worker shall furnish him a written notice stating the particular acts or
omissions constituting the grounds for his dismissal.
In cases of
abandonment of work, the notice shall be served at the workers last known
address.
. . .
Sec. 5. Answer and hearing. The worker may answer the allegations
stated against him in the notice of dismissal within a reasonable period from
receipt of such notice.
The employer shall afford the worker ample
opportunity to be heard and to defend himself with the assistance of his
representatives, if he so desires.
Sec. 6. Decision to dismiss. The employer shall immediately no tify a
worker in writing of a decision to dismiss him stating clearly the reasons
therefor.

In this case, we find that petitioners violated respondents right to due


process, particularly the requirement of first notice. The offense notice[26]
petitioners gave to respondent is insufficient first notice because it did not
comply with the requirement of the law that the first written notice must
apprise the employee that his termination is being considered due to the
acts stated in the notice.[27] The first notice issued in this case merely
stated that respondent is being charged of dispensing and drinking beer on
December 5, 1997, around 11:30 to 11:45 p.m.,[28] and nothing more.

HERMINIGILDO INGUILLO AND ZENAIDA BERGANTE,


Petitioners
- versus FIRST PHILIPPINE SCALES, INC. and/or AMPARO POLICARPIO,
MANAGER,
Respondents.
G.R. No. 165407

FACTS:
First Philippine Scales, Inc. (FPSI), a domestic corporation engaged in the
manufacturing of weighing scales, employed Bergante and Inguillo as
assemblers on August 15, 1977 and September 10, 1986, respectively.
In 1991, FPSI and First Philippine Scales Industries Labor Union (FPSILU)
[3] entered into a Collective Bargaining Agreement (CBA),[4] the duration of
which was for a period of five (5) years starting on September 12, 1991 until
September 12, 1996.
On September 19, 1991, the members of FPSILU
ratified the CBA in a document entitled RATIPIKASYON NG KASUNDUAN.[5]
Bergante and Inguillo, who were members of FPSILU, signed the said
document.[6]
During the lifetime of the CBA, Bergante, Inguillo and several FPSI
employees joined another union, the Nagkakaisang Lakas ng Manggagawa
(NLM), which was affiliated with a federation called KATIPUNAN (NLMKATIPUNAN, for brevity).
Subsequently, NLM-KATIPUNAN filed with the
Department of Labor and Employment (DOLE) an intra-union dispute[7]
against FPSILU and FPSI. In said case, the Med-Arbiter decided[8] in favor of
FPSILU.
It also ordered the officers and members of NLM-KATIPUNAN to
return to FPSILU the amount of P90,000.00 pertaining to the union dues
erroneously collected from the employees. Upon finality of the Med-Arbiter's
Decision, a Writ of Execution[9] was issued to collect the adjudged amount
from NLM-KATIPUNAN. However, as no amount was recovered, notices of
garnishment were issued to United Coconut Planters Bank (Kalookan City
Branch)[10] and to FPSI[11] for the latter to hold for FPSILU the earnings of
Domingo Grutas, Jr. (Grutas) and Inguillo, formerly FPSILU's President and
Secretary for Finance, respectively, to the extent of P13,032.18. Resultantly,
the amount of P5,140.55 was collected,[12] P1,695.72 of which came from
the salary of Grutas, while the P3,444.83 came from that of Inguillo.
Meanwhile, on March 29, 1996, the executive board and members of
the FPSILU addressed a document dated March 18, 1996 denominated as
Petisyon[13] to FPSI's general manager, Amparo Policarpio (Policarpio),
seeking the termination of the services of the following employees, namely:
Grutas, Yolanda Tapang, Shirley Tapang, Gerry Trinidad, Gilbert Lucero,

Inguillo, Bergante, and Vicente Go, on the following grounds:[14] (1)


disloyalty to the Union by separating from it and affiliating with a rival Union,
the NLM-KATIPUNAN; (2) dereliction of duty by failing to call periodic
membership meetings and to give financial reports; (3) depositing Union
funds in the names of Grutas and former Vice-President Yolanda Tapang,
instead of in the name of FPSILU, care of the President; (4) causing damage
to FPSI by deliberately slowing down production, preventing the Union to
even attempt to ask for an increase in benefits from the former; and (5)
poisoning the minds of the rest of the members of the Union so that they
would be enticed to join the rival union.
ISSUE:
Whether or not the security union clause was valid
RULING:
In terminating the employment of an employee by enforcing the Union
Security Clause, the employer needs only to determine and prove that: (1)
the union security clause is applicable; (2) the union is requesting for the
enforcement of the union security provision in the CBA; and (3) there is
sufficient evidence to support the union's decision to expel the employee
from the union or company.[43]
We hold that all the requisites have been sufficiently met and FPSI was
justified in enforcing the Union Security Clause, for the following reasons:
First. FPSI was justified in applying the Union Security Clause, as it
was a valid provision in the CBA, the existence and validity of which was not
questioned by either party.
Moreover, petitioners were among the 93
employees who affixed their signatures to the document that ratified the
CBA.
They cannot now turn their back and deny knowledge of such
provision.
Second. FPSILU acted on its prerogative to recommend to FPSI the
dismissal of the members who failed to maintain their membership with the
Union. Aside from joining another rival union, FPSILU cited other grounds
committed by petitioners and the other employees which tend to prejudice
FPSIs interests, i.e., dereliction of duty - by failing to call periodic
membership meetings and to give financial reports; depositing union funds in

the names of Grutas and former Vice-President Yolanda Tapang, instead of in


the name of FPSILU care of the President; causing damage to FPSI by
deliberately slowing down production, preventing the Union from even
attempting to ask for an increase in benefits from the former; and poisoning
the minds of the rest of the members of the Union so that they would be
enticed to join the rival union.
Third. FPSILU's decision to ask for the termination of the employees in
the Petisyon was justified and supported by the evidence on record.
Bergante and Inguillo were undisputably former members of FPSILU. In fact,
Inguillo was the Secretary of Finance, the underlying reason why his salary
was garnished to satisfy the judgment of the Med-Arbiter who ordered NLMKATIPUNAN to return the Union dues it erroneously collected from the
employees. Their then affiliation with FPSILU was also clearly shown by their
signatures in the document which ratified the CBA. Without a doubt, they
committed acts of disloyalty to the Union when they failed not only to
maintain their membership but also disaffiliated from it. They abandoned
FPSILU and even joined another union which works against the former's
interests.
This is evident from the intra-union dispute filed by NLMKATIPUNAN against FPSILU.
Once affiliated with NLM-KATIPUNAN, Bergante
and Inguillo proceeded to recruit other employees to disaffiliate from FPSILU
and even collected Union dues from them.
\

G.R. No. 75037 April 30, 1987


TANDUAY DISTILLERY LABOR UNION VS NLRC
FACTS:
Private respondents were all employees of Tanduay Distillery, Inc., (TDI) and
members of the Tanduay Distillery Labor Union (TDLU), a duly organized and

registered labor organization and the exclusive bargaining agent of the rank
and file employees of the petitioner company.
A Collective Bargaining Agreement (CBA), was executed between TDI and
TDLU. The CBA was duly ratified by a majority of the workers in TDI including
herein private respondents and contained a union security clause which
provides that all workers who are or may during the effectivity of the CBA,
become members of the Union in accordance with its Constitution and ByLaws shall, as a condition of their continued employment, maintain
membership in good standing in the Union for the duration of the
agreement.
While the CBA was in effect and within the contract bar period the private
respondents joined another union, the Kaisahan Ng Manggagawang Pilipino
(KAMPIL) and organized its local chapter in TDI. KAMPIL filed a petition for
certification election to determine union representation in TDI, which
development compelled TDI to file a grievance with TDLU.
TDLU created a committee to investigate its erring members in accordance
with its by-laws which are not disputed by the private respondents.
Thereafter, TDLU, through the Investigating Committee and approved by
TDLU's Board of Directors, expelled the private respondents from TDLU for
disloyalty to the Union. By letter, TDLU notified TDI that private respondents
had been expelled from TDLU and demanded that TDI terminate the
employment of private, respondents because they had lost their membership
with TDLU.
The private respondents were later on terminated. In their petition, private
respondents contend that their act of organizing a local chapter of KAMPIL
and eventual filing of a petition for certification election was pursuant to
their constitutional right to self-organization.
ISSUES:
a) whether or not TDI was justified in terminating private respondents'
employment in the company on the basis of TDLU's demand for the
enforcement of the Union Security Clause of the CBA between TDI and TDLU;
and
b) whether or not TDI is guilty of unfair labor practice in complying with
TDLU's demand for the dismissal of private respondents.
HELD:

The dismissal of an employee pursuant to a demand of the majority union in


accordance with a union security agreement following the loss of seniority
rights is valid and privileged and does not constitute an unfair labor practice.
Article 249 (e) of the Labor Code as amended specifically recognizes the
closed shop arrangement as a form of union security. The closed shop, the
union shop, the maintenance of membership shop, the preferential shop, the
maintenance of treasury shop, and check-off provisions are valid forms of
union security and strength. They do not constitute unfair labor practice nor
are they violations of the freedom of association clause of the Constitution.
There is no showing in these petitions of any arbitrariness or a violation of
the safeguards enunciated in the decisions of this Court interpreting union
security arrangements brought to us for review.

[G.R. No. 146621. July 30, 2004]


RENE P. VALIAO, petitioner, vs. HON. COURT OF APPEALS, NATIONAL
LABOR RELATIONS COMMISSION-FOURTH DIVISION (Cebu City),
WEST NEGROS COLLEGE, respondents.
FACTS:

On February 5, 1990, petitioner Rene Valiao was appointed by private


respondent West Negros College (WNC) as Student Affairs Office (SAO)
Director, with a starting salary of P2,800 per month. On May 14, 1990, he
was assigned as Acting Director, Alumni Affairs Office.
On July 29, 1990, petitioner was transferred to a staff position and designated
as Records Chief at the Registrars Office but was again re-assigned as a
typist on June 24, 1991.
The latest re-assignment was due to his tardiness and absences, as reflected
in the summary of tardiness and absences report, which showed him to have
been absent or late for work from a minimum of seven (7) to a maximum of
seventy-five (75) minutes for the period March to October 31, 1991, and to
have reported late almost every day for the period November to December
1991.
Copies of his tardiness/absences reports were furnished petitioner, along with
memoranda requiring him to explain but his explanations were either
unacceptable or unsatisfactory. Subsequent reports also showed that he did
not change his habits resulting in tardiness and absences. He was even
caught one time manipulating the bundy clock, thus necessitating another
memorandum to him asking him to explain his dishonest actuations in
accomplishing the daily attendance logbook and in using the bundy clock.
On December 10, 1991, petitioner received a suspension order without pay
for fifteen (15) days effective January 1, 1992, because of dishonesty in
reporting his actual attendance. After serving the suspension, the petitioner
reported back to office on January 16, 1992.
On June 15, 1992, another adverse report on tardiness and absences from
the Registrar was made against the petitioner prompting WNC to send him
another memorandum with an attached tardiness and absences report,
calling his attention on his tardiness and absences for the period February to
April 1992.
On June 20, 1992, petitioner sent a letter of appeal and explained his side to
the new college president, Suzette Arbolario-Agustin, who gave petitioner
another chance. The petitioner was then appointed as Information Assistant
effective immediately. However, the petitioner did not immediately assume
the post of Information Assistant prompting the President of private

respondent WNC to call his attention. When the petitioner finally assumed
his post, he was allowed a part-time teaching job in the same school to
augment his income.
Sometime in December 1992, WNC won a case against the officials of the
union before the NLRC. Petitioner was ordered to prepare a media blitz of
this victory but the petitioner did not comply with the order on the ground
that such a press release would only worsen the already aggravated situation
and strained relations between WNC management and the union officials.
When petitioner reported for work on the first day of January 1993, he was
relieved from his post and transferred to the College of Liberal Arts as
Records Evaluator. Not for long, the Dean of the Liberal Arts sent a letter to
the Human Resources Manager complaining about the petitioners poor
performance and habitual absenteeism, as shown in the daily absence
reports.
ISSUE:
Whether or not the acts of the petitioner constitute gross and habitual
neglect of duties
RULING
Considering the submissions of the parties as well as the records before us,
we find the petition without merit. Petitioners dismissal from employment is
valid and justified.
For an employees dismissal to be valid, (a) the dismissal must be for a valid
cause and (b) the employee must be afforded due process.[7]
Serious misconduct and habitual neglect of duties are among the just causes
for terminating an employee under the Labor Code of the Philippines. Gross
negligence connotes want of care in the performance of ones duties.
Habitual neglect implies repeated failure to perform ones duties for a period
of time, depending upon the circumstances.[8] The Labor Arbiters findings
that petitioners habitual absenteeism and tardiness constitute gross and
habitual neglect of duties that justified his termination of employment are
sufficiently supported by evidence on record. Petitioners repeated acts of
absences without leave and his frequent tardiness reflect his indifferent

attitude to and lack of motivation in his work. More importantly, his repeated
and habitual infractions, committed despite several warnings, constitute
gross misconduct unexpected from an employee of petitioners stature. This
Court has held that habitual absenteeism without leave constitute gross
negligence and is sufficient to justify termination of an employee.[9]
However, petitioner claims that he was dismissed not for his tardiness or
absences but for his arrest as a suspected drug user. His claim, however, is
merely speculative. We find such contention devoid of basis. First, the
decisions of the Labor Arbiter, the NLRC, and the Court of Appeals are
indubitable. They show that indeed petitioner had incurred numerous and
repeated absences without any leave. Moreover, he was not punctual in
reporting for work.
These unexplained absences and tardiness were
reflected on the summary reports submitted by WNC before the labor arbiter,
but petitioner failed to controvert said reports.
Second, contrary to
petitioners assertion, the NLRC did not base its conclusions on the fact of
the arrest of petitioner for violation of Rep. Act No. 6425 but on the totality of
the number of infractions incurred by the petitioner during the period of his
employment in different positions he occupied at WNC. Thus:
In the case of petitioner Valiao, his services were terminated by private
respondent after having been found guilty of serious misconduct and gross
habitual neglect of duty which was aggravated by the January 28, 1993
incident. In exercising such management prerogative, due process was
properly observed. Private respondent presented sufficient evidence to
support its act in terminating the services of petitioner. Private respondent
took into consideration the totality of the infractions or the number of
violations committed by petitioner during the period of employment.
Furthermore, it hardly needs reminding that, in view of petitioners position
and responsibilities, he must demonstrate a scrupulous regard for rules and
policies befitting those who would be role models for their young charges.
[10] (Emphasis and italics supplied)
Indeed, even without the arrest incident, WNC had more than enough basis
for terminating petitioner from employment.
It bears stressing that
petitioners absences and tardiness were not isolated incidents but
manifested a pattern of habituality. In one case, we held that where the
records clearly show that the employee has not only been charged with the
offense of highgrading but also has been warned 21 times for absences
without official leave, these repeated acts of misconduct and willful breach of

trust by an employee justify his dismissal and forfeiture of his right to


security of tenure. The totality of infractions or the number of violations
committed during the period of employment shall be considered in
determining the penalty to be imposed upon an erring employee. The
offenses committed by him should not be taken singly and separately but in
their
totality.
Fitness
for
continued
employment
cannot
be
compartmentalized into tight little cubicles of aspects of character, conduct,
and ability separate and independent of each other.
Needless to say, so irresponsible an employee like petitioner does not
deserve a place in the workplace, and it is within the managements
prerogative of WNC to terminate his employment. Even as the law is
solicitous of the welfare of employees, it must also protect the rights of an
employer to exercise what are clearly management prerogatives. As long as
the companys exercise of those rights and prerogative is in good faith to
advance its interest and not for the purpose of defeating or circumventing
the rights of employees under the laws or valid agreements, such exercise
will be upheld.

Alvarez vs. Golden Tri Bloc Inc., G.R. No. 202158, September 25,
2013
FACTS:

Sometime in November 1996, respondent GTBI hired the petitioner as a


Service Crew in one of its Dunkin Donuts franchise store in Antipolo City,
Rizal. Six (6) months later, he attained the status of a regular employee. He
was thereafter promoted as Shift Leader and served as such for four (4)
years. Sometime in 2001, he was again promoted as Outlet Supervisor and
was assigned to three (3) Dunkin Donuts outlets located at San Roque,
Cogeo and Super 8, Masinag, all in Antipolo City. He received a monthly
salary of P10,000.00.
On May 27, 2009, the petitioner reported for duty at around 12:30 in the
afternoon at Dunkin Donuts, Super 8, Masinag branch. Since his time card
was at the San Roque branch, he telephoned Chastine3 Kaye Sambo
(Sambo), shift leader, and requested her to punch-in his time card to
reflect that he is already on duty. She obliged. Roland Salindog (Salindog),
the petitioners senior officer called the Super 8, Masinag branch and verified
that he has indeed reported for work.
The following day, however, the petitioner was informed by Sambo that both
of them are suspended and that he had to prepare an incident report
regarding his time card.
In his incident report4 dated May 29, 2009, the petitioner admitted
instructing Sambo to punch-in his timecard. He explained that he went
straight to and arrived at the Super 8, Masinag branch at around 12:35 p.m.
He inspected the stocks in the branch and taught a certain Ritz on how to
prepare stocks acquisition report for June 2009. He owned up to his fault and
stated that he should have instead recorded the time of his arrival by writing
on the time card and that he should have brought it with him. He apologized
and promised that a similar incident will not happen again.
On June 5, 2009, GTBI sent him a letter directing him to report to the main
office for a dialogue on June 9, 2009 failing which would amount to the
waiver of his right to be heard and the management may make a decision
based only on his written explanation.5 The dialogue pushed through. After
which the petitioner was placed on preventive suspension for 30 days
without pay.
On June 23, 2009, GTBI notified the petitioner of its decision to terminate his
employment effective that day on the ground of loss of trust.

ISSUE:
Whether or not the termination on the ground of loss was valid?
RULING:
"In Merin v. NLRC, the Court ruled that in determining the sanction imposable
to an employee, the employee may consider and weigh his other past
infractions, thus:
"'The totality of infractions or the number of violations committed during the
period of employment shall be considered in determining the penalty to be
imposed upon an erring employee. The offenses committed by petitioner
should not be taken singly and separately. Fitness for continued employment
cannot be compartmentalized into tight little cubicles of aspects of
character, conduct and ability separate and independent of each other. While
it may be true that petitioner was penalized for his previous infractions, this
does not and should not mean that his employment record would be wiped
clean of his infractions. After all, the record of an employee is a relevant
consideration in determining the penalty that should be meted out since an
employee's past misconduct and present behavior must be taken together in
determining the proper imposable penalty. Despite the sanctions imposed
upon petitioner, he continued to commit misconduct and exhibit undesirable
behavior on board. Indeed, the employer cannot be compelled to retain a
misbehaving employee, or one who is guilty of acts inimical to its interests. It
has the right to dismiss such an employee if only as a measure of selfprotection. (Citations omitted)'"
The NLRC and the CA were thus correct in applying the totality of infractions
rule and in adjudging that the petitioner's dismissal was grounded on a just
and valid cause."

Yrasuegui vs. Phil Airlines, G.R. No. 168081, Oct. 17, 2008

Facts:
Armando G. Yrasuegui was a former international flight steward of Philippine
Airlines, Inc. (PAL). He stands five feet and eight inches (58) with a large
body frame. The proper weight for a man of his height and body structure is
from 147 to 166 pounds, the ideal weight being 166 pounds, as mandated by
the Cabin and Crew Administration Manual of PAL. His weight problem dates
back to 1984 when PAL advised him to go on an extended vacation leave
from December 29, 1984 to March 4, 1985 to address his weight concerns.
For failure to meet the weight standards another leave without pay from
March 5, 1985 to November 1985 was imposed. He met the required weight
and was allowed to work but his weight problem recurred, thus another leave
without pay from October 17, 1988 to February 1989. From 1989 to 1992 his
weight fluctuated from 209lb, 215lb, 217lb, 212lb, and 205. During that
period he was requested to lose weight and to report for weight checks
which he constantly failed to do. In the meantime his status was off-duty.
Finally in 1993, petitioner was formally informed by PAL that due to his
inability to attain his ideal weight, and considering the utmost leniency
extended to him which spanned a period covering a total of almost five (5)
years, his services were considered terminated effective immediately. He
then filed a complaint for illegal dismissal against PAL. The Labor Arbiter
ruled that he was illegally dismissed and entitles to reinstatement,
backwages and attorneys fees. The NLRC affirmed the LA. The CA reversed
the NLRC.
Issue:
Whether or not petitioner was illegally dismissed.
Ruling:
The obesity of petitioner is a ground for dismissal under Article 282(e) of the
Labor Code. The weight standards of PAL constitute a continuing qualification
of an employee in order to keep the job. Tersely put, an employee may be
dismissed the moment he is unable to comply with his ideal weight as
prescribed by the weight standards. The dismissal would fall under Article
282(e) of the Labor Code. As explained by the CA:
x x x [T]he standards violated in this case were not mere orders of the
employer; they were the prescribed weights that a cabin crew must

maintain in order to qualify for and keep his or her position in the company.
In other words, they were standards that establish continuing qualifications
for an employees position. The failure to meet the employers qualifying
standards is in fact a ground that does not squarely fall under grounds (a) to
(d) and is therefore one that falls under Article 282(e) the other causes
analogous to the foregoing.
By its nature, these qualifying standards are norms that apply prior to and
after an employee is hired. x x x
We hold that the obesity of petitioner, when placed in the context of his work
as flight attendant, becomes an analogous cause under Article 282(e) of the
Labor Code that justifies his dismissal from the service. His obesity may not
be unintended, but is nonetheless voluntary.

John Hancock Life Insurance Corp. vs. Davis, G.R. No. 169549, Sept.
3, 2008
Facts:

Joanna Cantre Davis was agency administration officer of John Hancock Life
Insurance Corporation. On October 18, 2000, Patricia Yuseco, JHLICs
corporate affairs manager, discovered that her wallet was missing. She
immediately reported the loss of her credit cards to AIG and BPI Express. To
her surprise, she was informed that "Patricia Yuseco" had just made
substantial purchases using her credit cards in various stores in the City of
Manila. She was also told that a proposed transaction in Abenson's-Robinsons
Place was disapproved because "she" gave the wrong information upon
verification. Because loss of personal property among its employees had
become rampant in its office, petitioner sought the assistance of NBI. The
NBI, obtained a security video from Abenson's showing the person who used
Yuseco's credit cards. Yuseco and other witnesses positively identified the
person in the video as Davis NBI and Yuseco filed a complaint for qualified
theft against Davis but because the affidavits presented by the NBI
(identifying respondent as the culprit) were not properly verified, the city
prosecutor dismissed the complaint due to insufficiency of evidence.
Meanwhile, petitioner placed Davis under preventive suspension and
instructed her to cooperate with its ongoing investigation. Davis filed a
complaint for illegal dismissal alleging that petitioner terminated her
employment without cause. The labor arbiter, in
May 21, 2002, found that Davis committed serious misconduct (she was the
principal suspect for qualified theft committed inside petitioner's office
during work hours). There was a valid cause for her dismissal. Thus, the
complaint was dismissed for lack of merit. Upon appeal, NLRC affirmed the
labor arbiter in July 31, 2003 and denied her motion for reconsideration in
October 30, 2003. Upon petition for certiorari filed with the CA, CA on July 4,
2005 granted the petition holding that the labor arbiter and NLRC merely
adopted the findings of the NBI regarding respondent's culpability. Because
the affidavits of the witnesses were not verified, they did not constitute
substantial evidence. The labor arbiter and NLRC should have assessed
evidence independently as "unsubstantiated suspicions, accusations and
conclusions of employers (did) not provide legal justification for dismissing
an employee". Petitioner moved for reconsideration but it was denied.
Hence, this petition where petitioner argues that the ground for an
employee's dismissal need only be proven by substantial evidence. Thus, the
dropping of charges against an employee (especially on a technicality such
as lack of proper verification) or his subsequent acquittal does not preclude
an employer from dismissing him due to serious misconduct.

Issue:
Whether or not petitioner substantially proved the presence of valid cause for
respondent's termination.
Ruling:
Supreme Court granted the petition and ruled that petitioner validly
dismissed Davis for cause analogous to serious misconduct. Article 282 of
the Labor Code provides: Termination by Employer. An employer may
terminate an employment for any of the following causes:
(a) Serious misconduct or willful disobedience by the employee of the lawful
orders of his employer or his representatives in connection with his work;
(e) Other causes analogous to the foregoing.
Misconduct involves "the transgression of some established and definite rule
of action, forbidden act, a dereliction of duty, willful in character, and implies
wrongful intent and not mere error in judgment". For misconduct to be
serious and therefore a valid ground for dismissal, it must be: of grave and
aggravated character and not merely trivial or unimportant and connected
with the work of the employee.
In this case, petitioner dismissed Davis based on the NBI's finding that the
latter stole and used Yuseco's credit cards. But since the theft was not
committed against petitioner itself but against one of its employees,
respondent's misconduct was not work-related and therefore, she could not
be dismissed for serious misconduct. Nonetheless, Article 282 (e) of the
Labor Code talks of other analogous causes or those which are susceptible of
comparison to another in general or in specific detail. For an employee to be
validly dismissed for a cause analogous to those enumerated in Article 282,
the cause must involve a voluntary and/or willful act or omission of the
employee. A cause analogous to serious misconduct is a voluntary and/or
willful act or omission attesting to an employee's moral depravity. Theft
committed by an employee against a person other than his employer, if
proven by substantial evidence, is a cause analogous to serious misconduct.
Did petitioner substantially prove the existence of valid cause for
respondent's separation? Yes. The labor arbiter and the NLRC relied not only
on the affidavits of the NBI's witnesses but also on that of respondent. They
likewise considered petitioner's own investigative findings. Clearly, they did
not merely adopt the findings of the NBI but independently assessed

evidence presented by the parties. Their conclusion (that there was valid
cause for respondent's separation from employment) was therefore
supported by substantial evidence.

Magnolia Dairy Products Corporation v. NLRC


G.R. NO. 114952, January 29, 1996

Facts: Petitioner, a division of San Miguel Corporation (SMC), entered into a


contract of service with Skillpower, Inc., a duly organized corporation
engaged in the business of offering and providing manpower services to the
public. On June 11, 1983, Skillpower, Inc., assigned private respondent Jenny
A. Calibo to petitioners Tetra Paster Division with these functions: (i)to
remove bulgings (damaged goods) from dilapidated cartons; (ii)to replace
damaged goods and re-paste the carton thereof; (iii)to dispose the damaged
goods or returned goods from Magnolias warehouse to avoid bad odors; and
(iv)to clean leftovers of leaking tetra pack by mopping or washing the
contaminated premises. In September 1986, Skillpower, Inc., pulled-out
private respondent from petitioners Tetra Paster Division, but assigned her
back on May 2, 1987 with the same functions. When petitioners contract
with Skillpower, Inc., expired, private respondent applied with Lippercon
Services, Inc., also a corporation engaged in providing manpower services. In
July 1987, Lippercon Services, Inc., assigned her to petitioners Tetra Paster
Division as a cleaning aide. In December 1987, she was terminated from
service due to petitioners installation of automated machines. On July 11,
1989, private respondent instituted a complaint for illegal dismissal against
petitioner. In answer thereto, petitioner averred that it has no employeremployee relationship with private respondent and that the dismissal was
prompted by the installation of labor saving devices - an authorized cause for
dismissal under the Labor Code, as amended. The Labor Arbiter ruled that
petitioner is the private respondents employer because Skilipower, Inc., and
Lippercon Services, Inc., were mere labor-only contractors falling under
Section 9, Rule VIII, Book III of the Omnibus Rules Implementing the Labor
Code. The installation of labor saving devices was also ruled a valid ground
for the termination of private respondents employment, but the Labor
Arbiter emphasized that this did not exculpate petitioner from the charge of
illegal dismissal for its failure to observe the due process of law in
terminating from service its employee.
Issue: Was there an authorized cause for the termination?
Ruling: The law authorizes an employer, like the herein petitioner, to
terminate the employment of any employee due to the installation of labor
saving devices. The installation of these devices is a management
prerogative, and the courts will not interfere with its exercise in the absence
of abuse of discretion, arbitrariness, or maliciousness on the part of
management, as in this case. Nonetheless, this did not excuse petitioner
from complying with the required written notice to the employee and to the

Department of Labor and Employment (DOLE) at least one month before the
intended date of termination. This procedure enables an employee to contest
the reality or good faith character of the asserted ground for the termination
of his services before the DOLE.
Redundancy
Asian Alcohol Corporation v. NLRC
G.R. No. 131108, March 25, 1999
Facts: In September, 1991, the Parsons family, who originally owned the
controlling stocks in Asian Alcohol, were driven by mounting business losses
to sell their majority rights to prior Holdings, Inc. (hereinafter referred to as
Prior Holdings). The next month, Prior Holdings took over its management
and operation. To thwart further losses, Prior Holdings implemented a
reorganizational plan and other cost-saving measures. Some one hundred
seventeen (117) employees out of a total workforce of three hundred sixty
(360) were separated. Seventy two (72) of them occupied redundant
positions that were abolished. Of these positions, twenty one (21) were held
by union members and fifty one (51) by non-union members. The six (6)
private respondents are among those union members whose positions were
abolished due to redundancy. Private respondents Carias, Martinez, and
Sendon were water pump tenders; Amacio was a machine shop mechanic;
Verayo was a briquetting plant operator while Tormo was a plant helper
under him. They were all assigned at the Repair and Maintenance Section of
the Pulupandan plant. In October, 1992, they received individual notices of
termination effective November 30, 1992.
Issue: Was the termination valid due to redundancy?
Ruling: In the case at bar, Prior Holdings took over the operations of Asian
Alcohol in October 1991. Plain to see, the last quarter losses in 1991 were
already incurred under the new management. There were no signs that these
losses would abate. Irrefutable was the fact that losses have bled Asian
Alcohol incessantly over a span of several years. They were incurred under
the management of the Parsons family and continued to be suffered under
the new management of Prior Holdings. Ultimately, it is Prior Holding that will
absorb all the losses, including those incurred under the former owners of
the company. The law gives the new management every right to undertake
measures to save the company from bankruptcy. We find that the

reorganizational plan and comprehensive cost-saving program to turn the


business around were nor designed to bust the union of the private
respondent. Retrenched were one hundred seventeen (117) employees.
Seventy two (72) of them including private respondent were separated
because their positions had become redundant. In this context, what may
technically be considered as redundancy may verily be considered as
retrenchment measures. Their positions had to be declared redundant to cut
losses. Redundancy exists when the service capability of the work is in
excess of what is reasonably needed to meet the demands on the enterprise.
A redundant position is one rendered superfluous by any number of factors,
such as overhiring of workers, decreased volume of business, dropping of a
particular product line previously manufactured by the company or phasing
out of a service activity priorly undertaken by the business. Under these
conditions, the employer has no legal obligation to keep in its payroll more
employees than are necessary for the operation of its business. For the
implementation of a redundancy program to be valid, the employer must
comply with the following requisites: (1) written notice served on both the
employees and the Department of Labor and Employment at least one moth
prior to the intended date of retrenchment; (2) payment of separation pay
equivalent to at least one month pay or at least one month pay for every
year of service whichever is higher; (3) good faith in abolishing the
redundant positions; and (4) fair and reasonable criteria in ascertaining what
positions are to be declared redundant and accordingly abolished.
Wiltshire File Co., Inc., v. NLRC
G.R. No. 82249 February 7, 1991
Facts: Private respondent Vicente T. Ong was the Sales Manager of petitioner
Wiltshire File Co., Inc. ("Wiltshire") from 16 March 1981 up to 18 June 1985.
As such, he received a monthly salary of P14,375.00 excluding commissions
from sales which averaged P5,000.00 a month. He also enjoyed vacation
leave with pay equivalent to P7,187,50 per year, as well as hospitalization
privileges to the extent of P10,000.00 per year. On 13 June 1985, upon
private respondent's return from a business and pleasure trip abroad, he was
informed by the President of petitioner Wiltshire that his services were being
terminated. Private respondent maintains that he tried to get an explanation
from management of his dismissal but to no avail. On 18 June 1985, when
private respondent again tried to speak with the President of Wiltshire, the
company's security guard handed him a letter which formally informed him
that his services were being terminated upon the ground of redundancy.

Private respondent filed, on 21 October 1985, a complaint before the Labor


Arbiter for illegal dismissal alleging that his position could not possibly be
redundant because nobody (save himself) in the company was then
performing the same duties. Private respondent further contended that
retrenching him could not prevent further losses because it was in fact
through his remarkable performance as Sales Manager that the Company
had an unprecedented increase in domestic market share the preceding
year. For that accomplishment, he continued, he was promoted to Marketing
Manager and was authorized by the President to hire four (4) Sales
Executives five (5) months prior to his termination.
Issue: Is the termination valid due to redundancy?
Ruling: Turning to the legality of the termination of private respondent's
employment, The Court finds merit in petitioner's basic argument. The Court
was unable to sustain public respondent NLRC's holding that private
respondent's dismissal was not justified by redundancy and hence illegal. In
the first place, we note that while the letter informing private respondent of
the termination of his services used the word "redundant", that letter also
referred to the company having "incur[red] financial losses which [in] fact
has compelled [it] to resort to retrenchment to prevent further losses". 3
Thus, what the letter was in effect saying was that because of financial
losses, retrenchment was necessary, which retrenchment in turn resulted in
the redundancy of private respondent's position.
Smart Communications, Inc., v. Astorga
G.R. No. January 28, 2008
Facts: Regina M. Astorga (Astorga) was employed by respondent Smart
Communications, Incorporated (SMART) on May 8, 1997 as District Sales
Manager of the Corporate Sales Marketing Group/ Fixed Services Division
(CSMG/FSD). She was receiving a monthly salary of P33,650.00. As District
Sales Manager, Astorga enjoyed additional benefits, namely, annual
performance incentive equivalent to 30% of her annual gross salary, a group
life and hospitalization insurance coverage, and a car plan in the amount of
P455,000.00. In February 1998, SMART launched an organizational
realignment to achieve more efficient operations. This was made known to
the employees on February 27, 1998.6 Part of the reorganization was the
outsourcing of the marketing and sales force. Thus, SMART entered into a
joint venture agreement with NTT of Japan, and formed SMART-NTT

Multimedia, Incorporated (SNMI). Since SNMI was formed to do the sales and
marketing work, SMART abolished the CSMG/FSD, Astorgas division. To
soften the blow of the realignment, SNMI agreed to absorb the CSMG
personnel who would be recommended by SMART. SMART then conducted a
performance evaluation of CSMG personnel and those who garnered the
highest ratings were favorably recommended to SNMI. Astorga landed last in
the performance evaluation, thus, she was not recommended by SMART.
SMART, nonetheless, offered her a supervisory position in the Customer Care
Department, but she refused the offer because the position carried lower
salary rank and rate. Despite the abolition of the CSMG/FSD, Astorga
continued reporting for work. But on March 3, 1998, SMART issued a
memorandum advising Astorga of the termination of her employment on
ground of redundancy, effective April 3, 1998. Astorga received it on March
16, 1998.
Issue: Was there redundancy to justify the termination of Astorga?
Ruling: The Court agrees with the CA that the organizational realignment
introduced by SMART, which culminated in the abolition of CSMG/FSD and
termination of Astorgas employment was an honest effort to make SMARTs
sales and marketing departments more efficient and competitive. Indeed, out
of our concern for those lesser circumstanced in life, this Court has inclined
towards the worker and upheld his cause in most of his conflicts with his
employer. This favored treatment is consonant with the social justice policy
of the Constitution. But while tilting the scales of justice in favor of workers,
the fundamental law also guarantees the right of the employer to reasonable
returns for his investment.38 In this light, we must acknowledge the
prerogative of the employer to adopt such measures as will promote greater
efficiency, reduce overhead costs and enhance prospects of economic gains,
albeit always within the framework of existing laws. Accordingly, we sustain
the reorganization and redundancy program undertaken by SMART. However,
as aptly found by the CA, SMART failed to comply with the mandated one (1)
month notice prior to termination. The record is clear that Astorga received
the notice of termination only on March 16, 199839 or less than a month
prior to its effectivity on April 3, 1998. Likewise, the Department of Labor and
Employment was notified of the redundancy program only on March 6, 1998.

Retrenchment

Flight Attendants and Stewards Association of the Philippines


(FASAP) v. PAL G.R. No. 178083, July 22, 2008
Facts: Petitioner FASAP is the duly certified collective bargaining
representative of PAL flight attendants and stewards, or collectively known as
PAL cabin crew personnel. Respondent PAL is a domestic corporation
organized and existing under the laws of the Republic of the Philippines,
operating as a common carrier transporting passengers and cargo through
aircraft. On June 15, 1998, PAL retrenched 5,000 of its employees, including
more than 1,400 of its cabin crew personnel, to take effect on July 15, 1998.
PAL adopted the retrenchment scheme allegedly to cut costs and mitigate
huge financial losses as a result of a downturn in the airline industry brought
about by the Asian financial crisis. During said period, PAL claims to have
incurred P90 billion in liabilities, while its assets stood at P85 billion. In
implementing the retrenchment scheme, PAL adopted its so-called Plan 14
whereby PALs fleet of aircraft would be reduced from 54 to 14, thus
requiring the services of only 654 cabin crew personnel. PAL admits that the
retrenchment is wholly premised upon such reduction in fleet, and to the
strike staged by PAL pilots since this action also translated into a reduction of
flights. PAL claims that the scheme resulted in savings x x x amounting to
approximately P24 million per month savings that would greatly alleviate
PALs financial crisis. Prior to the full implementation of the assailed
retrenchment program, FASAP and PAL conducted a series of consultations
and meetings and explored all possibilities of cushioning the impact of the
impending reduction in cabin crew personnel. However, the parties failed to
agree on how the scheme would be implemented. Thus PAL unilaterally
resolved to utilize the criteria set forth in Section 112 of the PAL-FASAP
Collective Bargaining Agreement (CBA) in retrenching cabin crew personnel:
that is, that retrenchment shall be based on the individual employees
efficiency rating and seniority. While consultations between FASAP and PAL
were ongoing, the latter began implementing its retrenchment program by
initially terminating the services of 140 probationary cabin attendants only to
rehire them in April 1998. Moreover, their employment was made permanent
and regular.
Issue: Was the termination valid due to retrenchment?
Ruling: The burden clearly falls upon the employer to prove economic or
business losses with sufficient supporting evidence. Its failure to prove these
reverses or losses necessarily means that the employees dismissal was not

justified. Any claim of actual or potential business losses must satisfy certain
established standards, all of which must concur, before any reduction of
personnel becomes legal. These are: (1) That retrenchment is reasonably
necessary and likely to prevent business losses which, if already incurred, are
not merely de minimis, but substantial, serious, actual and real, or if only
expected, are reasonably imminent as perceived objectively and in good
faith by the employer; (2) That the employer served written notice both to
the employees and to the Department of Labor and Employment at least one
month prior to the intended date of retrenchment; (3) That the employer
pays the retrenched employees separation pay equivalent to one (1) month
pay or at least one-half () month pay for every year of service, whichever is
higher; (4) That the employer exercises its prerogative to retrench
employees in good faith for the advancement of its interest and not to defeat
or circumvent the employees right to security of tenure; and, (5) That the
employer used fair and reasonable criteria in ascertaining who would be
dismissed and who would be retained among the employees, such as status,
efficiency, seniority, physical fitness, age, and financial hardship for certain
workers. In view of the facts and the issues raised, the resolution of the
instant petition hinges on a determination of the existence of the first, fourth
and the fifth elements set forth above, as well as compliance therewith by
PAL, taking to mind that the burden of proof in retrenchment cases lies with
the employer in showing valid cause for dismissal; that legitimate business
reasons exist to justify retrenchment.

Meaning of retrenchment to prevent losses

Revidad v. NLRC G.R. No. 111105, June 27, 1995


Facts: It appears that sometime in March, 1988, private respondent Atlantic,
Gulf and Pacific Company of Manila, Inc. (hereafter, AG & P ) terminated the
services of 178 employees, including herein petitioners, under a redundancy
program. As a consequence, a complaint for illegal dismissal with prayer for
reinstatement was filed by herein petitioners (except Jose Espaol) with
public respondent. These cases were subsequently decided in favor of
petitioners, as a result of which they were reinstated on July 8, 1991 and
assigned to the Batangas plant of private respondent. The records show,
however, that pursuant to Presidential Directive No. 0191 2 issued on July 25,
1991 by the company's president and containing management's decision to

lay off 40% of the employees due to financial losses incurred from 19891990, AG & P implemented and effected, starting August 3, 1991, the
temporary lay-off of some 705 employees. By reason thereof, the AG & P
United Rank and File Association (URFA, for facility), which was the
employees' union, staged a strike. In a conciliation conference over the labor
dispute held before the National Conciliation and Mediation Board on August
13, 1991, the parties agreed to submit the legality of the lay-offs to voluntary
arbitration.
Issue: Was there a need for retrenchment to prevent losses?
Ruling: The Court is accordingly convinced, and so hold, that both the
retrenchment program of private respondent and the dismissal of petitioners
were valid and legal. First, it has been sufficiently and convincingly
established by AG & P before the voluntary arbitrator that it was suffering
financial reverses. Even the rank and file union at AG & P did not contest the
fact that management had been undergoing financial difficulties for the past
several years. Hence, the voluntary arbitrator considered this as an
admission that indeed AG & P was actually experiencing adverse business
conditions which would justify the exercise of its management prerogative to
retrench in order to avoid the not so remote possibility of the closure of the
entire business which, in the opinion of the voluntary arbitrator, would in the
last analysis be adverse to both the management and the union. Second, the
voluntary arbitrator's conclusions were premised upon and substantiated by
the audited financial statements and the auditor's reports of AG & P for the
years 1987 to 1991. 14 These, financial statements audited by independent
external auditors constitute the normal and reliable method of proof of the
profit and loss performance of a company. Third, contrary to petitioners'
asseverations, proof of actual financial losses incurred by the company is not
a condition sine qua non, for retrenchment. Retrenchment is one of the
economic grounds to dismiss employees, which is resorted to by an
employer primarily to avoid or minimize business losses. In its ordinary
connotation, the phrase "to prevent losses" means that retrenchment or
termination of the services of some employees is authorized to be
undertaken by the employer sometime before the anticipated losses are
actually sustained or realized. It is not, in other words, the intention of the
lawmaker to compel the employer to stay his hand and keep all his
employees until after losses shall have in fact materialized. If such an intent
were expressly written into the law, that law may well be vulnerable to

constitutional attack as unduly taking property from one man to be given to


another.

Best evidence of losses


Saballa v. NLRC G.R. Nos. 102472-84, August 22, 1996
Facts: On April 23, 1988, Arturo Margallo, General Manager of the Camarines
Sur III Electric Cooperative, Inc. (private respondent herein), issued
Memorandum No. 24-88 providing for austerity measures (retrenchment).
On the same date, private respondent filed with the Department of Labor
and Employments Regional Office No. V in Legaspi City a Notice of
Retrenchment covering some thirty (30) employees on the basis of the
guidelines and priorities specified in the abovementioned memorandum. The
Regional Director in his Resolution dated June 6, 1988 granted authority to
terminate the employment of the said 30 employees pursuant to the
categories, priorities and effective dates under Memo No. 24-88. On June
20, 1988, Margallo issued Memorandum No. 60-88[8] declaring some fiftytwo (52) employees, including herein petitioners Juan Saballa, Lailani
Miranda, Nelia Ibarrientos, Helen Quiambao, Wilberto Amparado and Fidel
Manaog, on forced leave without pay for a period of three (3) months,
effective five (5) days after receipt by the employees concerned. Such
forced leave was purportedly part of the cost-saving measures instituted to
enable the Coop to meet (its) financial obligations especially with NPC and
NEA. The memo assured the subject employees of rehiring as soon as the
Coop shall have financially recovered/regained its financial viability expected
within the above specified period. A copy of said memorandum was
furnished the Regional Office of the DOLE in Legaspi City on June 23, 1988.
Issue: Is the termination justified?
Ruling: Given the preceding discussion, it is indisputable that private
respondent failed to meet the abovestated requirements for a valid
retrenchment due to imminent business losses, since (1) the expected losses
were not proven to be substantial; (2) the expected losses were not shown to
be imminent as private respondent was able to afford re-hiring of some of
the non-tenurial employees on forced leave; and (3) the retrenchment was

not shown to be reasonably necessary and likely to effectively prevent the


expected losses. And, neither the losses already realized nor the imminent
losses sought to be forestalled were proven by sufficient and convincing
evidence. Moreover, the private respondent admitted but failed to explain
why it rehired previously retrenched employees who were even non-tenurial,
during the pendency of the complaints for illegal dismissal, when there were
still a number of regular employees in the same situation. Petitioners also
alleged that, immediately after their termination, private respondent hired
replacements to fill their positions. This allegation, supported by the affidavit
of petitioners witness Marlene Cerillo, remained unrebutted and
uncontroverted by private respondent. This militates strongly against private
respondents claim of good faith in implementing reductions of its work force
to reduce costs. And, although Memorandum No. 24-88 set out the
priorities/categories to be observed in implementing the personnel reduction
program, the same was not applied to the petitioners, who, being regular
employees, did not fall under any of the categories mentioned in said
memorandum, and who therefore ought not have been retrenched at least
not under said memorandum. This Court has repeatedly enjoined employers
to adopt and observe fair and reasonable standards to effect retrenchment.
The private respondent adopted in its Memo No. 24-88 a set of criteria in
retrenching employees in accordance with its cost-reduction program, but
discarded these self-imposed criteria when it came to the retrenchment of
petitioners, thus rendering its action arbitrary. Further, it is undisputed that
Sec. 1, Article XI of the Collective Bargaining Agreement of September 13,
1988 between private respondent and the employees union stipulates that
seniority in service to the company shall be x x x considered in lay-off or
reduction of working force. Thus, the subject retrenchment is violative of
this stipulation as well. The private respondents demonstrated arbitrariness
in the selection of which of its employees to retrench is further proof of the
illegality of the subject retrenchment, not to mention private respondents
bad faith. And lastly, we note that the termination was made effective five
(5) days after receipt of notice. The lack of the thirty (30) days notice prior to
retrenchment as required under Article 283 of the Labor Code further bolster
the conclusion that the subject retrenchment was illegal. Thus, it is ineludible
that we should agree with petitioners contention that, contrary to the public
respondents finding, the retrenchment of petitioners by private respondent
constituted illegal dismissal.

Redundancy vs. Retrenchment


Andrada v. NLRC G.R. No. 173231, December 28, 2007
Facts: Petitioners Ruben Andrada, Jovencio Poblete, Filamer Alfonso, Harvey
Cayetano, Vicente Mantala, Jr., Bernaldo delos Santos, and Joven Pabustan
were hired on various dates from 1995 up to 1997 and worked as architects,
draftsmen, operators, engineers, and surveyors in the Subic Legend Resorts
and Casino, Inc. (Legend) Project Development Division on various projects.
Hwa Puay, Flordeliza Maria Reyes Rayel, and other corporate officers are
impleaded in this case in their official capacities as officers of Legend. On
January 6, 1998, Legend sent notice to the Department of Labor and
Employment of its intention to retrench and terminate the employment of
thirty-four (34) of its employees, which include petitioners, in the Project
Development Division. Legend explained that it would be retrenching its
employees on a last-in-first-out basis on the strength of the updated status
report of its Project Development Division, as follows: (1) shelving of the
condotel project until economic conditions in the Philippines improve; (2)
completion of the temporary casino in Cubi by mid-February 1998; (3)
subcontracting the super structure work of Grand Legend to a third party; (4)
completion of most of the rectification work at the Legenda Hotel; (5)
completion of the temporary casino in Cubi; and (6) abolition of the
Personnel and Administrative Department of the Project Development
Division and transfer of its function back to Legends Human Resources
Department. The following day, on January 7, 1998, Legend sent the 34
employees their respective notices of retrenchment, stating the same
reasons for their retrenchment.
Issue: Whether or not redundancy is established in this case at bar?
Ruling: The Court ruled that Legend failed to establish redundancy.
Retrenchment and redundancy are two different concepts; they are not
synonymous and therefore should not be used interchangeably. Thus, simply
put, redundancy exists when the number of employees is in excess of what is
reasonably necessary to operate the business. The declaration of redundant
positions is a management prerogative. The determination that the
employees services are no longer necessary or sustainable and therefore
properly terminable is an exercise of business judgment by the employer.
The wisdom or soundness of this judgment is not subject to the discretionary

review of the Labor Arbiter and NLRC. It is however not enough for a
company to merely declare that positions have become redundant. It must
produce adequate proof of such redundancy to justify the dismissal of the
affected employees. In Panlilio v. NLRC, we said that the following evidence
may be proffered to substantiate redundancy: the new staffing pattern,
feasibility studies/proposal, on the viability of the newly created positions,
job description and the approval by the management of the restructuring. In
another case, it was held that the company sufficiently established the fact
of redundancy through affidavits executed by the officers of the respondent
PLDT, explaining the reasons and necessities for the implementation of the
redundancy program. According to the CA, Legend proved the existence of
redundancy when it submitted a status review of its project division where it
reported that the 78-man personnel exceeded the needs of the company.
The report further stated that there was duplication of functions and
positions, or an over supply of employees, especially among architects,
engineers, draftsmen, and interior designers. Thus, in the same way, we held
that the basis for retrenchment was not established by substantial evidence,
we also rule that Legend failed to establish by the same quantum of proof
the fact of redundancy; hence, petitioners termination from employment
was illegal.

Closure or Cessation of Business Operations


Eastridge Golf Club, Inc., v. Eastridge Golf Club, Inc., Labor UnionSuper G.R. No. 166760, August 22, 2008
Facts: Petitioner employed respondents as kitchen staff in its Food and
Beverage (F&B) Department. Effective October 1, 1999, petitioner
terminated the employment of respondents on the ground that the
operations of the F&B Department had been turned over to concessionaire
Mother's Choice Meat Shop & Food Services. Petitioner filed with the
Department of Labor and Employment (DOLE) an Establishment Termination
Report, stating that it laid off the respondents due to company
reorganization/downsizing and transfer of operations to a concessionaire.
Respondents filed with the National Labor Relations Commission (NLRC),
Regional Arbitration Branch, a complaint for illegal dismissal, unfair labor
practice and payment of 13th month pay. They claimed that their dismissal
was not based on any of the causes allowed by law, and that it was effected
without due process. Petitioner denied respondents' claims, pointing out that

several months before their dismissal, it issued various office memoranda


informing respondents that, to minimize company losses, the management
decided to bid out a part of its operations, specifically the F&B Department,
to a concessionaire. The partial cessation of operations was bonafide.
Issue: Was there a valid cessation to justify the dismissal of n employee?
Ruling: Closure or cessation of business is the complete or partial cessation of
the operations and/or shut-down of the establishment of the employer. It is
carried out to either stave off the financial ruin or promote the business
interest of the employer. Unlike retrenchment, closure or cessation of
business, as an authorized cause of termination of employment, need not
depend for validity on evidence of actual or imminent reversal of the
employer's fortune. Article 283 authorizes termination of employment due to
business closure, regardless of the underlying reasons and motivations
therefor, be it financial losses or not. In the case under review, the cause
invoked by petitioner in terminating the employment of respondents is not
retrenchment but cessation of a single aspect of its business undertaking,
i.e., the F&B Department. This is evident in the notices of termination it sent
to respondents where petitioner indicated that it had withdrawn from the
direct operation of the F&B Department and had transferred the
management thereof to the concessionaire.[53] Also, in the various office
memoranda it posted, petitioner explained that the underlying reason for the
cessation of its F&B undertaking was that the economic depression had
affected its sales and operations and resulted in increased overhead
expenses and decreased incomes. Cessation of its F&B operations being the
cause invoked by petitioner to terminate the employment of respondents, it
need not present evidence of financial losses to justify such business
decision. Thus, the Court agrees with petitioner that the CA erred when it
declared that, for lack of evidence of financial losses, petitioner's cessation
of its F&B operations was not a valid cause to terminate the employment of
respondents. The decision to close business is a management prerogative
exclusive to the employer, the exercise of which no court or tribunal can
meddle with, except only when the employer fails to prove compliance with
the requirements of Art. 283, to wit: a) that the closure/cessation of business
is bona fide, i.e., its purpose is to advance the interest of the employer and
not to defeat or circumvent the rights of employees under the law or a valid
agreement; b) that written notice was served on the employees and the
DOLE at least one month before the intended date of closure or cessation of
business; and c) in case of closure/cessation of business not due to financial

losses, that the employees affected have been given separation pay
equivalent to month pay for every year of service or one month pay,
whichever is higher. The decision to close business is a management
prerogative exclusive to the employer, the exercise of which no court or
tribunal can meddle with, except only when the employer fails to prove
compliance with the requirements of Art. 283, to wit: a) that the
closure/cessation of business is bona fide, i.e., its purpose is to advance the
interest of the employer and not to defeat or circumvent the rights of
employees under the law or a valid agreement; b) that written notice was
served on the employees and the DOLE at least one month before the
intended date of closure or cessation of business; and c) in case of
closure/cessation of business not due to financial losses, that the employees
affected have been given separation pay equivalent to month pay for
every year of service or one month pay, whichever is higher.
Industrial Timber Corporation v. NLRC G.R. No. 107302 and 107306,
June 10, 1997
Facts: Industrial Timber Corporation (ITC) is a corporation registered under
Philippine laws and is engaged in the business of manufacturing and
processing veneer and plywood products. It used to operate a veneer
processing plant known as the Butuan Logs Plant and a veneer and plywood
processing plant known as the Stanply Plant. Both plants occupied a single
compound with a common point for ingress and egress and were both leased
from Industrial Plywood Group Corporation. Both plants had also two (2)
distinct bargaining units represented by separate labor unions and had
separate collective bargaining agreements with their respective principals.
ITC Butuan Logs Workers Union-WATU (Union) represented the rank and file
employees of the Butuan Logs Plant. Sometime in 1989, ITC decided to
permanently stop and close its veneer production at its Butuan Logs Plant
due to impending heavy financial losses resulting from high production
costs, erratic supply of raw materials and depressed prices and market
conditions for its wood products. Accordingly, on November 9, 1989, ITC
served a written notice to all its employees in the said plant and to the
Butuan District Office of the Department of Labor and Employment (DOLE)
stating that effective December 10, 1989 or thirty (30) days thereafter, it
would cease operations at said plant. After receiving the notice, the
employees therein, through their union representative, filed a formal
objection to the intended shutdown. Consequently, conciliation proceedings
were conducted at the DOLE District Office pursuant to the provisions of the

Collective Bargaining Agreement (CBA) on grievances. The parties, however,


failed to settle their differences.
Issue: Was there an established valid cessation?
Ruling: Article 283 of the Labor Code clearly provides inter alia that the
employer may terminate the employment of his employees to prevent
losses. Closure or cessation of operations for economic reasons is, therefore,
recognized as a valid exercise of management prerogative. The
determination to cease operations is a prerogative of management which the
State does not usually interfere with, as no business or undertaking must be
required to continue operating at a loss simply because it has to maintain its
workers in employment. Such an act would be tantamount to a taking of
property without due process of law. However, the burden of proving that
such closure is bona fide falls upon the employer. In this case, petitioner
corporation presented the analysis of an independent certified public
accountant, showing in detail the imminent losses it would suffer should it
continue its operations. It is understandable that no audited financial
statements or other similar documents were presented as the company is
claiming impending future losses, not past or actual ones. Moreover, the fact
that petitioner company has ceased operations and has not resumed to do
so only reinforces its claim to a valid closure, not to mention the other
established fact that its Stanply Plant has also the capacity and capability to
produce veneer, the product it solely manufactured in its now closed plant.
The foregoing notwithstanding, petitioner corporation complied with the
requirements mandated by law to effectuate valid termination of
employment on account of closure. Under the law, for an employer to validly
terminate the service of his employees under the aforesaid ground, he has to
comply with two (2) requirements, namely: (a) serving a written notice on the
workers and the DOLE at least one (1) month before the effective date of the
closure and (b) payment of separation pay equivalent to one (1) month pay
or at least one-half (1/2) month pay for every year of service, whichever is
higher, with a fraction of at least six (6) months to be considered one (1)
whole year. The records bear out that petitioner had sufficiently complied
with the aforecited requirements. It informed its employees and the DOLE
District Office at Butuan of the termination of service of the employees
effective December 10, 1989 in a Letter dated November 9, 1989. The
employees were, likewise, informed of the availability for release of the funds
for their separation pay and other CBA benefits. Unfortunately, only 63
employees availed of the benefits. The rest chose to file the instant action.

Disease or illness
Crayons Processing, Inc., v. Pula G.R. No. 167727, July 30, 2007
Facts: Petitioner Crayons Processing, Inc. (Crayons) employed respondent
Felipe Pula (Pula) as a Preparation Machine Operator beginning June 1993.
On 27 November 1999, Pula, then aged 34, suffered a heart attack and was
rushed to the hospital, where he was confined for around a week. Pulas wife
duly notified Crayons of her husbands medical condition. Upon his discharge
from the hospital, Pula was advised by his attending physician to take a
leave of absence from work and rest for three (3) months. Subsequently, on
25 February 2000, Pula underwent an Angiogram Test at the Philippine Heart
Center under the supervision of a Dr. Recto, who advised him to take a twoweek leave from work. Following the angiogram procedure, respondent was
certified as fit to work by Dr. Recto. On 11 April 2000, Pula returned to
work, but 13 days later, he was taken to the company clinic after
complaining of dizziness. Diagnosed as having suffered a relapse, he was
advised by his physician to take a leave of absence from work for one (1)
month. Pula reported back for work on 13 June 2000, armed with a
certification from his physician that he was fit to work. However, Pula
claimed that he was not given any post or assignment, but instead, on 20
June 2000, he was asked to resign with an offer from Crayons of P12,000 as
financial assistance. Pula refused the offer and instead filed a complaint for
illegal dismissal with prayer for damages and the payment of holiday
premium, 5 days service incentive leave pay, and 13th month pay for 1999.
The complaint was filed against Crayons, Clothman Knitting Corp., Nixon Lee,
Paul Lee, Peter Su, and Ellen Caluag.
Issue: Was the dismissal valid on account of illness?
Ruling: For a dismissal on the ground of disease to be considered valid, two
requisites must concur: (a) the employee must be suffering from a disease
which cannot be cured within six months and his continued employment is
prohibited by law or prejudicial to his health or to the health of his coemployees; and (b) a certification to that effect must be issued by a
competent public health authority. The burden falls upon the employer to
establish these requisites and in the absence of such certification, the
dismissal must necessarily be declared illegal. As succinctly stressed in Tan v.
NLRC, it is only where there is a prior certification from a competent public

authority that the disease afflicting the employee sought to be dismissed is


of such nature or at such stage that it cannot be cured within six (6) months
even with proper medical treatment that the latter could be validly
terminated from his job. Without the required certification, the
characterization or even diagnosis of the disease would primarily be shaped
according to the interests of the parties rather than the studied analysis of
the appropriate medical professionals. The requirement of a medical
certificate under Article 284 cannot be dispensed with; otherwise, it would
sanction the unilateral and arbitrary determination by the employer of the
gravity or extent of the employee's illness and thus defeat the public policy
in the protection of labor.
Competent Public Health Authority
Cebu Royal Plant v. Minister of Labor G.R. No. L-58639, August 12,
1987
Facts: Ramon Pilones, private respondent, was employed on February 16,
1978 on a probationary period of employment for six (6) months with
petitioner CRP. After said period, he underwent medical examination for
qualification as regular employee but the results showed that he is suffering
from PTB minimal. Consequently, he was informed of the termination of his
employment by respondent since his illness was not curable within 6 months.
Pilones complained against his termination before the Ministry of Labor which
dismissed the same. The dismissal was reversed by the public respondent
who ordered the reinstatement and payment of back wages. Granting
reinstatement, the public respondent argues that Pilones was already a
permanent employee at the time of his dismissal and so was entitled to
security of tenure. The alleged ground for his removal, to wit, pulmonary
tuberculosis minimal, was not certified as incurable within six months as to
justify his separation and that the petitioner should have first obtained a
clearance, as required by the regulations then in force, for the termination of
his employment. CRP claims that the private respondent was still on
probation at the time of his dismissal and so had no security of tenure. The
dismissal was necessary for the protection of the public health, as he was
handling ingredients in the processing of soft drinks which were being sold to
the public.
Issue: Was the dismissal proper?

Ruling: No. The dismissal was not proper. Under Article 282 of the Labor
Code, an employee who is allowed to work after a probationary period shall
be considered a regular employee. Pilones was already on permanent status
when he was dismissed on August 21, 1978, or four days after he ceased to
be a probationer. As such, he could validly claim the security of tenure
guaranteed to him by the Constitution and the Labor Code. The petitioner
claims it could not have dismissed the private respondent earlier because
the x-ray examination was made only on August 17, 1978, and the results
were not immediately available. That excuse is untenable. We note that
when the petitioner had all of six months during which to conduct such
examination, it chose to wait until exactly the last day of the probation
period. The applicable rule on the ground for dismissal invoked against him is
Section 8, Rule I, Book VI, of the Rules and Regulations Implementing the
Labor Code which states that the employer shall not terminate his
employment unless there is a certification by a competent public health
authority that the disease is of such nature or at such a stage that it cannot
be cured within a period of six (6) months even with proper medical
treatment. The record does not contain the certification required by the
above rule. Hence, dismissal was illegal. It is also worth noting that the
petitioners application for clearance to terminate the employment of the
private respondent was filed with the Ministry of Labor only on August 28,
1978, or seven days after his dismissal. As the NLRC has repeatedly and
correctly said, the prior clearance rule (which was in force at that time) was
not a trivial technicality. It required not just the mere filing of a petition or
the mere attempt to procure a clearance but that the said clearance be
obtained prior to the operative act of termination. Although we must rule in
favor of his reinstatement, this must be conditioned on his fitness to resume
his work, as certified by competent authority.
Procedural due process
1) Procedure to be observed in just causes termination
Twin-Notice Rule
Ruffy v. NLRC G.R. No. 84193, February 15, 1990
Facts: The complainant was employed in December 1977 by the respondent
with the salary of P37.31 a day. He was assigned in the Materials and Supply
Section, Supply and Warehousing Department of the respondent. His duties,
among others, were to verify and check incoming materials and supplies and
issuing requisitioned materials and supplies to authorized personnel of the

various departments. On November 3, 1984, complainant issued twenty-five


(25) sets of roller bearings valued at P15,650.00 covered by Material Issue
Slip (M.I.S.) No. 121676 to a person who signed his name as Role. These
bearings were never received by the requisitioning section concerned. In the
investigation that ensued, it was gathered by the respondent that the
bearings were sold to factoria de Nasugbu for P8,250 by Anastacio Maulleon,
Jr., an employee of the respondent whose employment was terminated in
connection with this case. In the process, the complainant was asked
whether Alfredo Role, also an employee of respondent, was the same person
who received said bearings. In reply, complainant answered that he could not
remember. Role on his part denied having received said bearings.
Consequently, on December 31, 1984, the complainant was dismissed from
the service for breach of trust, gross negligence and flagrant inefficiency with
forfeiture of all rights and privileges.
Issue: Was the procedure for dismissal proper?
Ruling: The law lays down the procedure prior to the dismissal of an
employee. It need not be observed to the letter, but at least, it must be done
in the natural sequence of notice, hearing and judgment. In the case at bar,
there is no doubt that at the very outset, that is, prior to investigation, the
petitioner was informed that his services had been terminated. He was made
to air his side subsequently, it is true, yet the stubborn fact remains that
notwithstanding such an opportunity, if an opportunity it was, he had been
dismissed from the firm. The Court has held that the procedure under Batas
Blg. 130 and the rules implementing it are conditions sine qua non, before
dismissal may be validly effected.
King of Kings Transport, Inc., v. Mamac G.R. No. 166208, June 29, 2007
Facts: Petitioner KKTI is a corporation engaged in public transportation and
managed by Claire Dela Fuente and Melissa Lim. Respondent Mamac was
hired as bus conductor of Don Mariano Transit Corporation (DMTC) on April
29, 1999. The KKTI employees later organized the Kaisahan ng mga Kawani
sa King of Kings (KKKK) which was registered with DOLE. Respondent was
elected KKKK president. Respondent was required to accomplish a
Conductors Trip Report and submit it to the company after each trip. As a
background, this report indicates the ticket opening and closing for the
particular day of duty. After submission, the company audits the reports.
Once an irregularity is discovered, the company issues an Irregularity

Report against the employee, indicating the nature and details of the
irregularity. Thereafter, the concerned employee is asked to explain the
incident by making a written statement or counter-affidavit at the back of the
same Irregularity Report. After considering the explanation of the employee,
the company then makes a determination of whether to accept the
explanation or impose upon the employee a penalty for committing an
infraction. That decision shall be stated on said Irregularity Report and will be
furnished to the employee. Upon audit of the October 28, 2001 Conductors
Report of respondent, KKTI noted an irregularity. It discovered that
respondent declared several sold tickets as returned tickets causing KKTI to
lose an income of eight hundred and ninety pesos. While no irregularity
report was prepared on the October 28, 2001 incident, KKTI nevertheless
asked respondent to explain the discrepancy. In his letter,[3] respondent said
that the erroneous declaration in his October 28, 2001 Trip Report was
unintentional. He explained that during that days trip, the windshield of the
bus assigned to them was smashed; and they had to cut short the trip in
order to immediately report the matter to the police. As a result of the
incident, he got confused in making the trip report. On November 26, 2001,
respondent received a letter terminating his employment effective November
29, 2001. The dismissal letter alleged that the October 28, 2001 irregularity
was an act of fraud against the company.
Issue: Was there compliance with the procedural due process for dismissal?
Ruling: None. First, respondent was not issued a written notice charging him
of committing an infraction. The law is clear on the matter. A verbal appraisal
of the charges against an employee does not comply with the first notice
requirement. In Pepsi Cola Bottling Co. v. NLRC, the Court held that
consultations or conferences are not a substitute for the actual observance
of notice and hearing. Also, in Loadstar Shipping Co., Inc. v. Mesano, the
Court, sanctioning the employer for disregarding the due process
requirements, held that the employees written explanation did not excuse
the fact that there was a complete absence of the first notice. Second, even
assuming that petitioner KKTI was able to furnish respondent an Irregularity
Report notifying him of his offense, such would not comply with the
requirements of the law. We observe from the irregularity reports against
respondent for his other offenses that such contained merely a general
description of the charges against him. The reports did not even state a
company rule or policy that the employee had allegedly violated. Likewise,
there is no mention of any of the grounds for termination of employment

under Art. 282 of the Labor Code. Thus, KKTIs standard charge sheet is not
sufficient notice to the employee. Third, no hearing was conducted.
Regardless of respondents written explanation, a hearing was still necessary
in order for him to clarify and present evidence in support of his defense.
Moreover, respondent made the letter merely to explain the circumstances
relating to the irregularity in his October 28, 2001 Conductors Trip Report.
He was unaware that a dismissal proceeding was already being effected.
Thus, he was surprised to receive the November 26, 2001 termination letter
indicating as grounds, not only his October 28, 2001 infraction, but also his
previous infractions.
IBM Philippines v. NLRC G.R. No. 117221, April 13, 1999
Facts: Private Respondent filed a complaint before the Arbitration Branch of
the Department of Labor and Employment (DOLE) for an illegal dismissal by
herein petitioner, private respondent claimed that he was not given the
opportunity to be heard and was summarily dismissed. Petitioner contend
that he was given a chance or warning to improve his attitude toward
attendance but never did, and was duly informed thru emails, it also pointed
out that as an employee of IBM they are assigned IDs and passwords,
employees may also respond/reply thru email by encoding his messageresponse and admits also that the system automatically records the time and
date of each message was sent or received including the identification of the
sender and the receiver thereof. Petitioner attached to its position papers
copies of print-outs which allegedly contains computer message/entries sent
by petitioner to private respondent thru IBMs internal computer system.
Through this computer print-outs petitioner sought to prove that private
respondent was sufficiently notified of the charges and was guilty thereof for
failure to deny the same. Prior to the release of the labor arbiters decision
private respondent filed a Motion to admit attached new evidence for the
complainant. The Labor arbiters decision upheld the print-out attached by
petitioner as evidence and promulgated a resolution ordering petitioner to
pay private respondent salary from June 1 to August 31, 1990 excluding all
benefits. Aggrieved with the decision private respondent appealed to the
NLRC which ordered reinstatement to complainant to its former position with
his seniority rights, backwages from August 31, 1990 in the amount of P40,
516, 65 a month including all its benefits and bonuses. Hence, this petition.

Issue: Did NLRC commit grave abuse of discretion in holding that no just
cause or due process was observed in dismissing private respondent
because computer print-outs are inadmissible in evidence?
Ruling: Petitioner contend that in administrative /labor cases the technical
rules on evidence are not binding hence, the computer print-outs need not
be identified nor authenticated, same reason why private respondent was
allowed to submit additional evidences even after the case was deemed
submitted for resolution. However, the liberality of procedure in
administrative actions is subject to limitations imposed by basic
requirements of due process; this procedural rule should not be construed as
a license to disregard certain fundamental evidenciary rules. The evidence
presented before us must be at least have a modicum of admissibility for it
to be given some probative value. The computer print-outs, which constitute
only evidence of petitioners, afford no assurance of their authenticity since
they are unsigned. The liberal view in the conduct of proceedings before
administrative agencies, have nonetheless consistently required some
PROOF OF AUTHENTICITY OR RELIABILITY as condition for the admission of
documents. The procedural technicality and concerns are more paramount
principles and requirements of due process, which may not be sacrificed to
speed or expediency, Article 22 of the Labor Code which states that DUE
PROCESS MUST NEVER BE SUBORDINATED TO EXPEDIENCY OR DISPATCH
Administrative Hearing
Asia Terminals, Inc., v. Marbella G.R. No. 149074, August 10, 2006
Facts: Asian Terminals, Inc. (Asian Terminals) petitioner, is a domestic
corporation and the exclusive provider of arrastre and stevedoring services
at the Manila South Harbor. Rodolfo G. Corvite, Jr., also a petitioner, is its
President. Respondents are employees as stevedores of Asian Terminals. It is
not disputed that early in the evening of April 30, 1994, respondents and
other stevedores, who formed one group, were assigned to unload the cargo
of the M/V Huang Jin Shua. The work of the group could not be completed if
one stevedore was absent. The parties, however, have opposing versions of
what transpired next. Officer-in-charge R.F. Salazar of Asian Terminals Legal
Department conducted an investigation of the incident. In his report and
investigation dated May 15, 1994, he found respondents liable for refusal to
work penalized by dismissal from the service considering that they
committed the same offense for the second time. On September 23, 1994,

respondents were dismissed from the service. On December 14, 1994,


respondents filed with the Arbitration Branch of the National Labor Relations
Commission (NLRC) a complaint for illegal dismissal. Impleaded as
respondents were herein petitioners. On appeal, the NLRC, in its Decision
dated November 26, 1997, reversed the Labor Arbiters judgment. The NLRC
found that respondents were denied due process and that they did not refuse
to work. It was an officer of Asian Terminals who ordered respondents to stop
working when no personnel could replace the absent employee. Asian
Terminals filed a motion for reconsideration, but it was denied by the NLRC.
Asian Terminals then seasonably filed a petition for certiorari with this Court.
Issue: Was the dismissal proper?
Ruling: Here, Asian Terminals cited as a just cause respondents willful
disobedience or refusal to work. The offense of willful disobedience requires
the concurrence of two (2) requisites: (1) the employees assailed conduct
must have been willful, that is characterized by a wrongful and perverse
attitude; and (2) the order violated must have been reasonable, lawful, made
known to the employee and must pertain to the duties which he had been
engaged to discharge. Settled is the rule that in an illegal dismissal case, the
onus probandi is on the employer to prove that the dismissal of an employee
is for a valid cause. In this case, we agree with the Court of Appeals that
Asian Terminals failed to prove willful disobedience on the part of
respondents. Its own Position Paper states that they did not refuse to work.
Shoppers Gain Supermart v. NLRC G.R. No. 110731, July 26, 1996
Facts: From 1982 to 1990, private respondents had worked in the Shoppers
Gain Supermarket in various capacities as "merchandiser, cashier, bagger,
check-out personnel, sales lady, printer/film and warehouseman" for at least
one year each. Private respondents were part of a pool of workers supplied
by three (3) manpower service companies under "labor-only" contracts. In
December of 1990, due to an unavoidable circumstance, petitioner
constrained to stop its business and consequently terminate its contract with
the three (3) manpower service companies. Petitioner was able to pay
separation pays for its regular employees but not for private respondents. A
complaint for illegal dismissal was filed for which the Labor Arbiter rendered
a decision finding Shoppers Gain Supermarket guilty of labor-only contracting
and ordered it to pay separation pay and backwages to respondents. On
appeal, the National Labor Relations Commission affirmed said decision.

Elevating the case to the Supreme Court, petitioners raised the following
grounds inter alia: (a) That for employer-employee to exist, the following
requirements must be satisfied, namely: (1) selection and engagement of the
employees; (2) the payment of wages; (3) the power of dismissal; and (4) the
power to control employees' conduct; and (b) Since the manpower agencies
themselves admitted per their respective position papers that they selected,
hired, paid, disciplined, dismissed and controlled the private respondents, it
followed that the latter are not the employees of the petitioner corporation
but of the agencies only.
Issue: Was the dismissal illegal?
Ruling: The Supreme Court held that what was controlling in the issue is the
provisions of Artcile 106 of the Labor Code and not that of Article 208. The
former clearly defines what constitute labor-only contractor as differentiated
from a direct contractor, including the legal effects of each, while the latter is
merely for the purpose of determining whether or not an employee is
considered regular. Based on the provision of Article 106, the Supreme Court
ruled that the petitioner was indeed the direct employer of private
respondents and was therefore 0bliged to pay them separation pay. The
Supreme Court reasoned that since it is undeniable that the private
respondents' work as merchandisers, cashiers, baggers, check-out
personnel, sales ladies, warehousemen and so forth were directly related,
necessary and vital to the day-to-day operations of the supermarket and that
their jobs involved normal and regular functions in the ordinary business of
the petitioner corporation, the provision of Article 106 clearly applied thus
making the manpower agencies merely agents of petitioner corporation.
Consequently, private respondents are considered employees of petitioner
Shoppers Gain Supermart.
Caingat v. NLRC G.R. No. 154308, March 10, 2005
Facts: Petitioner Benardino A. Caingat was hired by respondent Sta. Lucia
Realty and Development, Inc. (SLRDI) as the General Manager of SLRDIs
sister companies, R.S. Night Hawk Security and Investigation Agency, Inc.,
and R.S. Maintenance and Services Inc. both organized to service the malls
and subdivisions owned by SLRDI. In connection with this, he was allowed to
use 10% of the total payroll of respondent R.S. Maintenance to defray
operating expenses. Later, the Finance Manager discovered that petitioner
deposited company funds in the latters personal account and used the funds

to pay his credit card purchases, utility bills, trips abroad and acquisition of a
lot in Laguna. Thus, complainant received a memorandum stating that upon
verification of financial records, it was found that the latter have
misappropriated company funds in the sum of about P5, 000,000.00 and is
hereby suspended from his duties as Manager of the stated companies.
Without conducting any investigation, respondent R.S. Maintenance filed a
complaint for sum of money and damages with prayer for writ of preliminary
attachment. Petitioner in turn filed a complaint for illegal dismissal against
the respondents.
Issue: Did respondents illegally dismiss petitioner?
Ruling: As firmly entrenched in our jurisprudence, loss of trust and confidence
as a just cause for termination of employment is premised on the fact that an
employee concerned holds a position where greater trust is placed by
management and from whom greater fidelity to duty is correspondingly
expected. This includes managerial personnel entrusted with confidence on
delicate matters, such as the custody, handling, or care and protection of the
employers property. The betrayal of this trust is the essence of the offense
for which an employee is penalized. Managements loss of trust and
confidence on petitioner was well justified. Private respondents had every
right to dismiss petitioner. Petitioners long period of disappearance from the
scene and departure for abroad before making a claim of illegal dismissal
does not contribute to its credibility. Nonetheless, while dismissal may truly
be justified by loss of confidence, the management failed to observe fully the
procedural requirement of due process for the termination of petitioners
employment. Two notices should be sent to the employee. The respondents
only sent the first notice, gleaned from the memorandum. There was no
second notice.
Wenphil Corporation v. NLRC G.R. No. 80587, February 8, 1989
Facts: Private respondent was hired by petitioner on January 18, 1984 as a
crew member at its Cubao Branch. He thereafter became the assistant head
of the Backroom department of the same branch. At about 2:30 P.M. on May
20, 1985 private respondent had an altercation with a co-employee, Job
Barrameda, as a result of which he and Barrameda were suspended on the
following morning and in the afternoon of the same day a memorandum was
issued by the Operations Manager advising private respondent of his
dismissal from the service in accordance with their Personnel Manual. The

notice of dismissal was served on private respondent on May 25, 1985. Thus
private respondent filed a complaint against petitioner for unfair labor
practice, illegal suspension and illegal dismissal. After submitting their
respective position papers to the Labor Arbiter and as the hearing could not
be conducted due to repeated absence of counsel for respondent, the case
was submitted for resolution. Thereafter a decision was rendered by the
Labor Arbiter on December 3, 1986 dismissing the complaint for lack of
merit.
Issue: Was there compliance with the procedural due process for dismissal?
Ruling: The failure of petitioner to give private respondent the benefit of a
hearing before he was dismissed constitutes an infringement of his
constitutional right to due process of law and equal protection of the laws.
The standards of due process in judicial as well as administrative
proceedings have long been established. In its bare minimum due process of
law simply means giving notice and opportunity to be heard before judgment
is rendered.
Maneja v. NLRC G.R. No. 124013, June 5, 1998
Facts: Petitioner Rosario Maneja worked with private respondent Manila
Midtown Hotel beginning January, 1985 as a telephone operator. She was a
member of the National Union of Workers in Hotels, Restaurants and Allied
Industries (NUWHRAIN) with an existing Collective Bargaining Agreement
(CBA) with private respondent. In the afternoon of February 13, 1990, a
fellow telephone operator, Rowena Loleng received a Request for Long
Distance Call (RLDC) form and a deposit of P500.00 from a page boy of the
hotel for a call by a Japanese guest named Hirota Ieda. The call was
unanswered. The P500.00 deposit was forwarded to the cashier. In the
evening, Ieda again made an RLDC and the page boy collected another
P500.00 which was also given to the operator Loleng. The second call was
also unanswered. Loleng passed on the RLDC to petitioner for follow-up.
Petitioner monitored the call. On February 15, 1990, a hotel cashier inquired
about the P1,000.00 deposit made by Ieda. After a search, Loleng found the
first deposit of P500.00 inserted in the guest folio while the second deposit
was eventually discovered inside the folder for cancelled calls with deposit
and official receipts. When petitioner saw that the second RLDC form was not
time-stamped, she immediately placed it inside the machine which stamped
the date February 15, 1990. Realizing that the RLDC was filed 2 days

earlier, she wrote and changed the date to February 13, 1990. Loleng then
delivered the RLDC and the money to the cashier. The second deposit of
P500.00 by Ieda was later returned to him. Petitioner and Loleng thereafter
submitted their written explanation. On March 20, 1990, a written report was
submitted by the chief telephone operator, with the recommendation that
the offenses committed by the operators concerned covered violations of the
Offenses Subject to Disciplinary Actions (OSDA): (1) OSDA 2.01: forging,
falsifying official document(s), and (2) OSDA 1.11: culpable carelessness negligence or failure to follow specific instruction(s) or established
procedure(s). On March 23, 1990, petitioner was served a notice of dismissal
effective April 1, 1990. Petitioner refused to sign the notice and wrote therein
"under protest."
Issue: Whether or not the Labor Arbiter has jurisdiction over the illegal
dismissal case.
Ruling: The procedure introduced in RA 6715 of referring certain grievances
originally and exclusively to the grievance machinery, and when not settled
at this level, to a panel of voluntary arbitrators outlined in CBAs does not
only include grievances arising from the interpretation or implementation of
the CBA but applies as well to those arising from the implementation of
company personnel policies. No other body shall take cognizance of these
cases. x x x. (Sanyo v. Caizares, 211 SCRA 361, 372) THE Court finds that
the respondent Commission has erroneously interpreted the aforequoted
portion of our ruling in the case of Sanyo, as divesting the Labor Arbiter of
jurisdiction in a termination dispute. Moreover, the dismissal of petitioner
does not fall within the phrase grievances arising from the interpretation or
implementation of collective bargaining agreement and those arising from
the interpretation or enforcement of company personnel policies, the
jurisdiction of which pertains to the grievance machinery or thereafter, to a
voluntary arbitrator or panel of voluntary arbitrators. It is to be stressed that
under Article 260 of the Labor Code, which explains the function of the
grievance machinery and voluntary arbitrator, (T)he parties to a Collective
Bargaining Agreement shall include therein provisions that will ensure the
mutual observance of its terms and conditions. They shall establish a
machinery for the adjustment and resolution of grievances arising from the
interpretation or implementation of their Collective Bargaining Agreement
and those arising from the interpretation or enforcement of company
personnel policies. Article 260 further provides that the parties to a CBA
shall name or designate their respective representative to the grievance

machinery and if the grievance is unsettled in that level,


automatically be referred to the voluntary arbitrators designated in
by the parties to a CBA of the union and the company. It can
deduced that only disputes involving the union and the company
referred to the grievance machinery or voluntary arbitrators.

it shall
advance
thus be
shall be

Eastern
Overseas
Employment
Center,
Inc.
v.
NagkakaisangxEmpleyadongWellcome-DFA G.R. No. 149349, March
11, 2005
Facts: On July 20, 1990, [respondent] union NAGKAKAISANG EMPLEYADO NG
WELLCOME-DFA (NEW-DFA) filed a Petition for Certification Election with the
DOLE-NCR seeking to represent the bargaining unit comprised of all the
regular rank-and-file employees of [petitioner] company GLAXO-WELLCOME.
Acting upon such petition, the Med-Arbiter ordered that a Certification
Election be conducted on September 10, 1990. The election, however,
resulted in a stand-off or a tie between NO UNION and NEW-DFA. As a
consequence thereof, NEW-DFA filed an election protest with the Medarbitration Branch of the Department of Labor and Employment.
[Respondent] union claimed that its failure to obtain the required majority
vote can be ascribed to several acts of manipulation, interference and
intimidation committed by [petitioner] company GLAXO-WELLCOME prior to
and during the conduct of the certification elections. Perceiving the
enumerated events to be unduly oppressive to labor, [respondent] union
NEW-DFA, Jossie De Guzman and Norman Cerezo lodged a complaint before
the Labor Arbiter against [petitioner] company, GLAXO-WELLCOME, for unfair
labor practice, illegal dismissal and illegal suspension. According to the
[respondent] union, the massive electioneering and manipulative acts of
GLAXO-WELLCOME prior to and during the certification election unduly
interfered with the workers right to self-organization and are constitutive of
unfair labor practice. NEW-DFA likewise averred that the new Car Allocation
Policy adopted by the company was intended to harass, retaliate and
discriminate against union officers and members. [Respondent] union also
challenged the legality of the suspension and dismissal of two of its officers,
namely: Norman Cerezo and Jossie Roda de Guzman. It argued that the
suspension and dismissal were effected without any prior hearing.
Issue: Did petitioner observe due process in dismissing the employees?

Ruling: The CA affirmed the findings of the labor arbiter and the NLRC that
the termination of the employment of De Guzman and the suspension of
Cerezo were based on a just cause. These findings are not at issue here.
The only question to be determined is whether the notice and hearing
requirements were complied with. To stress, if the dismissal is based on a just
cause under Article 282 of the Labor Code, the employer must give the
employee (1) two written notices and (2) a hearing (or at least, an
opportunity to be heard). The first notice is intended to inform the employee
of the employers intent to dismiss and the particular acts or omissions for
which the dismissal is sought. The second notice is intended to inform the
employee of the employers decision to dismiss. This decision, however,
must come only after the employee has been given a reasonable period,
from receipt of the first notice, within which to answer the charge; and ample
opportunity to be heard with the assistance of counsel, if the employee so
desires. The twin requirements of (a) two notices and (b) hearing are
necessary to protect the employees security of tenure, which is enshrined in
the Constitution, the Labor Code and related laws. The notices to be given
and the hearing to be conducted generally constitute the two-part due
process requirement of law that the employer must accord the employee. In
Kingsize Manufacturing Corporation v. NLRC, the Court held that this
requirement was not a mere technicality but a requirement of due process
to which every employee is entitled to insure that the employers prerogative
to dismiss or lay-off is not abused or exercised in an arbitrary manner.
Development of Doctrines
Wenphil Corporation v. NLRC G.R. No. 80587 February 8, 1989, En
Banc
Facts: Private respondent was hired by petitioner on January 18, 1984 as a
crew member at its Cubao Branch. He thereafter became the assistant head
of the Backroom department of the same branch. At about 2:30 P.M. on May
20, 1985 private respondent had an altercation with a co-employee, Job
Barrameda, as a result of which he and Barrameda were suspended on the
following morning and in the afternoon of the same day a memorandum was
issued by the Operations Manager advising private respondent of his
dismissal from the service in accordance with their Personnel Manual. The
notice of dismissal was served on private respondent on May 25, 1985. Thus
private respondent filed a complaint against petitioner for unfair labor
practice, illegal suspension and illegal dismissal. After submitting their

respective position papers to the Labor Arbiter and as the hearing could not
be conducted due to repeated absence of counsel for respondent, the case
was submitted for resolution. Thereafter a decision was rendered by the
Labor Arbiter on December 3, 1986 dismissing the complaint for lack of
merit.
Issue: Was the dismissal proper without compliance with the requirements of
due process?
Ruling: The failure of petitioner to give private respondent the benefit of a
hearing before he was dismissed constitutes an infringement of his
constitutional right to due process of law and equal protection of the laws. 2
The standards of due process in judicial as well as administrative
proceedings have long been established. In its bare minimum due process of
law simply means giving notice and opportunity to be heard before judgment
is rendered. 3 The claim of petitioner that a formal investigation was not
necessary because the incident which gave rise to the termination of private
respondent was witnessed by his co- employees and supervisors is without
merit. The basic requirement of due process is that which hears before it
condemns, which proceeds upon inquiry and renders judgment only after
trial. However, it is a matter of fact that when the private respondent filed a
complaint against petitioner he was afforded the right to an investigation by
the labor arbiter. He presented his position paper as did the petitioner. If no
hearing was had, it was the fault of private respondent as his counsel failed
to appear at the scheduled hearings. The labor arbiter concluded that the
dismissal of private respondent was for just cause. He was found guilty of
grave misconduct and insubordination. This is borne by the sworn
statements of witnesses. The Court is bound by this finding of the labor
arbiter.
Serrano v. NLRC G.R. No. 117040, May 4, 2000, En Banc
Facts: Respondent Isetann Department Store moves for reconsideration of
the decision in this case insofar as it is ordered to pay petitioner full
backwages from the time the latter's employment was terminated on
October 11, 1991 up to the time it is determined that the termination of
employment is for an authorized cause. The motion is opposed by petitioner.
The decision is based on private respondent's failure to give petitioner a
written notice of termination at least thirty (30) days before the termination
of his employment as required by Art. 283 the Labor Code. Kycalr In support

of its motion, private respondent puts forth three principal arguments, to wit:
(1) that its failure to give a written notice to petitioner at least thirty (30)
days in advance in accordance with Art. 283 of the Labor Code is not in issue
in this case because, as a matter of fact, it gave its employees in the
affected security section thirty (30) days pay which effectively gave them
thirty (30) days notice, and petitioner accepted this form of notice although
he did not receive payment; (2) that payment of thirty (30) days pay in lieu
of the thirty (30) days prior formal notice is more advantageous to an
employee because instead of being required to work for thirty (30) days, the
employee can look for another job while being paid by the company; and (3)
that in any event the new ruling announced in this case should only be
applied prospectively.
Issue: Was the dismissal valid?
Ruling: The decision in Columbia Pictures does not mean that if a new rule is
laid down in a case, it should not be applied in that case but that said rule
should apply prospectively to cases arising afterwards. Private respondent's
view of the principle of prospective application of new judicial doctrines
would turn the judicial function into a mere academic exercise with the result
that the doctrine laid down would be no more than a dictum and would
deprive the holding in the case of any force. Indeed, when the Court
formulated the Wenphil doctrine, which we reverse in this case, the Court did
not defer application of the rule laid down imposing a fine on the employer
for failure to give notice in a case of dismissal for cause. To the contrary, the
new rule was applied right then and there. For that matter, in 20th Century
Fox Film Corp. v. Court of Appeals the Court laid down the rule that in
determining the existence of probable cause for the issuance of a search
warrant in copyright infringement cases, the court must require the
production of the master tapes of copyrighted films in order to compare
them with the "pirated" copies. The new rule was applied in opinion of the
Court written by Justice Hugo E. Gutierrez, Jr. in the very same case of 20th
Century Fox in which the new requirement was laid down. Where the new
rule was held to be prospective in application was in Columbia Pictures and
that was because at the time the search warrant in that case was issued, the
new standard had not yet been announced so it would be unreasonable to
expect the judge issuing the search warrant to apply a rule that had not been
announced at the time. A good illustration of the scope of overruling
decisions is People v. Mapa, where the accused was charged with illegal
possession of firearms. The accused invoked the ruling in an earlier case that

appointment as a secret agent of a provincial governor to assist in the


maintenance of peace and order sufficiently put the appointee in the
category of a "peace officer" equal to a member of the municipal police
authorized under 879 of the Administrative Code of 1917 to carry firearms.
The Court rejected the accused's contention and overruled the prior decision
in People v. Macarandang on the ground that 879 of the Administrative
Code of 1917 was explicit and only those expressly mentioned therein were
entitled to possess firearms. Since secret agents were not among those
mentioned, they were not authorized to possess firearms. Although in People
v. Jabinal the Court refused to give retro active effect to its decision in Mapa,
because the new doctrine "should not apply to parties who had relied on the
old doctrine and acted in good faith thereon" and, for this reason, it
acquitted the accused of illegal possession of firearms, nonetheless it applied
the new ruling (that secret agents of provincial governors were not
authorized to possess firearms) in the very case in which the new rule was
announced and convicted the accused. In the case at bar, since private
respondent does not even claim that it has relied in good faith on the former
doctrine of Wenphil and its progeny Sebuguero v. NLRC, there is no reason
not to apply the new standard to this case.
Agabon v. NLRC G.R. No. 158693, November 17, 2004, En Banc
Facts: Private respondent Riviera Home Improvements, Inc. is engaged in the
business of selling and installing ornamental and construction materials. It
employed petitioners Virgilio Agabon and Jenny Agabon as gypsum board
and cornice installers on January 2, 1992 until February 23, 1999 when they
were dismissed for abandonment of work. Petitioners then filed a complaint
for illegal dismissal and payment of money claims and on December 28,
1999, the Labor Arbiter rendered a decision declaring the dismissals illegal
and ordered private respondent to pay the monetary claims.
Issue: Was the termination valid?
Ruling: The rule thus evolved: where the employer had a valid reason to
dismiss an employee but did not follow the due process requirement, the
dismissal may be upheld but the employer will be penalized to pay an
indemnity to the employee. This became known as the Wenphil or Belated
Due Process Rule. On January 27, 2000, in Serrano, the rule on the extent of
the sanction was changed. We held that the violation by the employer of the
notice requirement in termination for just or authorized causes was not a

denial of due process that will nullify the termination. However, the dismissal
is ineffectual and the employer must pay full backwages from the time of
termination until it is judicially declared that the dismissal was for a just or
authorized cause. The rationale for the re-examination of the Wenphil
doctrine in Serrano was the significant number of cases involving dismissals
without requisite notices. We concluded that the imposition of penalty by
way of damages for violation of the notice requirement was not serving as a
deterrent. Hence, we now required payment of full backwages from the time
of dismissal until the time the Court finds the dismissal was for a just or
authorized cause. Serrano was confronting the practice of employers to
dismiss now and pay later by imposing full backwages. We believe,
however, that the ruling in Serrano did not consider the full meaning of
Article 279 of the Labor Code which states: ART. 279. Security of Tenure. In
cases of regular employment, the employer shall not terminate the services
of an employee except for a just cause or when authorized by this Title. An
employee who is unjustly dismissed from work shall be entitled to
reinstatement without loss of seniority rights and other privileges and to his
full backwages, inclusive of allowances, and to his other benefits or their
monetary equivalent computed from the time his compensation was
withheld from him up to the time of his actual reinstatement. This means
that the termination is illegal only if it is not for any of the justified or
authorized causes provided by law. Payment of backwages and other
benefits, including reinstatement, is justified only if the employee was
unjustly dismissed.

Reinstatement to previous position or substantially equivalent position


Asian Terminals, Inc., v. Villanueva G.R. No. 143219, November 28,
2006
Facts: Respondents were employees of Marina Port Services, Inc. (MPSI) and
members of the Associated Workers Union of the Philippines (AWU). In a
letter dated 9 June 1993 to MPSI, the AWU president sought the dismissal
from service of respondents who were expelled from AWU. On 11 June 1993,
the MPSI issued a memorandum to respondents terminating them effective

immediately pursuant to the closed-shop provision of the MPSI-AWU


Collective Bargaining Agreement. Respondents filed a complaint for
constructive illegal dismissal and unfair labor practice with the Arbitration
Branch of the NLRC.
Issue: Whether MPSI reinstated respondents to their former or equivalent
positions.
Ruling: Reinstatement means restoration to a state or condition from which
one had been removed or separated. The person reinstated assumes the
position he had occupied prior to his dismissal. Reinstatement presupposes
that the previous position from which one had been removed still exists, or
that there is an unfilled position which is substantially equivalent or of similar
nature as the one previously occupied by the employee. Reinstatement
means restoration to the former position occupied prior to dismissal or to
substantially equivalent position. Reinstatement does not mean promotion.
Promotion is based primarily on an employees performance during a certain
period. Just because their contemporaries are already occupying higher
positions does not automatically entitle respondents to similar positions.
Separation pay as financial assistance
PLDT v. NLRC G.R. No. L-80609, August 23, 1988, En Banc
Facts: Marilyn Abucay, a traffic operator of the Philippine Long Distance
Telephone Company, was accused by two complainants of having demanded
and received from them the total amount of P3,800.00 in consideration of
her promise to facilitate approval of their applications for telephone
installation. 1 Investigated and heard, she was found guilty as charged and
accordingly separated from the service. She went to the Ministry of Labor
and Employment claiming she had been illegally removed. After
consideration of the evidence and arguments of the parties, the company
was sustained and the complaint was dismissed for lack of merit.
Issue: Should she be awarded with separation pay?
Ruling: The Court holds that henceforth separation pay shall be allowed as a
measure of social justice only in those instances where the employee is
validly dismissed for causes other than serious misconduct or those
reflecting on his moral character. Where the reason for the valid dismissal is,

for example, habitual intoxication or an offense involving moral turpitude,


like theft or illicit sexual relations with a fellow worker, the employer may not
be required to give the dismissed employee separation pay, or financial
assistance, or whatever other name it is called, on the ground of social
justice. A contrary rule would, as the petitioner correctly argues, have the
effect, of rewarding rather than punishing the erring employee for his
offense. And we do not agree that the punishment is his dismissal only and
that the separation pay has nothing to do with the wrong he has committed.
Of course it has. Indeed, if the employee who steals from the company is
granted separation pay even as he is validly dismissed, it is not unlikely that
he will commit a similar offense in his next employment because he thinks
he can expect a like leniency if he is again found out. This kind of misplaced
compassion is not going to do labor in general any good as it will encourage
the infiltration of its ranks by those who do not deserve the protection and
concern of the Constitution.
Separation pay in lieu of reinstatement Strained relation rule
Golden Ace Builders v. Talde G.R. No. 187200, May 5, 2010
Facts: In 1990, Golden Ace Builders hired Jose A. Talde (Talde) as a carpenter.
In February 1999, the owner-manager, Arnold Azul, stopped giving Talde
work assignment due allegedly to the unavailability of construction projects.
Consequently, Talde filed a complaint for illegal dismissal. The Labor Arbiter
ruled in Taldes favor and ordered his immediate reinstatement without loss
of seniority rights, with payment of full backwages as well as premium pay
for rest days, service incentive leave pay and 13th month pay. The company
brought the case to the National Labor Relations Commission (NLRC) for
review. Pending such appeal, the company advised Talde to report for work
within 10 days from notice. Talde, however, manifested to the Labor Arbiter
that due to actual animosity between him and the company and threats to
his life and his familys safety, he opted for payment of separation pay. The
company denied there was such an animosity. The NLRC later dismissed the
companys appeal. The companys appeal to the Court of Appeals was
likewise dismissed. The Court of Appeals decision attained finality. The
monetary award, as recomputed by the NLRCs Fiscal Examiner, was
approved by the Labor Arbiter who thereupon issued the writ of execution.
The company questioned the recomputation before the NLRC, arguing that
since Talde refused to report back to work as the company advised, he
should be deemed to have abandoned the same, thus, the re-computation

should not be beyond 15 May 2001, the day he manifested his refusal to be
reinstated. T he NLRC vacated the re-computation, holding that since Talde
did not appeal the Labor Arbiters decision granting him only reinstatement
and backwages, not separation pay in lieu of reinstatement, he may not be
afforded affirmative relief, and since he refused to go back to work, he may
recover backwages only up to 20 May 2001, the day he was supposed to
return to the job site. When Taldes motion for reconsideration was denied by
the NLRC, he filed a petition for certiorari with the Court of Appeals. The
Court of Appeals set aside the NLRC findings and held that Talde was entitled
to both backwages and separation pay, even if separation pay was not
granted by the Labor Arbiter, in view of the strained relations between the
parties. Consequently, the company filed a petition for review on certiorari
before the Supreme Court.
Issue: Whether or not Talde was entitled to separation pay in lieu of actual
reinstatement on account of strained relations between him and the
company.
Ruling: An illegally dismissed employee is entitled to two reliefs: backwages
and reinstatement. The two reliefs are separate and distinct. When
reinstatement is no longer feasible because of strained relations between the
employee and the employer, separation pay equivalent to one (1) month
salary for every year of service should be awarded as an alternative. The
payment of separation pay is in addition to payment of backwages. In effect,
an illegally dismissed employee is entitled to either reinstatement, if viable,
or separation pay if reinstatement is nolonger viable, and backwages. (Citing
Macasero v. Southern Industrial Gases Philippines, G.R. No. 178524; 30
January 2009) Under the doctrine of strained relations, the payment of
separation pay is considered an acceptable alternative to reinstatement
when the latter option is no longer desirable or viable. On one hand, such
payment liberates the employee from what could be a highly oppressive
work environment. On the other hand, it releases the employer from the
grossly unpalatable obligation of maintaining in its employ a worker it could
no longer trust. Strained relations must be demonstrated as a fact and must
be supported by substantial evidence showing that the relationship between
the employer and the employee is indeed strained as a necessary
consequence of the judicial controversy. In this case, the Labor Arbiter found
that actual animosity existed between the owner-manager Azul and Talde as
a result of the filing of the illegal dismissal case. Such finding, especially
when affirmed by the appellate court as in the case at bar, is binding upon

the Supreme Court, consistent with the prevailing rules that the Supreme
Court will not try facts anew and that findings of facts of quasi-judicial bodies
are accorded great respect, even finality. The Court of Appeals erroneously
computed his separation pay from 1990 (when he was hired to 1999 (when
he was unjustly dismissed), covering a period of 8 years. He must be
considered to have been in the service of the company not only until 1999,
but until 30 June 2005, the day he is deemed to have been actually
separated (his reinstatement having been rendered impossible) from the
company, or for a total of 15 years.

Leopard Security and Investigation Agency v. Quitoy G.R. No.


186344, February 20, 2013
Facts: Alongside Numeriano Ondong, respondents Tomas Quitoy, Raul Sabang
and Diego Morales were hired as security guards by petitioner Leopard
Security and Investigation Agency (LSIA). Thru its representative, Rogelio
Morales, LSIA informed respondents on 29 April 2005 of the termination of its
contract with Union Bank which had decided to change its security provider.
Upon Morales instruction, respondents went to the Union Bank Cebu
Business Park Branch on 30 April 2005, for the turnover of their service
firearms to Arnel Cortes, Union Banks Chief Security Officer. On 3 May 2005,
respondents and Ondong filed a complaint for illegal dismissal, unpaid 13th
month pay and service incentive leave pay (SILP), moral and exemplary
damages as well as attorneys fees.
Issue: Are respondents entitled to separatin pay and reinstatement?
Ruling: As a relief granted in lieu of reinstatement, however, it consequently
goes without saying that an award of separation pay is inconsistent with a
finding that there was no illegal dismissal. Standing alone, the doctrine of
strained relations will not justify an award of separation pay, a relief granted
in instances where the common denominator is the fact that the employee
was dismissed by the employer. Even in cases of illegal dismissal, the
doctrine of strained relations is not applied indiscriminately as to bar
reinstatement, especially when the employee has not indicated an aversion
to returning to work or does not occupy a position of trust and confidence in
or has no say in the operation of the employers business. Although litigation
may also engender a certain degree of hostility, it has likewise been ruled
that the understandable strain in the parties relations would not necessarily

rule out reinstatement which would, otherwise, become the rule rather than
the exception in illegal dismissal cases. Absent illegal dismissal on the part of
LSIA and abandonment of employment on the part of respondents, we find
that the latters reinstatement without backwages is, instead, in order. In
addition to respondents alternative prayer therefor in their position paper,
reinstatement is justified by LSIAs directive for them to report for work at its
Mandaluyong City office as early of 10 May 2005. As for the error ascribed
the CA for failing to correct the NLRCs disregard of the evidence showing
LSIAs payment of respondents SILP, suffice it to say that the NLRC is not
precluded from receiving evidence, even for the first time on appeal,
because technical rules of procedure are not binding in labor cases.
Considering that labor officials are, in fact, encouraged to use all reasonable
means to ascertain the facts speedily and objectively, with little resort to
technicalities of law or procedure, LSIA correctly faults the CA for likewise
brushing aside the evidence of SILP payments it submitted during the appeal
stage before the NLRC.

Backwages and Separation Pay, Distinguished


Golden Ace Builders v. Talde G.R. No. 187200, May 5, 2010
Facts: In 1990, Golden Ace Builders hired Jose A. Talde (Talde) as a carpenter.
In February 1999, the owner-manager, Arnold Azul, stopped giving Talde
work assignment due allegedly to the unavailability of construction projects.
Consequently, Talde filed a complaint for illegal dismissal. The Labor Arbiter
ruled in Taldes favor and ordered his immediate reinstatement without loss
of seniority rights, with payment of full backwages as well as premium pay
for rest days, service incentive leave pay and 13th month pay. The company
brought the case to the National Labor Relations Commission (NLRC) for
review. Pending such appeal, the company advised Talde to report for work
within 10 days from notice. Talde, however, manifested to the Labor Arbiter
that due to actual animosity between him and the company and threats to
his life and his familys safety, he opted for payment of separation pay. The
company denied there was such an animosity. The NLRC later dismissed the
companys appeal. The companys appeal to the Court of Appeals was
likewise dismissed. The Court of Appeals decision attained finality. The
monetary award, as recomputed by the NLRCs Fiscal Examiner, was

approved by the Labor Arbiter who thereupon issued the writ of execution.
The company questioned the recomputation before the NLRC, arguing that
since Talde refused to report back to work as the company advised, he
should be deemed to have abandoned the same, thus, the re-computation
should not be beyond 15 May 2001, the day he manifested his refusal to be
reinstated. The NLRC vacated the re-computation, holding that since Talde
did not appeal the Labor Arbiters decision granting him only reinstatement
and backwages, not separation pay in lieu of reinstatement, he may not be
afforded affirmative relief, and since he refused to go back to work, he may
recover backwages only up to 20 May 2001, the day he was supposed to
return to the job site. When Taldes motion for reconsideration was denied by
the NLRC, he filed a petition for certiorari with the Court of Appeals. The
Court of Appeals set aside the NLRC findings and held that Talde was entitled
to both backwages and separation pay, even if separation pay was not
granted by the Labor Arbiter, in view of the strained relations between the
parties. Consequently, the company filed a petition for review on certiorari
before the Supreme Court.
Issue: Is respondent entitled to backwages?
Ruling: The basis for the payment of backwages is different from that for the
award of separation pay. Separation pay is granted where reinstatement is
no longer advisable because of strained relations between the employee and
the employer. Backwages represent compensation that should have been
earned but were not collected because of the unjust dismissal. The basis for
computing backwages is usually the length of the employees service while
that for separation pay is the actual period when the employee was
unlawfully prevented from working. As to how both awards should be
computed, Macasero v. Southern Industrial Gases Philippines instructs: [T]he
award of separation pay is inconsistent with a finding that there was no
illegal dismissal, for under Article 279 of the Labor Code and as held in a
catena of cases, an employee who is dismissed without just cause and
without due process is entitled to backwages and reinstatement or payment
of separation pay in lieu thereof: Thus, an illegally dismissed employee is
entitled to two reliefs: backwages and reinstatement. The two reliefs
provided are separate and distinct. In instances where reinstatement is no
longer feasible because of strained relations between the employee and the
employer, separation pay is granted. In effect, an illegally dismissed
employee is entitled to either reinstatement, if viable, or separation pay if
reinstatement is no longer viable, and backwages. The normal consequences

of respondents illegal dismissal, then, are reinstatement without loss of


seniority rights, and payment of backwages computed from the time
compensation was withheld up to the date of actual reinstatement. Where
reinstatement is no longer viable as an option, separation pay equivalent to
one (1) month salary for every year of service should be awarded as an
alternative. The payment of separation pay is in addition to payment of
backwages.

Constructive dismissal
Uniwide Sales Warehouse Club v. NLRC G.R. No. 154503, February
29, 2008
Facts: Amalia P. Kawada (private respondent) started her employment with
Uniwide sometime in 1981 as a saleslady. Over the years, private respondent
worked herself within Uniwides corporate ladder until she attained the rank
of Full Assistant Store Manager with a monthly compensation of P13,000.00
in 1995. As a Full Assistant Store Manager, private respondents primary
function was to manage and oversee the operation of the Fashion and
Personal Care, GSR Toys, and Home Furnishing Departments of Uniwide, to
ensure its continuous profitability as well as to see to it that the established
company policies and procedures were properly complied with and
implemented in her departments. Sometime in 1998, Uniwide received
reports from the other employees regarding some problems in the
departments managed by the private respondent. Thus, on March 15, 1998,
Uniwide, through Store Manager Apduhan, issued a Memorandum addressed
to the private respondent summarizing the various reported incidents
signifying unsatisfactory performance on the latters part which include the
commingling of good and damaged items, sale of a voluminous quantity of
damaged toys and ready-to-wear items at unreasonable prices, and failure to
submit inventory reports. Uniwide asked private respondent for concrete
plans on how she can effectively perform her job. In a letterdated March 23,
1998, private respondent answered all the allegations contained in the March

15, 1998 Memorandum. On June 30, 1998, Apduhan sent another


Memorandum seeking from the private respondent an explanation regarding
the incidents reported by Uniwide employees and security personnel for
alleged irregularities committed by the private respondent such as allowing
the entry of unauthorized persons inside a restricted area during non-office
hours, falsification of or inducing another employee to falsify personnel or
company records, sleeping and allowing a non-employee to sleep inside the
private office, unauthorized search and bringing out of company records,
purchase of damaged home furnishing items without the approval from
superior, taking advantage of buying damaged items in large quantity,
alteration of approval slips for the purchase of damaged items and
abandonment of work. In a letter dated July 9, 1998, private respondent
answered the allegations made against her.
Issue: Was there constructive dismissal in this case?
Ruling: The test of constructive dismissal is whether a reasonable person in
the employees position would have felt compelled to give up his position
under the circumstances. It is an act amounting to dismissal but made to
appear as if it were not. In fact, the employee who is constructively
dismissed may be allowed to keep on coming to work. Constructive dismissal
is therefore a dismissal in disguise. The law recognizes and resolves this
situation in favor of employees in order to protect their rights and interests
from the coercive acts of the employer. In the present case, private
respondent claims that from the months of February to June 1998, she had
been subjected to constant harassment, ridicule and inhumane treatment by
Apduhan, with the hope that the latter can get the private respondent to
resign. The harassment allegedly came in the form of successive memoranda
which private respondent would receive almost every week, enumerating a
litany of offenses and maligning her reputation and spreading rumors among
the employees that private respondent shall be dismissed soon. The last
straw of the imputed harassment was the July 31, 1998 incident wherein
private respondents life was put in danger when she lost consciousness due
to hypertension as a result of Apduhans alleged hostility and shouting. The
Court finds that private respondents allegation of harassment is a specious
statement which contains nothing but empty imputation of a fact that could
hardly be given any evidentiary weight by this Court. Private respondents
bare allegations of constructive dismissal, when uncorroborated by the
evidence on record, cannot be given credence.

Globe Telecom v. Florendo-Flores G.R. No. 150092, September 27,


2002
Facts: Petitioner GLOBE TELECOM, INC. (GLOBE) is a corporation duly
organized and existing under the laws of the Philippines. Petitioners Delfin
Lazaro Jr. was its President and Roberto Galang its former Director-Regional
Sales. Respondent Joan Florendo-Flores was the Senior Account Manager for
Northern Luzon. On 1 July 1998 Joan Florendo-Flores filed with the Regional
Arbitration Branch of the National Labor Relations Commission (NLRC) an
amended complaint for constructive dismissal against GLOBE, Lazaro,
Galang, and Cacholo M. Santos, her immediate superior, Luzon HeadRegional Sales. In her affidavit submitted as evidence during the arbitration
proceedings, Florendo-Flores bared that Cacholo M. Santos never
accomplished and submitted her performance evaluation report thereby
depriving her of salary increases, bonuses and other incentives which other
employees of the same rank had been receiving; reduced her to a house-tohouse selling agent (person-to-person sales agent or direct sales agent) of
company products ("handyphone") despite her rank as supervisor of
company dealers and agents; never supported her in the sales programs and
recommendations she presented; and, withheld all her other benefits, i.e.,

gasoline allowance, per diems, representation


maintenance, to her extreme pain and humiliation.

allowance,

and

car

Issue: Was there constructive dismissal in this case?


Ruling: Where there is conflict between the dispositive portion of the decision
and the body thereof, the dispositive portion controls irrespective of what
appears in the body.[14] While the body of the decision, order or resolution
might create some ambiguity in the manner the court's reasoning
preponderates, it is the dispositive portion thereof that finally invests rights
upon the parties, sets conditions for the exercise of those rights, and
imposes the corresponding duties or obligations. Hence, for the Court of
Appeals to have affirmed the assailed judgment is to adopt and uphold the
NLRC finding of abandonment and its award of full back wages to respondent
as an "act of grace" from petitioners. However, the Court believes this is not
the proper view as the records reveal that respondent was constructively
dismissed from service. Constructive dismissal exists where there is
cessation of work because "continued employment is rendered impossible,
unreasonable or unlikely, as an offer involving a demotion in rank and a
diminution in pay." All these are discernible in respondent's situation. She
was singularly edged out of employment by the unbearable or undesirable
treatment she received from her immediate superior Cacholo M. Santos who
discriminated against her without reason - not preparing and submitting her
performance evaluation report that would have been the basis for her
increased salary; not forwarding her project proposals to management that
would have been the source of commendation; diminishing her supervisor
stature by assigning her to house-to-house sales or direct sales; and
withholding from her the enjoyment of bonuses, allowances and other similar
benefits that were necessary for her efficient sales performance. Although
respondent continued to have the rank of a supervisor, her functions were
reduced to a mere house-to-house sales agent or direct sales agent. This was
tantamount to a demotion. She might not have suffered any diminution in
her basic salary but petitioners did not dispute her allegation that she was
deprived of all benefits due to another of her rank and position, benefits
which she apparently used to receive.

You might also like